You are on page 1of 92

REG 1 ALL Files AND Reviewers naixvamzbx

bauzvajxubamcualoyxbanxyqnuxvamxuyabcnxiapabcywneyxvqmzuxvwmwodywbaoxybbquxv
hayxbwj
BS in Accountancy (BSA)

1. There is no negotiorum gestio in one of these instances:


I. When the property or business is not neglected or abandoned. 11. Choose.
II. If in fact the manager has been tacitly authorized by the owner. I. If the thing is determinate, the debtor can be compelled to deliver the thing
a. Both are true promised and upon failure, the creditor has a right to ask for damages.
b. Only II is true II. If the object is generic and the debtor does not comply with the
c. Both are false obligation, the creditor can ask a third person to comply with the prestation
d. Only I is true at the expense of the debtor, plus damages.
a. False; True
2. Synonymous to obligor b. True; False
I. Creditor c. False; False
II. Active subject d. True; True
III. Debtor
IV. Passive subject 12. The obligor is not liable for a fortuitous event when
a. III and IV a. The object is generic.
b. I and II b. The object is determinate.
c. I and IV c. He promised to deliver the same thing to two or more persons.
d. II and III d. There is default or mora on his part.

3. A is obliged to deliver a Rolex watch to B on December 1, 2019. 13. One is not a requisite needed in order that the obligation shall be
a. A is not required to exercise the diligence of a good father of a family. extinguished by loss or destruction of the thing due:
b. Before December 1, 2019, A has to take care of the Seiko watch with the a. When the thing lost is generic.
diligence of a good father of the family. b. When the thing is lost without the fault of the debtor.
c. In case of loss of the Seiko watch by negligence of A, A is liable in damages. c. When the thing lost is specific.
d. The obligation of A to B is not valid. d. When the thing is lost before the debtor has incurred in delay.

4. In an obligation to deliver a generic thing: 14. Choose.


a. The creditor has the right of choice. Statement 1: Delay or mora in one of the sources of obligation.
b. The debtor is not liable for fraud or delay or negligence in the performance Statement 2: There is no delay in an obligation not to do.
of the obligation. a. False; True
c. The debtor can choose which thing pertaining to the class to deliver. b. True; True
d. Loss by fortuitous event extinguishes the obligation. c. True; False
d. False; False
5. A binds himself to pay B P1,000 on December 10, 2019.
a. If December 10 has elapsed but A has not paid B, A is in delay. 15. Demand is not needed to put debtor in default except
b. A shall be liable to pay B P1,000 plus legal interests. a. When demand would be useless as when the obligor has rendered it beyond
c. A is not in delay unless B has made a demand. his power to perform.
d. If the P1,000 is lost by fortuitous event the obligation is extinguished. b. When time is of the essence of the contract.
c. When the thing is lost due to fortuitous event.
6. A agreed to B not to construct a fence between their 2 houses. d. When the law so provides.
a. This is a negative real obligation.
b. A will never be in delay. 16. Who is liable for the loss of the subject matter by fortuitous event?
c. The obligation is not valid. a. Both creditor and debtor
d. This is valid but cannot be enforced. b. Creditor
c. None of them.
7. Not a source of liability for damages: d. Debtor
a. Dolo incidente
b. Dolo causante 17. When the subject matter of the contract is lost through a fortuitous event,
c. Culpa aquiliana who is liable?
d. Mora accipiendi a. Both creditor and debtor
b. The debtor
8. Unless the law or stipulation of the parties requires another standard of care, c. The creditor
the obligation to give a thing carries with it the obligation to take care of it d. None of them
with:
a. Diligence of a good father of a family 18. No person shall be responsible for events which could not be foreseen, or
b. Extra-ordinary diligence which, though foreseen, were inevitable, except:
c. Ordinary diligence a. All of the above.
d. Answer not given b. When the nature of the obligation requires the assumption of risk.
c. When the law expressly provides for the debtor’s liability even in cases of
9. The creditor has a right to the fruits of the thing from fortuitous events.
a. The time the thing is delivered. d. When the subject of the obligation is a generic thing.
b. The time the obligation to deliver the thing arises. e. When the debtor is guilty of delay.
c. The time the fruits are delivered.
d. The time the sale is perfected. 19. D obliged himself to give a specific house to C on April 15, 2010, stipulating
that D is liable even if the house is lost through a fortuitous event, and
10. Which of the following is a determinate thing? without the need of a demand. On the due date, the house was destroyed by
a. A car. a typhoon. Which of the following is correct?
b. A 2010 Mitsubishi adventure GSL Sport DSL a. The obligation remains to subsist but converted into monetary
c. A car owned by Pedro Cruz. consideration.
d. A Toyota Revo Sport Runner with Plate No. XCS 888. b. The obligation is totally extinguished.
c. C can compel D to deliver another house.
d. C can require another person to deliver a house at the expense of D.

0 0
c. Reimbursement due the person who saved property during fire or storm
without the knowledge of the owner
d. Solutio indebiti
20. A bound himself to deliver a determinate horse to B on January 15, 2011. e. None of the above.
On January 16, 2011 the horse was struck and killed by a lightning.
a. A’s obligation is extinguished.
b. B may claim damages from A. 31. When A voluntarily takes charge of the neglected business of B
c. A has the obligation to replace the horse. without the latter’s authority where reimbursement must be made for
d. A is liable for the loss of the horse because he was in delay. necessary and useful expenses, there is a
a. Quasi contract
21. A obliged himself to deliver a specific cow to B which will be butchered b. Solutio indebiti
and served to the latter’s guest on the occasion of his wedding on June 20. c. Quasi delict
On June 20 A did not deliver the cow. The following day, it was killed due d. Negotiorum gestio
to a flood that occurred in their place.
a. A is not liable because the cause of the loss is a fortuitous event. 32. A juridical relation known as negotiorum gestio takes place
b. A is obliged to replace the cow. a. None of the above.
c. A’s obligation has been extinguished. b. When a person voluntarily takes charge of another’s abandoned
d. A is liable for the loss of the cow because he was in delay. business or property without the owner’s consent.
c. When a person is appointed by a court to take the property or business
22. It is not a source of liability which will entitle the injured party to damages: of another.
a. Culpa aquiliana or negligence committed in the performance of a d. When something is received and there is no right to demand it and it
spontaneous act. was delivered through mistake.
b. Mora or delay.
c. Contravention of the tenor of obligation. 33. All of the following except one are civil liabilities arising from crime.
d. Dishonesty, malice or bad faith in the performance of an existing valid a. Imprisonment
obligation. b. Indemnification for consequential damages
c. Restitution
23. Which of the following is not one of the sources of liability for damages? d. Reparation of the damage caused
a. Dolo causante
b. Delay 34. A civil liability which involves the return of the object of the crime
c. Fraud whenever possible, plus allowances for any deterioration or
d. Negligence diminution of value as may be determined by the court to the rightful
owner –
24. Which of the following is not an exception to the rule that “all rights a. Restitution
acquired in virtue of an obligation are transmissible”? b. Imprisonment
a. When the parties agreed against its transmission. c. Indemnification
b. When the nature of the obligation is purely personal. d. Reparation
c. When the obligation is real.
d. When the law prohibits the transmission of rights. 35. Culpa aquiliana as distinguished from culpa contractual
a. Proof of due diligence in the selection and supervision of employees
25. The duty not to recover what has been voluntarily paid although payment is not available as a defense.
was no longer required: b. The negligence of the defendant is merely an incident in the
a. Civil obligation performance of the obligation.
b. Moral obligation c. The source of liability is the defendant’s negligent act or omission
c. Natural obligation itself.
d. None of the above d. Proof of the contact and its breach is sufficient prima facie to warrant
recovery.
26. It is one of the essential elements of an obligation
a. An object or prestation 36. A wrong committed independent of contract and without criminal
b. Money intent is –
c. A maker or drawer a. Culpa aquiliana
d. A drawee b. Quasi-contract
c. Delict
27. Three of the following are requisites of an obligation. Which is the d. Culpa contractual
exception?
a. Prestation 37. A wrong committed independent of a contract and with criminal
b. Efficient cause intent –
c. Passive subject a. Quasi-contract
d. Delivery b. Culpa contractual
c. None of the above
28. Which of the following is not an obligation arising from law? d. Culpa aquiliana
a. Quasi-delict
b. Quasi-contract 38. A borrowed money from B payable on December 10, 2008. If A
c. Delict failed to pay on due date. Will A be in delay?
d. Contract a. Yes, if the obligation is in writing.
b. Yes, because there is stipulation as regards the due date.
29. The obligation of the employer to pay death benefits and funeral expenses c. No, if A has the money to pay B.
for his employee’s death while in the course of employment as sanctioned d. No, because demand has not been made by B.
by the Workmen’s Compensation Act is one that arises from:
a. Answer not given 39. Damages awarded to vindicate a right.
b. Contracts a. Exemplary
c. Quasi-contracts b. Liquidated
d. Law c. Actual
d. Nominal
30. Which of the following is not considered as quasi-contract?
a. When the third person, without the knowledge of the debtor, pays the debt. 40. Damages awarded to set an example.
b. Negotiorum gestio a. Exemplary

0 0
b. Moral d. Accion subrogatoria
c. Liquidated
d. Nominal 51. Latin term for delay 18.
mora

52. Latin term for fraud


dolo

41. When the exact amount of damages cannot be ascertained. 53. Latin term for negligence
a. Temperate culpa
b. Liquidated
c. Moral 54. Latin term for negligence arising from breach of contract
d. Exemplary culpa contractual

42. Damages determined beforehand. 55. Latin term for delay on the part of both debtor and creditor at the same time
a. Actual compensatio morae
b. Moral
c. Liquidated 56. Latin term for the right to impugn contracts entered to defraud creditors
d. Temperate accion pauliana

43. With regard to the right as to the fruits of the thing, which is not correct? 57. Latin term for the right to be subrogated on all the rights of the debtor.
a. If there is no condition or term for its fulfillment, the obligation to deliver accion subrogatoria
arises from the perfection of the contract or creation of the obligation.
b. If the obligation arises from the contract of sale, the vendor has a right to 58. Latin term for the wrong that causes injury to another
the fruits of the thing from the time the obligation to deliver arises. tort
c. If the obligation is subject to a suspensive condition, the obligation to deliver
arises upon the expiration of the term or period. 59. Latin term for voluntary administration of a property without the owner’s
d. If the obligation is subject to a suspensive condition, the obligation to deliver consent
arises from the moment the condition happens. negotiorum gestio

44. A is obliged to deliver his only car to B on November 20, 2009. If A does 60. Latin term for "whenever you received something you are not entitled to
not deliver, and on November 22, 2009, a typhoon destroys the car. demand, you have the obligation to return it'
a. A is not liable because the obligation is extinguished. solutio indebiti
b. A and B will divide the loss equally.
c. A’s obligation is converted into a monetary obligation. 61. Every person obliged to give something is also obliged to take care of it with
d. A is liable because he is in delay. the proper extraordinary diligence, unless the law or the stipulation of the
parties requires another standard of care.
45. What is the basis of the liability of a school when a student is stabbed inside True
the campus by a stranger in the school? False
a. Quasi-delicts
b. Contracts 62. The creditor has a right to the fruits of the thing from the time the obligation
c. Delicts to deliver it arises
d. Quasi-contracts True
False
46. The following except one, are included in civil liability. The exception is
a. Restitution 63. A person shall acquire no right over a fruit of a thing until the same has been
b. Reparation delivered to him.
c. Starvation True
d. Indemnification False

47. The thing itself shall be restored, as a rule 64. The very difference between fraud and negligence is intent.
a. Reparation True
b. Starvation False
c. Restitution
d. Indemnification 65. When what is to be delivered is a determinate thing, the creditor may compel
the debtor to make the delivery.
48. The court determines the amount of damage taking into consideration the True
price of the thing and its sentimental value to the injured person False
a. Starvation
b. Restitution 66. If the thing is indeterminate or generic, he may ask that the obligation be
c. Indemnification complied with at the expense of a third person.
d. Reparation True
False
49. The consequential damages suffered by the injured person and those
suffered by his family or third person by reason of the act. 67. Fortuitous events include only acts of god but not acts of men.
a. Restitution True
b. Starvation False
c. Reparation
d. Indemnification 68. If the obligor delays, or has promised to deliver the same thing to two or
more persons who do not have the same interest, he shall be responsible for
50. Action to impugn or rescind acts or contracts done by the debtor to defraud any fortuitous event until he has effected the delivery.
the creditors. True
a. Accion quanti-minoris False
b. Accion reinvindicatoria
c. Accion pauliana

0 0
69. The obligation to give a determinate thing includes that of delivering all its False
accessions and accessories if mentioned.
True 83. Usurious transactions shall be governed by the civil code.
False True
False
70. If a person obliged to do something fails to do it, there is no other remedy
except payment of damages. 84. There is no delay in obligations not to do.
True True
False False

71. In culpa contractual, the basis of liability is the negligence itself.


True
False
85. The receipt of the principal by the creditor without reservation with respect
72. When the obligation consists in not doing, and the obligor does what has to the interest, shall give rise to a conclusive presumption that said interest
been forbidden him, it shall be undone at his expense. However, this rule has been paid.
may not apply in obligations to do. True
True False
False
86. The receipt of a later installment of a debt without reservation as to prior
73. Those obliged to deliver or to do something incur in delay from the time the installments, shall likewise raise a rebuttable presumption that such
obligee judicially or extrajudicially demands from them the fulfillment of installments have been paid.
their obligation. True
True False
False
87. The creditors, after having pursued the property in possession of the debtor
74. Demand is not necessary for delay to exist when the obligation or the law to satisfy their claims, may exercise all the rights and bring all the actions
expressly so declare. of the latter for the same purpose, save those which are inherent in his
True person; they may also impugn the acts which the debtor may have done to
False defraud them.
True
75. In reciprocal obligations, neither party incurs in delay if the other does not False
comply or is not ready to comply in a proper manner with what is incumbent
upon him. From the moment one of the parties fulfills his obligation, he 88. Subject to the laws, all rights acquired in virtue of an obligation are
must make a demand for delay by the other party to begin. transmissible, if there has been no stipulation to the contrary.
True True
False False

76. Whenever delay is present for both creditor and debtor, there will be as if 89. In culpa aquiliana, there must be a pre-existing contractual relation between
no delay. the parties.
True True
False False

77. Those who in the performance of their obligations are guilty of fraud, 90. A waiver for an action of future negligence is void.
negligence, or delay, and those who in any manner contravene the tenor True
thereof, are liable for damages. False
True
False

78. Responsibility arising from fraud is demandable in all obligations. Any


waiver of an action for fraud is void.
True
False

79. Responsibility arising from negligence in the performance of every kind of


obligation is also demandable, but such liability may be regulated by the
courts, according to the circumstances. Also, any waiver of an action for
negligence is valid.
True
False

80. Generally, there is no delay when there is no demand.


True
False

81. Diligence of a father of a good family is also known as ordinary diligence


True
False

82. Except in cases expressly specified by the law, or when the obligation
consists in giving an indeterminate thing, or when it is otherwise declared
by stipulation, or when the nature of the obligation requires the assumption
of risk, no person shall be responsible for those events which could not be
foreseen, or which, though foreseen, were inevitable.
True

0 0
1. Every obligation whose performance does not depend upon a condition or a
period is demandable at once.
True 11. Kivin obtained a loan from Kingfisher bank. The loan was embodied in a
False promissory note. As security, Kivin executed a chattel mortgage on his
standing crops. Said crops were however subsequently destroyed by
2. When the debtor binds himself to pay when his means permit him to do so, typhoon “Grace”. Kingfisher bank demanded payment from Kivin although
the obligation shall be deemed to be with a potestative suspensive condition the maturity date of the loan is not yet due. Is Kingfisher bank demand
and shall be void. correct?
True
False First Answer - Yes, because of the loss of security which resulted to Kivin
losing his right to make use of the period.
3. If the obligation is divisible, that part thereof which is affected by the Second Answer – No, because the loss of the security is through a fortuitous
impossible or unlawful condition shall be void. event.
True
False a. False; False
b. True; True
4. Every obligation which contains a resolutory condition or a period is c. False; True
demandable at once. d. True; False
True
False 12. Noreen agreed, that in case of non-payment of his debt, to render services
as a servant. Which of the following is not correct?
5. The condition shall be deemed fulfilled when the obligor voluntarily a. If the services will be “for free”, the stipulation is void for being contrary to
prevents its fulfillment. law and morals.
True b. If the services will not be gratuitous, specific performance of the service will
False be the proper remedy in case of non-compliance.
c. Should there be a valid stipulation as regards the rendition of services, an
6. Obligations for whose fulfillment a day certain has been fixed, shall be action for damages should be brought in case of non-compliance.
demandable only when that day comes. d. If the services will be rendered in satisfaction of the debt, the stipulation is
True valid.
False
13. The following is considered fraud or fraudulent.
7. Gerome bound himself to deliver a specific horse to Sidfrey. Which of the a. Misrepresentation made not in bad faith.
following is true? b. The usual exaggeration in trade, when the other party had the opportunity to
a. The obligation is deemed to be potestative on the part of the debtor. know the facts.
b. The obligation is demandable at once. c. Failure to disclose facts when there is duty to reveal them.
c. The period shall be fixed by the court. d. “Caveat Emptor” or let the buyer beware.
d. None of the choices
14. The obligation is demandable on the date of the obligation and shall
8. Jervin bound himself to support Genela in her college expenses until he continue to be in force until the arrival of the day certain
graduates. This obligation best illustrates a. Indefinite period
a. Resolutory condition b. Legal period
b. None of the choices c. Resolutory period
c. Resolutory period d. Suspensive period
d. Suspensive condition
e. Suspensive period 15. When the debtor binds himself to pay when his means permit him to do so,
the obligation is
9. Avedon bounds himself to buy Kristy a mobile phone if he decides to go the a. Conditional
mall tomorrow. The status of this obligation is b. Simple
a. Void, since it illustrates a potestative resolutory condition c. With a period
b. Valid, since it illustrates a potestative resolutory period d. Pure
c. Void, since it illustratesa potestative resolutory period
d. Valid, since it illustrates a potestative resolutory condition 16. Facultative as distinguished from alternative obligation
e. None of the choices a. The right of choice is given only to the debtor.
b. If one of the prestations is illegal, the others may be valid and the obligation
10. Leo bound himself to pay Jervin P50,000 if he could win against Avedon in remains.
a basketball match. In addition, Leo further bounds himself to pay P10,000 c. Various things are due, but the giving of one is sufficient.
if Jervin could seriously injure Avedon in the match. Which of the following d. It is impossible to give all except one, that last one must still be given.
is incorrect?
a. If Jervin wins the match, he is entitled for the P50,000 17. On June 24, 2019 Kathleen is obliged to give Heidi her specific car. There
b. Jervin cannot claim the P10,000 even if the corresponding condition was was no delivery until June 30 when the garage of the car collapsed due to
met. heavy rain and strong winds of Typhoon Janny, and the car was totally
c. The whole obligation is void since one of the condition is contrary to law. destroyed. Is Kathleen still liable?
d. All of the choices are correct statements. a. Yes, because the contract is perfected.
b. No, because there was no demand by Heidi to deliver the car.
c. Yes, the obligation to deliver the car is converted into payment of equivalent
monetary value because Kathleen is in legal delay.
d. No, even if Kathleen was in default, he could plead impossibility of
performance.

18. This obligation is demandable at once when it


a. Has a period.
b. has a suspensive condition.

0 0
c. Is with a term ex-die. a. 6
d. has a resolutory condition. b. 5
c. 1
d. 3

19. A period with a suspensive effect. 28. Grace, Kristy, and Genela are solidary debtors of Janny. The total amount
a. I will support you until January 1 of next year. due is P100,000 to be paid in the following terms.
b. I will support you beginning January 1 of next year.
c. I will support you if A dies of cancer. Share of Grace – monthly payment of P10,000 each from January to May.
d. I will support you if A marries B. Share of Kristy – monthly payment of P5,000 each from February to May.
Share of Genela – the balance to be paid by Genela if Janny passess the
20. A period with a resolutory effect. Board Exam on May.
a. I will support you until A dies.
b. I will support you beginning January 1 of next year. How much can Janny demand Kristy on January?
c. I will support you if A dies of cancer.
d. I will support you if A dies. a. P5,000
b. P10,000
21. Elaisa bound herself to give David a specific laptop, a specific phone, or a c. P0
specific tablet. If the tablet is lost through Elaisa’s negligence, which of the d. P15,000
following statements is true? e. None of the choices
a. David may still choose the tablet but may only receive the value of the thing
with indemnity to damages. 29. Grace, Kristy, and Genela are solidary debtors of Janny. The total amount
b. None of the statements is true due is P100,000 to be paid in the following terms.
c. Elaisa is bound to pay damages
d. The obligation remains to be an alternative obligation Share of Grace – monthly payment of P10,000 each from January to May.
Share of Kristy – monthly payment of P5,000 each from February to May.
22. Elaisa bound herself to give David a specific laptop, a specific phone, or a Share of Genela – the balance to be paid by Genela if Janny passess the Board
specific tablet. Assuming all objects are lost through Elaisa’s fault. Which Exam on May.
of the following is true?
a. None of the statements is true How much can Janny demand from Grace on February?
b. Elaisa is bound to pay damages
c. David may still choose the tablet but may only receive the value of the thing a. None of the choices
with indemnity to damages. b. P10,000
d. The obligation remains to be an alternative obligation c. P0
d. P15,000
23. Elaisa bound herself to give David a specific laptop, a specific phone, or a e. P5,000
specific tablet. If the tablet is lost through Elaisa’s negligence and she
waived her right of choice, which of the following is true? 30. Grace, Kristy, and Genela are solidary debtors of Janny. The total amount
a. The obligation remains to be an alternative obligation due is P100,000 to be paid in the following terms.
b. All of the statements are true
c. David may still choose the tablet but may only receive the value of the thing Share of Grace – monthly payment of P10,000 each from January to May.
with indemnity to damages. Share of Kristy – monthly payment of P5,000 each from February to May.
d. Elaisa is bound to pay damages Share of Genela – the balance to be paid by Genela if Janny passess the
Board Exam on May.
24. Leo bound himself to deliver a laptop or as a substitute a television to Heidi.
Assuming the laptop is lost through a fortuitous event, which of the How much can Janny demand from Genela in May assuming Janny passed
following is correct? the board exam and previous installments were not paid?
a. Leo’s obligation is extinguished
b. Leo will be bound to deliver the television a. None of the choices
c. Leo must still deliver a laptop b. P15,000
d. None of the choices c. P5,000
d. P10,000
25. Leo bound himself to deliver a laptop or as a substitute a television to Heidi. e. P30,000
Assume it was the television that got lost through a fortuitous event. Which
of the following is correct? 31. Grace, Kristy, and Genela are solidary debtors of Janny. The total amount
a. If Heidi chose the delivery of the television, but since it was through a due is P100,000 to be paid in the following terms.
fortuitous event, Leo’s obligation is extinguished.
b. None of the choices Share of Grace – monthly payment of P10,000 each from January to May.
c. The loss of the television is irrelevant to the Leo’s liability Share of Kristy – monthly payment of P5,000 each from February to May.
d. Leo’s obligation still subsist since the laptop is not lost Share of Genela – the balance to be paid by Genela if Janny passess the
Board Exam on May.
26. Avedon, Jervin, and Sidfrey are debtors of solidary creditors Gerome and
Genela. The debt amounts to P18,000. Which of the following is incorrect? Assuming Janny remitted Kristy’s share in the obligation, how much can
a. If Sidfrey is insolvent, Gerome can collect up P12,000 only Janny demand from Kristy on March?
b. All of the choices are correct statements
c. Gerome may collect the whole of P18,000 from any debtor a. P5,000
d. If Genela collected P18,000, she is obliged to pay Gerome his share
b. P10,000
c. P0
27. Avedon, Jervin, and Sidfrey are debtors of solidary creditors Gerome a nd
d. None of the choices
Genela. The debt amounts to P18,000. How many distinct debts are there?

0 0
38. In obligations with a penal clause, the creditor as a rule may recover from
the debtor in case of non-compliance the following:
a. The principal, the penalty as agreed upon, plus damages and interest.
b. The principal and the penalty.
c. The principal, the penalty and damages.
d. The principal, the penalty, plus interest.

32. Grace, Kristy, and Genela are solidary debtors of Janny. The total amount
due is P100,000 to be paid in the following terms.
39. Jervin obligated himself to pay Sidfrey the amount of P30,000, 30 days after
Share of Grace – monthly payment of P10,000 each from January to May. May 1, 2019 plus a penalty of P3,000 if he fails to pay the obligation on due
Share of Kristy – monthly payment of P5,000 each from February to May. date. After demand for payment by Sidfrey, Jervin offered to pay on July
Share of Genela – the balance to be paid by Genela if Janny passess the 30, 2019. Sidfrey can demand from Jervin:
Board Exam on May. a. P30,000 plus P3,000 plus legal interest.
b. P30,000 plus P3,000.
Assuming Grace becomes insolvent on April, how much will Kristy pay (net c. P30,000 plus legal interest.
of reimbursement) on the said month? d. P30,000 plus P3,000 plus legal interest plus damages.

a. P5,000 40. In Facultative obligations, if substitution has been made, which of the
b. P15,000 following is false?
c. P10,000 a. The loss of the original prestation is immaterial.
d. None of the choices b. The obligation is converted into a simple obligation.
c. The obligation is extinguished.
33. Grace, Kristy, and Genela are solidary debtors of Janny. The total amount d. The obligation ceases to be facultative.
due is P100,000 to be paid in the following terms.
41. Ariane, Yvone, and Tintin are solidary debtors of Jerusel for P3,000. Jerusel
Share of Grace – monthly payment of P10,000 each from January to May. remitted Tintin’s share. Ariane therefore paid later only P2,000. Ariane can
Share of Kristy – monthly payment of P5,000 each from February to May. recover reimbursement from Yvone in the amount of
Share of Genela – the balance to be paid by Genela if Janny passess the Board a. P500
Exam on May. b. P1,000
c. P1,500
Assuming the debtors are joint. On March, how much can Janny collect from d. P0
Grace if Kristy becomes insolvent?
42. The following are the instances when in addition to the penalty, the creditor
a. P15,000 can also claim damages and interest. Choose the incorrect one.
b. P10,000 a. The debtor is in delay
c. None of the choices b. Obligor refuses to pay the penalty
d. P12,500 c. All of the choices are instances when the creditor can claim damages and
interest in addition to penalty
34. Which of the following is not a characteristic of a joint indivisible d. Stipulated under law or contract
obligation? e. Debtor is guilty of fraud in the fulfillment of the obligation
a. The creditor must proceed against all the debtors.
b. If any of the debtor be insolvent, the others shall not be liable for his share. 43. The debtor loses the benefit of the period, and his obligation becomes
c. Demand must be made to all the debtors. demandable when
d. Each of the debtor can be held liable to pay for the entire obligation. a. The debtor attempts to abscond.
b. Demand by the creditor would be useless.
35. Debtors Gerome, Kivin, and David are bound to deliver a specific cow to c. After contracting the obligation, the creditor suspects that the debtor is
Noreen. Upon valid demand by Noreen, Kivin objected to the delivery of becoming insolvent.
the cow. Choose the incorrect statement. d. The guarantees as promised and delivered by the debtor are not acceptable
a. Gerome can validly delivery the cow to Noreen to the creditor.
b. All the choices are correct statements
c. If the cow is not delivered it will be converted to a monetary one. 44. An obligation wherein various things are due but the complete performance
d. Kivin alone shall pay damages of all of them is necessary to extinguish the obligation
a. Facultative obligation
36. Debtors Gerome, Kivin, and David are bound to deliver a specific cow to b. Simple obligation
Noreen. Upon valid demand by Noreen, Kivin objected to the delivery of c. Alternative obligation
the cow. d. Conjoint obligation

Assume that the debtors are solidary. Which of the following is an incorrect 45. Daryl and Warren are liable in solidum to Derrek in the amount of
statement? P1,000.00. Derrek remitted Daryl’s share. Subsequently, Warren paid
a. Kivin alone shall pay damages Derrek P1,000. Which of the following is correct?
b. All the choices are correct statements a. Warren is not entitled to claim reimbursement from Daryl
c. Gerome can validly delivery the cow to Noreen b. None of the choices is correct
d. If the cow is not delivered it will be converted to a monetary one. c. The obligation is not yet extinguished
d. The remission of Daryl’s share extends to the benefit of Warren so Warren
37. Which of the following statement is false? should not have paid.
a. Accomplishment of work by metrical units are divisible.
b. Execution of a certain number of days of work shall be divisible. 46. A, B and C secured a loan from D. the promissory note which evidence the
c. An obligation to pay a certain amount in ten annual installments is divisible. obligation states: “I promise to pay or order P10,000 payable on demand”.
d. Obligations to give definite things and those that are not susceptible of (Signed) A B C. the obligation is:
partial performance shall be deemed to be divisible. a. Joint
b. Divisible

0 0
c. Solidary
d. Indivisible

47. Every time a period is established in an obligation, it shall be considered for


the benefit of the creditor.
True
False

48. The creditor may receive part of one and part of the other undertaking in
alternative obligations.
True
False

49. The indivisibility of an obligation does not necessarily give rise to solidarity.
Nor does solidarity of itself imply indivisibility.
True
False

50. Solidarity may exist although the creditors and the debtors may not be bound
in the same manner and by the same periods and conditions.
True
False

0 0
1. In three of the following cases, compensation cannot be claimed by the
debtor except:
a. By the bailee in commodatum
b. By the person obliged to give support due by gratuitous title
c. By the debtor in an obligation arising from a penal offense
d. By the depositor in a contract of deposit 9. A owes B a sum of money and secured by a pledge on his car. Later, the car
is found in the possession of A.
2. Which of the following is not a requisite for a legal compensation? a. It is presumed that the debt is extinguished
a. Both are liquidated and demandable b. It is presumed the debt of A has been condoned
b. By the person obliged to give support due by gratuitous title c. It is presumed that both the debt and pledge has been remitted
c. Both debts are due d. It is presumed that the pledge is condoned.
d. By the debtor in an obligation arising from a penal offense
10. In which of the following is consignation alone without prior tender of
3. A owes B P1M guaranteed by X. In turn, B owes A P 0.5M. A failed to pay payment not valid to extinguish the obligation?
B and the latter is now collecting from X. a. When he is incapacitated to receive the payment at the time it is due.
a. X is liable to B for 1M because as guarantor he is liable for the debt of the b. When with just cause, the creditor refuses to issue receipt
debtor in case of non-payment thereof c. When two or more persons claim the same right to collect
b. X cannot claim compensation against B because the latter owes him nothing d. When the title of the obligation has been lost
c. X can set up compensation as regards what B owes A
d. X shall be liable to B for 0.5M only if the latter agrees to the compensation 11. Which of the following is not a requisite of payment by cession?
a. One debtor and two or more debtors
4. A owes B P1M guaranteed by C. B assigns his credit to X. X assigns his b. The creditors become owners of the properties assigned to them
credit to Y. Y assigns his credit to C, the guarantor. Does A have to pay C? c. The creditors are authorized to sell the properties of the debtor
a. No, because it is the obligation of the guarantor to pay the creditor for the d. The debtor is insolvent
benefit of the debtor
b. Yes, because the guarantee has been extinguished by confusion or merger 12. Which of the following is not a requisite of dation en payment?
of rights a. Acceptance of the creditor is not necessary
c. Yes, because the guarantor has now become the creditor b. Debtor is not necessary insolvent
d. No, because the guarantor is liable to the creditor B and not to debtor A c. The creditor becomes owner of the property used as payment
d. One debtor and one creditor
5. A owes B P1M. B owes A P0.5M. B assigned credit to C with the consent
of A who reserved his right to compensation. 13. D obtained a loan from C payable on or before December 31, 2002. Which
a. A cannot claim compensation against C without his consent of the following statements is correct?
b. A can claim compensation against C even without the reservation a. C can demand payment from D even before December 31, 2002
c. A can claim compensation against C as to what B owes him b. D can pay before December 31, 2002.
d. A cannot claim compensation against C because he consented to the c. C can demand payment from D on or before December 31, 2002
assignment d. None of the above
e. D can only pay the obligation on December 31, 2002.
6. A owes B P1M due on January 1, 2001. B owes a P0.2M due on January 20,
2001 and P0.3M due on March 2, 2001. B assigned the credit to C on 14. The obligation of Pedro in favor of Juan amounts to P30,000 payable on
February 2, 2001 without the k nowledge of A who obtained knowledge of May 5, 2002. On April 15, 2002 Ciriaco and Pedro agreed that Ciriaco will
the assignment of credit of February 4, 2001. pay the obligation of Pedro. Juan gave his consent to the said agreement.
a. A can claim compensation of all the debts of B to him The change of the debtor in this case is called:
b. A cannot claim compensation against C because of his knowledge of the a. Substitution
assignment b. Expromission
c. A can claim compensation up to P0.2M only c. Delegacio
d. Legal compensation cannot take place d. Subrogation

7. A owes B P1M due on February 3, 2001. B owes A a lancer GSR worth 15. X issued a promissory note to A amounting to P10,000. A purchased a
P1M due on February 4, 2001. computer from B and he indorsed the promissory note to B. B indorsed the
a. On February 4, 2001, A can claim compensation because both debts are now promissory note to C in payment of his loan. C indorsed the said promissory
due note to X as payment for the repair of his car. The obligation of X in this
b. Legal compensation cannot take place case is extinguished by:
c. The parties can agree on compensation only on February 4, 2001 when both a. Confusion or merger of rights
debts are due b. Condonation or remission of debts
d. On February 3, 2001, A cannot claim compensation because one of the debts c. Novation
is not yet due d. Compensation or merger of rights

8. A owes B P1M with 12% interest. As agreed upon among the parties, the 16. C bought the only car of D on February 15, 2002 and D agreed to deliver it
principal amount shall be paid to B but the interest shall be given to C who to C on April 30, 2002. They did not agree on the place of delivery. Where
accepted the stipulation in his favor. Later, A and B agreed that instead of is the place of delivery?
cash it shall be parcel of land worth P1M. a. Domicile of the debtor
a. In the absence of the consent of C to the novation, he is still entitled to the b. Where the car is on April 30, 2002
interest c. Where the car is on February 15, 2002
b. The novation of the obligation shall also extinguish the accessory obligation d. Domicile of the creditor
to pay the interest to C.
c. Since the parcel of land could not possibly earn interest, the obligation to 17. D has an obligation to deliver his only dog to C on October 16, 2001. The
pay interest is extinguished said dog gave birth to 7 puppies. On October 16, 2001, D failed to deliver
d. With or without the consent of C, the obligation to give the interest to him the said dog. Who shall be the owner of the dog on October 16, 2001?
is also extinguished because of novation of the principal obligation a. C because he has the right to demand its delivery
b. C because that is the agreed date of delivery
c. D because the delivery of the dog is dependent upon his will

0 0
d. D because the dog is still in his possession

18. When the obligor binds himself to pay when his means permit him to do so, 27. Dela Cruz obtained himself to deliver a determinate horse to Mary Jane on
the obligation is: November 5. When the date of delivery of the horse arrived, the horse has
a. Pure already an offspring. Dela Cruz is obliged to deliver the
b. With a period a. Dela Cruz can ask Mary Jane to pay the offspring
c. Conditional b. Horse only
d. Facultative c. Horse as well as the offspring
d. Dela Cruz can refuse to deliver both the horse and the offspring
19. A obtained a loan from B in the amount of P 10,000 payable on December
31, 2002 plus 10% interest. A won in the lotto and on March 30, 2002 he 28. When the obligor voluntarily prevents the happening of the condition, it is
offered to pay his loan to B plus P1,000 as interest but B refused to accept deemed that the condition is
his payment. Which of the following statements is correct? a. Constructively extinguished
a. B can be compelled to accept the payment if the interest will be increased b. Constructively demandable
b. A cannot compel B to accept his payment because the loan is not yet due. c. Constructively fulfilled
c. B must accept the payment because it is complete d. Constructively nullified
d. A can make consignation in court because B refused to accept his payment
without justifiable cause 29. A bound himself to deliver a determinate horse to B on January 14, 2002.
On January 16, 2002, the horse was struck and killed by lightning.
20. D has the obligation to give a two-year old black male dog to C on April 30, a. B may claim damages from A
2002. On April 2, 2002, all of the dogs of D were hit by a lightning and they b. A is liable for the loss of the horse because he was in delay
all died. Therefore; c. A’s obligation is extinguished
a. D must still give a two-year old black male dog to C d. A has the obligation to replace the horse
b. The obligation of D to C is extinguished
c. C can demand for the dog plus damages 30. Payment by cession as distinguished from dation in payment
d. C can only demand for damages from D a. The debtor is not necessarily in a state of financial difficulty
b. What is delivered by the debtor is merely a thing to be considered as the
21. In alternative obligation, the right of choice belongs to: equivalent of the performance of the obligation
a. Both the creditor and the debtor c. The property is alienated by the debtor to the creditor in satisfaction of a
b. The debtor debt in money
c. The creditor d. The effect is to release the debtor for the net proceeds of the things cede or
d. Third person assigned

22. In facultative obligation the right of choice belongs to: 31. A owes B P20,000 which became due and payable last June 23, 2001. On
a. Both the creditor and the debtor that date A offered B P10,000, the only money he then had but B refused to
b. Third person accept the payment. A therefore met C, B’s 23 year-old son, to whom he
c. The creditor gave the P10,000 with the request that he turn the money to B. The money
d. None of the above was stolen while in C’s possession. How much may B still recover form A?
e. The debtor a. P0
b. P15,000
23. The sources of liability for damages are the following except c. P 20,000
a. Solutio indebiti d. P10,000
b. Negligence
c. Delay 32. If a third person pays an obligation, what are the rights which are available
d. Fraud to him if he pays the obligation with the knowledge and consent of the
debtor?
24. When the fulfillment of the condition depends upon the sole will of the
debtor, the conditional obligation shall be: First Answer – He can recover from the debtor the entire amount which he
a. Unenforceable has paid
b. Void Second Answer – He is subrogated to all of the rights of the creditor
c. Voidable
d. Valid a. Only the second statement is correct
b. Only the first statement is correct
25. Z obtained a loan from X on the amount of P10,000. Z promised to pay the c. Both are correct
said loan as soon as possible. Two years had lapsed but the loan is still d. Both are wrong
unpaid. The remedy available to X is:
a. To ask the court to fix the period 33. A, B and C executed a promissory note binding themselves to pay P9,000
b. To file an action against Z for collection of debt to X, Y and Z. The note is now due and demandable. Can the creditors
c. To attach the property of Z proceed against A alone for the payment of the entire debt?
d. To demand payment for damages a. Yes, since the promissory note is silent with respect to the rights of the
creditors, the obligation is presumed to be solidary
26. A, B, and C are solidary creditors of D in the amount of P15,000. C b. Yes, either X, Y or Z can collect P9,000 from A
demanded payment from D. After the demand for payment by C, D in order c. No, each creditors can collect only P3,000 from A
to extinguish the entire obligation may pay to: d. No, each creditor can only collect P1,000 from A
a. C only
b. B only 34. No person shall be held responsible by reason of fortuitous event except:
c. A only a. When the obligation is purely personal in nature
d. To either A or B b. When demand would be useless

10

0 0
c. When the nature of the obligation requires the assumption of risk 43. Mel sold to Jay her car and promised to deliver the car on January 30, 2004.
d. When the time is of the essence of the contract On January 15, 2004, Mel sold the same car to Patrick and Patrick
immediately possessed the car. As of February 5, 2004, Jay has not received
the car from Mel.
a. The sale is rescissible because of damage caused to Jay
b. Jay can cancel the contract of sale between Mel and Patrick because the
contract of sale between him and Mel was perfected first
c. Mel cannot be considered in delay because there was no demand yet from
Jay
35. Indivisibility as distinguished from solidarity d. Mel is already in delay even if there was no demand from Jay and she shall
a. Refers to the legal tie or vinculum be liable for damages
b. Plurality of subject is invisible
c. Refers to the prestation which constitutes the object of the obligation
d. When the obligation is converted into one of the indemnity for damages 44. S sold his car with plate no. XYZ 123 to B and B paid P150,000 to S. They
because of breach, the character of the obligation remains agreed that the said car will be delivered by S to B on December 14, 2002.
Which of the following statements is true based on the facts given?
36. A source of obligation not arising from law a. S will be in delay if he will not be able to deliver the car on maturity date
a. Culpa aquiliana b. B may demand for delivery of the car on December 13, 2002
b. Contract c. S may deliver the car to B on December 12, 2002 and compel B to accept it
c. Negotiorum gestio d. B is considered the owner of the car on December 14, 2002
d. Solutio indebiti
45. Which of the following statements is true and correct?
37. I. The loss or deterioration of the thing intended as a substitute through the a. Unless otherwise agreed upon by the parties, the sale of the mortgaged
negligence of the obligor does not render him liable. property extinguishes in full the mortgage constituted thereon
II. A person alternatively bound by different prestations shall completely b. Where an obligation is secured by a pledge or mortgage and it is not paid
perform one of them when due, the pledgee or mortgagee may appropriate the thing given by the
a. False, true way of pledge and mortgage
b. True, false c. Pledge and mortgage are accessory contracts because they cannot exist by
c. False, false themselves
d. True, true d. In both pledge and mortgage, the creditor is entitled to deficiency judgment

38. There being no express stipulation and if the undertaking is to deliver a 46. A, B and C borrowed P3M from D, E and F evidenced by a promissory note
determinate thing the payment shall be made at worded as follows to wit: “I promise to pay D, E and F P3M (Sgd.) A, B
a. At the domicile of the debtor and C” How much can D collect from A?
b. At the domicile of creditor a. P3M
c. Whatever the thing might be at the moment b. P1M
d. Whenever the thing might be thing might be at the moment the obligation is c. P1.5M
to be fulfilled. d. P0.5M

39. Demand is not intended to put the debtor in default, except 47. A, B and C bound themselves to deliver to X a specific car worth P3M. Due
a. When the time is of the essence to the fault of A, the car was lost. In this case:
b. When the parties so stipulate a. X can claim only from A the whole amount of damages other than the value
c. When a quasi-contract exists of the car
d. When the demand would be useless b. Since it is solidary liabilities for damages, X can claim the same from any
of the three
40. Culpa Aquiliana is distinguished from culpa contractual c. Only A is liable for damages although B and C are liable for their respective
a. The source of liability is the defendants negligent act or omission itself shares in the obligation
b. Proof of the contract and of its breach is sufficient prima facie to warrant d. X can claim damages from any one of the three for his proportionate part of
recovery liability because the obligation is indivisible
c. The negligence of the defendant is merely an incident in the performance of
the obligation 48. L, M and N are solidary debtors of Y for the amount of P300,000 payable
d. Proof of due diligence in the selection and supervision of employees is not as follows:
as a defense
L – P50,000 payable when he passed the CPA board exam
41. This is the kind of diligence that the obligor must observe in the performance M - P100,000 payable on February 14, 2004
of his obligation to give something N – P150,000 payable upon completion of his construction project
a. Diligence of a father of a good family If L will pass the CPA Board Exam, what shall be the right of Y?
b. Extraordinary diligence of a good father of a family
c. Diligence of a good father of a family a. Demand P300,000 from L only
d. Diligence of a good father of a family even if the law or stipulation requires b. Demand P50,000 from L, M and N
another standard of care c. Demand P300,000 from L, M and N
d. Demand P 50,000 from L only
42. D owes C P1M. X without the knowledge or against the will of D paid C
P2M. Can X get reimbursement from D? 49. A obliged himself to deliver specific cans of powdered milk of B from
a. No reimbursement because the payment was not proper being without the Pangasinan to Manila. While his truck was traveling on the North
knowledge or against the will of D. Expressway, it was hi-jacked by a band of robbers who also took the cans
b. P2M by way of reimbursement from D to prevent unjust enrichment on the of milk belonging to E. Is A liable for the loss of the goods?
part of D at the expense of X a. No, because they were generic and they and they cannot be lost
c. P1M plus interest from the time of payment until reimbursement b. Yes, because there was no stipulation exempting him from loss in case of
d. P1M only for that is the extent of benefit of D fortuitous event
c. No, because the loss was due to a fortuitous event

11

0 0
d. Yes, because he was in possession of the same at the time of the loss and
therefore presumed at fault

50. A brought her diamond ring to a jewelry shop for cleaning and the latter
undertook to return the ring by February 1, 2000. When the said date arrived,
the jewelry shop informed A that the job was not yet finished and they asked
A to return 5 days after. On February 6, 2000, A went to the shop to claim
the ring but she was not informed that the same was stolen by a thief the
night before. Decide.
a. The jewelry shop is liable if it was so stipulated between the parties
b. The jewelry shop is not liable because there was no delay on its part
c. The jewelry shop is liable for the loss despite the force majeure because of
delay
d. The jewelry shop is not liable for the loss because it took place due to force
majeure

12

0 0
1. Statement 1: An obligation where the debtor binds himself to pay when his
means permit him to do so is a conditional obligation.
Statement 2: Solidarity may exist although the creditors and debtors may
not be bound in the same manner and by the same periods and conditions
a. Both statements are false 10. Sidfrey has been missing for some time leaving no one to manage his
b. Both statements are true properties. Avedon and Jervin jointly took charge of the management
c. Statement 1 is false, statement 2 is true thereof. However, due to the fault of Avedon, the properties of Sidfrey were
d. Statement 1 is true, statement 2 is false damaged. The liability therefore to Sidfrey for damages shall be:
a. they are not liable since Sidfrey is at fault for having abandoned his
2. In quasi-delict, the liability of a person who is held liable for the fault or properties
negligence of another person who is under his or responsibility Shall cease b. Both Avedon and Jervin jointly liable
if c. Both Avedon and Jervin solidarity liable
a. He was not present at the time of the act, hence it was not possible to him to d. Only Avedon being liable
prevent the damage
b. He observed ordinary diligence to prevent the damage 11. "Either of us promises to pay X or order Pl,000. (Signed) A and B" The
c. He observed the diligence of a father of a good family liability of the makers is
d. There was contributory negligence on the part of the person injured a. joint
b. solidary
3. Not a ground for damages c. secondary
a. Culpa d. none, until the makers are furnished a notice of dishonor
b. Dolo causante
c. Bad faith 12. Using the preceding number, if Ronald is insolvent, Liera can recover from
d. Mora Sandra the amount of:
e. Next Liera, Ronald and Sandra are solidary debtors of Raymond for P3,000.
Raymond remitted Sandra's share. Liera therefore paid later only P2,000.
4. Kingfisher School Review Center (KSRC) stated in the leaflets it distributed Liera can recover reimbursement from Ronald in the amount of
last January that any reviewee who places first in the licensure examinations a. P0
for engineers this year will receive a cash prize, of Pl,000,000 b. P1,500
a. The obligation of KSRC is a pure obligation c. P500
b. The obligation of KSRQ is subject to a resolutory condition d. P1,000
c. The obligation of KSRC is subject to a suspensive condition
d. The obligation of KSRGJs an obligation with a suspensive period 13. A leased his house to B with the agreement that the telephone bills shall be
shouldered by B. Six months later, B left the premises leaving behind unpaid
5. Joshua, Eunice, and Patrick solidarity owe Jewel and Rhobie P30,000. Jewel telephone bills. A refuse to pay the unpaid bills on the ground that B had
remitted the entire obligation in favor of Joshua without the consent of already substituted him as customer of the telephone company. Decide.
Rhobie. The effect is a. A is liable for the unpaid bills of B because the contract was. between A and
a. The obligation is not extinguished until Joshua collects from Eunice and the telephone company and A cannot be substituted by B without the
Patrick consent of the company.
b. Joshua can recover from Eunice and Patrick their respective share of the b. A is correct to assert no liability to the telephone company due to novation
debt of the contract by way of substitution of debtor.
c. Joshua cannot recover from Eunice and Patrick because remission in her c. A is not liable because the telephone company failed to exercise due
favor extends to the benefit of Eunice and Patrick diligence in collecting from B his unpaid bills
d. The obligation is not yet extinguished until Rhobie is paid by Jewel since d. A is liable to the telephone company subsidiary, only after it fails to collect
Rhobie did not consent to the remittance from B

6. The person who is bound to the fulfillment of an obligation: 14. The right of the creditor to exercise all the rights of his debtor to satisfy his
a. Guarantor claim, except rights which are inherent and personal on the part of the
b. Passive subject debtor.
c. Active subject a. Accion pauliana
d. Consignation b. Accion publiciana
c. Accion quanti-minoris
7. Leo owes Jervin P20,000 which became due on December 20, 2011. On that d. Accion subrogatoria
date, Leo offered P10,000, the only money he had then, but Jervin refused
to accept the payment. Thereafter, Leo met Romelita, Jervin's wife, to whom 15. I. As a general rule, the debtor incurs in delay if he does not perform his
she gave the P10,000 with the request that she turn the money owe to her obligation on the date it is due.
husband. The money was stolen while in Romelita's possession. How much II. A waiver of an action for future fraud is valid.
may Jervin still recover from Leo? a. Both statements are false
a. P20,000 b. Both statements are true
b. P0 c. Statement 1 is true, statement 2 is false
c. P15,000 d. Statement 1 is false, statement 2 is True
d. P10,000
16. In the execution of obligation, liability for malice or bad faith:
8. On July 1, 2018, Kycer obliged himself to give Nicole a specific car if a. Extends only to results intended but excluding exemplary damages.
Nicole will not marry Edison on or before December 31, 2018. The b. Extends to natural consequences even if they exceed the debtor’s
condition of the obligation is a: exceptions.
a. Negative condition c. Is demandable in all obligations but may be renounced in advance.
b. impossible condition d. Extends not only to results intended but also to their foreseen consequences.
c. divisible condition
d. Positive condition 17. A sold his cow to B for P25,000. No date was stipulated for the delivery of
the cow. While still in the possession of A, the cow gave birth to a calf.
9. One of the following obligations is not immediately demandable. a. B is entitled to the calf which was born after the perfection of the contract.
a. Pure obligation b. A is entitled to the calf because it was born before his obligation to deliver
b. Obligation with an ex die period condition the cow arises.
c. Obligation with a resolutory c. A is entitled to the calf as B has not paid the price.
d. Obligation with an in-diem period d. B, in order to be entitled to the calf, should pay additional cost for the calf
to be agreed upon by both parties.

13

0 0
18. It is a type of fraud which is employed in obtaining consent but does not
necessarily vitiate consent
a. Causal Fraud
b. Fraud in Performance of Obligation 25. What is the effect of the loss of the prestations in alternative obligation
c. Incidental Fraud where the right of choice belongs to the creditor and the loss due to the fault
d. Accidental Fraud of the debtor?
First Answer: If only one remains the obligation still subsists, but it ceases
19. N, R and J solidarily bound themselves to deliver to S a Honda motorcycle to be alternative as it becomes a simple obligation;
valued at P 60,000. The obligation was not fulfilled through the fault of J. Second Answer: If 2 or more objects remain, the obligation still subsists and
Thereupon, S filed an action in court against N and the court awarded P the choice is limited to the remaining objects
72,000 to S representing the value of the motorcycle plus damages. Which Third Answer: If none remains, the Obligation is extinguished because the
of the following situation is valid? debtor chose not to give anything
a. N can refuse to pay the penalty because it should be charged, against J, the a. All answers are correct
guilty party b. Only one of the answers is correct
b. If N pays S the P 72,000, N can collect R and J P 24,000 each c. All answers are wrong
c. S has to collect P 24,000 each from N, R and J to satisfy the court's award d. Only the first and second answers are correct
of P 72,000
d. If S succeeds in collecting the P 72,000 from N, N in turn can collect from 26. What are the different acts or omission of the obligor or debtor which will
R P 20 000' and from J P 32,000 result in the breach of the obligation for which he can be held liable for
damages?
20. Alleria owes Rylai the following debts: P6,000 due on June 12; P6,000 due a. Fraud
on June 15; P6,000 due on June 18; and P6,000 due on June 20. All the debts b. Contravention of the tenor of the obligation
are unsecured except the debt due on June 20 which is secured by a pledge c. All of the above
of Alleria's diamond ring to Rylai. By agreement, the benefit of the term on d. Negligence
the 4 debts was made in favor of Rylai. Assuming that Allefia has P6,000 e. Default
on June 18 and is ready to pay Rylai, which of the following statements is
correct? 27. The court determines the amount of damage taking into consideration the
a. Alleria must apply the payment proportionately to the debts due as of June price of the thing and its sentimental value to the injured person
1? at P2,000 each. a. Restitution
b. Alleria may apply the payment to any of the four debts b. Reparation
c. Alleria may apply his payment of P6,000 to any of the debts due June 12, c. Starvation
June 15, and June 18 since they are all due as of June 18 d. Indemnification
d. Alleria may apply it only to the debt due June 20 because it is the most
burdensome to him 28. Anything paid or delivered before the arrival of the period, the obligor being
unaware of the period or believing that the obligation has become due and
21. The debtor losses the benefit of the period and his obligation becomes demandable
demandable when: a. May be recovered without interests
a. Demand by the creditor would be useless b. May be recovered with interests
b. The guarantees as promised and delivered by the debtor are not acceptable c. May be recovered without interests
to the creditor d. May be recovered with interests
c. The debtor attempts to abscond
d. After contracting the obligation, the creditor suspects that the debtor is 29. An obligation which is not subject to any condition and no specific date is
becoming insolvent. mentioned for its fulfillment and is therefore immediately demandable is:
a. Solidary obligation
22. Which of the following is wrong in an alternative obligation? b. Alternative obligation
a. The obligee cannot be*compelled to receive part of one and part of the other c. Indivisible obligation
undertaking d. Pure obligation
b. The debtor shall have no right to choose those prestations which are
impossible 30. Ka, Ba, Yo, and Kaw owe Papa, Sa, Bako the sum of P12,000. Based on the
c. The obligor shall completely perform one of them foregoing data, which of the following statements is incorrect?
d. The right of choice belongs to the creditor, unless it has been expressly a. Each creditor is entitled to collect a maximum of P4,000
granted to the debtor b. There are 7 distinct debts in the obligation
c. The obligation is joint
23. Legal compensation is allowed when one of the debts d. Each debtor is liable only for a total P3,000
a. Arises from a sale of real property made by a minor to a capacitated person
b. Arises from the obligation of a depository 31. A juridical person known as negotiorum gestio takes place
c. Consists in civil liability arising from a penal offense a. None of the above
d. Arises from a claim for support by gratuitous title b. When a person is appointed by a court to take the property or business of
another.
24. Which of the following is not considered as quasi-contract? c. When something is received and there is no right to demand it and it was
a. Those whose fulfillment depends upon chance like insurance. delivered through mistake.
b. Solutio indebiti d. When a person voluntarily takes charge of another’s abandoned business or
c. Negotiorum gestio property without the owner’s consent.
d. Reimbursement due the person who saved property during fire or storm
without the knowledge of the owner. 32. Jun offered Zo a specific parcel of land at a specific price. Jun gave Zo 60
e. When the third person, without the knowledge of the debtor, pays the debt days within which to accept. Zo agreed.
a. Within 60 days, Jun cannot withdraw but may increase the price.
b. There can't be no withdrawal because the period is binding.
c. Jun may withdraw or increase the price within the 60-day period
d. There can be no withdrawal because there is perfected option contract.

14

0 0
33. It is the failure to observe the care that the law requires to be observed in a
given situation.
a. Negligence
b. Culpa criminal
c. Quasi-delicts
d. Due care

34. A condition which is imposed in an obligation will be disregarded and will


therefore make the obligation immediately demandable 42. X, who has a savings deposit with Y Bank in the sum of Pl,000,000.00
a. If Lili passes the CPA Board Exam incurs a loan obligation with the said Bank in the sum of P800.000.00 which
b. If Lala kills Lele has become due. When X tries to withdraw his deposit, Y Bank allows only
c. If Lolo commits suicide P200.000.00 to be withdrawn, less service charges, claiming that
d. If Lulu will not rise from the dead compensation, has extinguished its obligation under the savings account to
the concurrent amount of X's debt. X contends that compensation is
35. Delay or default on the part of the obligor is known as improper when one of the debts, as here, arises from a contract of deposit.
a. Mora solvendi ex re Assuming that the promissory note signed by X to evidence the loan does
b. Mora solvendi not provide for compensation between said loan and his savings deposit,
c. Compensatio morae who is correct?
d. Mora solvendi ex persona a. X is correct since the compensation that must take place Is a facultative ode
e. Mora accipiendi whereby Y Bank must acquire the consent of X before offsetting
b. Y Bank Is correct because the silence of the promissory note does not intend
36. Jamila is obliged to give Christopher a specific ring. The parties agreed that to prohibited legal compensation, therefore, legal compensation may tak e
Jamila may give a specific bracelet as a substitute. Which of the following place
statements is true? c. Y Bank is correct because there a debtor-creditor relationship that exist
a. If the ring is lost through a fortuitous event after substitution, the obligation between the parties in the two contracts ,
is extinguished. d. X Is correct since the promissory note does not provide any stipulation
b. If the ring is lost through the debtor's fault after substitution, the debtor shall regarding compensation, hence, legal compensation may not take place
pay damages.
c. If the bracelet is lost through a fortuitous event before substitution, the 43. Jon owes Arya P50,000. Subsequently, Jon proposed to Arya that Bran will
obligation is extinguished. assume his (Jon) debt. Arya accepted the proposal of Jon. Assume that on
d. If the ring is lost through a fortuitous event before substitution, the due date, Bran could not pay because of his insolvency which was in fact
obligation is extinguished. subsisting but was not known to Jon or of public knowledge at the time that
Jon delegated his debt. In this case
37. When the debtor binds himself to pay when his means will permit him to do a. The novation is void because Jon did not take steps to determine the
so, the obligation is: solvency of Bran when he (Jon) delegated his debt
a. Pure b. Arya can revive Jon's debt because Bran's insolvency Was already existing
b. Conditional at the time that Ion delegated his debt
c. With a period c. Arya cannot hold Jon liable because his (Jon) obligation was extinguished
d. Simple when he was substituted by Bran -
d. Arya can revive Jon's debt whether or not he (Jon) was aware of Bran's
38. Liera, Ronald and Sandra are solidary debtors of Raymond for P3,000. insolvency since he (Jon) proposed the substitution
Raymond remitted Sandra's share. Liera therefore paid later only P2,000.
Liera can recover reimbursement from Ronald in the amount of 44. Printado is engaged in the printing business. Suplico supplies printing paper
a. P1,000 to Printado pursuant to an order agreement under which Suplico binds
b. P0 himself to deliver the same volume of paper every month for a period of 18
c. P500 months, with Printado in turn agreeing to pay within 60 days after each
d. P1,500 delivery. Suplico has been faithfully delivering under the order agreement
for 10 months but thereafter stopped doing so, because Printado has not
39. Facultative obligation as distinguished from alternative obligation made any payment at all. Printado has also a standing contract with
a. If it is impossible to give all except one, that last one must still be given publisher Publico for the printing of 10,000 volumes of school textbooks.
b. The right of choice is given only to the debtor Suplico was aware of said printing contract. After printing 1,000 volumes,
c. If one of the prestation is illegal, the others may be valid the obligation Printado also fails to perform under its printing contract with Publico.
remains Suplico sues Printado for the value of the unpaid deliveries under their order
d. Various things are due, but the giving of one is sufficient agreement. At the same time Publico sues Printado for damages for breach
of contract with respect to their own printing agreement. In the suit filed by
40. Demand must be made on the due date of the obligation in order for delay Suplico, Printado counters that: (a) Suplico cannot demand payment for
to exist in one of the following cases. Which is it? deliveries made under their order agreement until Suplico has completed
a. When the obligation does not indicate whether demand must be made or not performance under said contract; (b) Suplico should pay damages for breach
on due date of contract; and (c) with Publico should be liable for Printado's breach of his
b. When the law provides that demand need not be made contract with Publico because the order agreement between Suplico and
c. When time is of essence of the contract Printado was for the benefit of Publico. Are the contentions of Printado
d. When it was stipulated by the parties that demand need not be made tenable?
a. The first contention is not valid. However, the damages claim by Printado
41. Damages predetermined beforehand where Suplico should pay damages by reason breach of contract is valid
a. Liquidated provided that Printado also pay the damages cause by its breach. The last
b. Temperate contention is also valid since Suplico has knowledge about the contract of
c. Mora Publico and Printado which clearly portrays a stipulation pour atrui.
d. Actual b. The first contention is not valid. The damages claim by Printado where
Suplico should pay damages by reason breach of contract is also not valid
because Suplico has the right to stop supplying Printado because of the

15

0 0
latter’s breach of contract: The last contention is also not valid because c. Law
Suplico is not a party to the contract of Printado and Publico. d. Quasi-contracts
c. The first contention is not valid. The damages claim by Printado where
Suplico should pay damages by reason breach of contract is also not valid 52. The delivery of a private document evidencing a credit, made voluntarily by
because Suplico has the right to stop supplying Printado because of the the creditor to the debtor even before the debt is paid, extinguishes the
latter's breach of contract. The last contention, however, is valid since obligation by:
Suplico has knowledge about the contract of Publico and Printado which a. Express remission
clearly portrays a stipulation pour autrui. b. Implied remission
d. None of the above c. Novation
d. Compensation
45. A kind of obligation where each one of the debtors is proportionately liable
and the object is hot physically divisible into different parts. 53. Obligation which is susceptible of partial performance based on the purpose
a. Solidary-Indivisible obligation of the obligation:
b. Divisible-solidary obligation a. Divisible
c. Joint-divisible obligation b. Indivisible
d. Joint-indivisible obligation c. Joint
d. Solidary

54. Indivisibility as distinguished from solidarity


a. Refers to the legal tie or vinculum
b. Refers to the prestation which constitute the object of the obligation
46. Which of the following statements concerning delay is incorrect? c. Plurality of subject is indivisible
a. Demand, as a rule, is required, in order for delay to exist. d. When the obligation is converted into one indemnity for damages because
b. Delay, as a rule, exists, when the debtor does not perform his obligation on of breach, the character of the obligation remains
the date is due
c. There is no delay in obligations not to do
d. When time is of the essence of the contract, delay on the part of the debtor 55. The delivery and transmission of ownership of a thing by the debtor to the
exists despite the absence of demand creditor as an accept equivalent of performance is known as:
a. consignation
47. It is the meeting of one person of the qualities of creditor and debtor with b. application of payment
respect to the same obligation: c. Dation in payment
a. Remission d. Payment by cession
b. Novation
c. Confusion 56. Rudolf borrowed P IM from Rodrigo and Fernando who acted as solidary
d. Compensation creditors. When the loan matured, Rodrigo wrote a letter to Rudolf
demanding payment of the loan directly to him. Before Rudolf could
48. Which of the following obligations is not subject to a period? comply, Fernando went to see him personally to collect and he paid him.
a. payable “little by little” Did Rudolf make a valid payment?
b. payable “when my means permits me to” a. Yes, since the payment covers the whole obligation
c. payable soonest b. No, since Rudolf should have split the payment between Rodrigo and
d. payable whenever “I like it” Fernando
c. Yes, since Fernando was a solidary creditor, payment to him extinguished
49. "I will give you this book provided that if I like to have it back, you will the obligation
return the same to me. d. No, since Rodrigo, the other solidary creditor, already made a prior demand
a. B. The obligation is void, because the fulfillment depends upon the will of for payment from Rudolf
the creditor.
b. Combination of A and B. 57. Leo left home on a business trip without leaving anyone to take charge of
A. The obligation is void, because the fulfillment depends upon the will of the his poultry and piggery farm. While he was away, a storm visited the area
debtor. where the farm was located placing the farm in danger. Avedon, a neighbor
c. The obligation is valid, because the condition merely causes the loss of of Leo saw the need to act under the situation, so he summoned all his
rights already acquired helpers to save the farm by providing shelter to the animals and taking care
of their products, but incurring P10,000 in necessary and useful expenses in
50. A and B executed a conditional sale of the land of A with the latter making the process. Owing to the effort of Avedon, the farm of Leo was saved.
a downpayment and the balance to be paid after the squatters have been Under the circumstances, Leo is obliged to reimburse Avedon for the
removed. If the squatters were not removed after six months, the necessary and useful expenses which the latter incurred by reason of
downpayment shall be returned. A sued the squatters but failed to eject a. Contract
them. Thereafter, A offered to return the downpayment on that ground that b. Contract
the condition did not happen. B refused and demanded that A execute the c. Quasi-delict
absolute deed of sale, at which time he will pay the balance of the price. d. Negotiorum gestio
Incidentally, the value of the land has doubled by-that time. Who is correct
between them? 58. When the obligation consists in the delivery of a generic thing whose quality
a. A is correct because the suspensive condition failed to happen without his and circumstances have not been agreed upon
fault and the contract provides for the return of the downpayment if the a. The debtor can deliver of inferior quality
squatters were not removed. b. The debtor can deliver of superior quality even against the will of the
b. B has the right to receive the return of the downpayment since A failed to creditor
fulfill the condition of removing the squatters c. The purpose and other circumstance shall be taken into consideration in
c. B is correct having waived the condition and being the aggrieved party and determining the quality of the object to be delivered
A who was the one who failed to comply with the condition shall not benefit d. The creditor can demand a thing of superior quality
from his fault.
d. A is correct because the condition did not materialize and therefore he has 59. What is the basis of liability of a school when a student is stabbed inside the
no obligation to sell his land to B. campus by a stranger in the school?
a. Contracts
51. The source of obligations which is a rule of conduct, just and obligatory, b. Quasi-delicts
promulgated by legitimate authorities for common good, benefit and c. Quasi-contracts
observance d. Delicts
a. Delicts
b. Contracts

16

0 0
60. A has been leasing to B his building for a number of years and repeatedly 67. An obligation which then subject matter is a thing which the obligor must
assured the latter that if he should decide to sell the same he will give the deliver to the oblige:
latter the right first refusal. On June 30,2016, A informed B that he was a. Real obligation
willing to sell the building for P5M. The following day latter sent a letter to b. Mixed obligation
the former offering to buy the building for P4.5M. A did not reply. One c. Obligation to do or no to do
week later, B received a letter from C informing him that the building has d. Personal obligation
been sold to him by A for P5M and that he will not renew the lease when it
expires. B is now claiming damages from A and wants to compel A to 68. It is the care that the law requires to be observed in a given situation.
execute the absolute deed of sale in his favor. Decide. a. Culpa criminal
a. A is not liable for damages but he may be compelled to execute the absolute b. Due care
deed of sale. c. Quasi-delicts
b. A is liable for damages but he may not be compelled to execute the absolute d. Negligence
deed of sale because C is an innocent purchaser for value.
c. A is liable for breach of promise to sell because B already sent a letter to 69. D borrowed P500,000 from C payable at the end of five years. Due to
him offering to buy at P4.5M to which he did not reply. supervening extraordinary inflation, the value of the debt fell to P250,000
d. A is liable for damages and cannot be compelled to execute the absolute on the date of maturity. How much must D pay C on the date of maturity?
deed of sale because there was no sale perfected between them. a. P250,000
b. P1,000,000
61. The creditor has a right to the fruits of the thing subject matter of the c. P500,000
obligation d. P2,000,000
a. From the time the fruits are physically delivered to him
b. From the perfection of a contract 70. Culpa aquiliana as distinguished from culpa contractual
c. From the moment of the meeting of the minds a. The negligence of the defendant is merely an incident in the performance of
d. From the time the obligation to deliver the principal thing-arises the obligation
b. Proof of due diligence in the selection and supervision of employees is not
available as a defense
c. The source of liability is the defendant's negligent act or omission itself
d. Proof of the contract and of its breach is sufficient prima facie to warrant
recovery

62. D obliged himself to give a specific car to C on December 25, 2022,


stipulating that D is liable even if the thing is lost due to fortuitous event,
and without the need of a demand. On the due date, the car got lost to
fortuitous event. Which of the following is correct? 71. Statement 1: the receipt of later installment of a debt without reservation as
a. C can require another person to deliver a car, expenses chargeable to D. to prior installments, shall give rise to a rebuttable presumption that such
b. Obligation is totally extinguished. installment has been paid.
c. Obligation remains to subsist, but converted into monetary consideration. Statement 2: if a taxpayer pays his income tax liability for the current year,
d. C can compel D to deliver another car. there is a presumption that tax liability for the previous year has been paid.
a. Group of answer choices
63. The return of what has been paid by mistake is known as: b. False, true
a. negotiorum gestio c. True, false
b. natural obligation d. True, true
c. quasi-delict e. False, false
d. solutio indebiti
72. Which of the following is a remedy of the creditor to enforce payment of his
64. Three of the following statements pertain to natural obligation. Which one claims against the debtor?
does not? a. Impugn the acts which the debtor may have done to defraud his creditors
a. obligation exists by reason of equity and moral justice b. Pursue the property in the possession of the debtor including those exempt
b. there is juridical necessity to perform it by law
c. if performed voluntarily, recovery can no longer be made c. All of the above
d. it is not enforceable in court of justice d. Exercise all the rights and bring all the actions of the debtor including those
personal to him
65. Manali Ngataw, a pedestrian, was hit by a taxicab because of the
recklessness of the taxi driver. In its effort to avoid the accident, the driver 73. If an obligation is with a penal clause
swerved the car to the left and bumped an electric post. Pesato, a passenger I. proof of actual damages suffered by the creditor is not necessary in order
of the taxi suffered injuries. Which of the following statements is not that the penalty may be demanded
correct? II. when the debtor is guilty of fraud in the fulfillment of the obligation,
a. Pesato can bring action of culpa aquilana against the driver and the owner the creditor can demand payment of damages and interest in addition to the
of the taxicab. penalty
b. Manali Ngataw can bring action of culpa aquilana only against the driver. a. False, false
c. Manali Ngataw can bring action of culpa criminal and culpa aquilana against b. True, false
the driver and the owner of the taxicab. c. True, true
d. Pesato can bring action of culpa criminal and culpa contractual against the d. False, true
driver and the owner of the taxicab.
74. Statement 1: Liability for damages arising from fraud is demandable and
66. A passenger on a bus was hurt, but in a criminal case against the driver, said there can be a waiver of an action for past fraud.
driver was acquitted. The victim now sues the owner of the truck for culpa Statement 2: There can be a waiver of an action for future negligence but
contractual. May the suit still prosper? not fraud.
a. No, this will constitute double jeopardy a. Second is true, first is false
b. Yes, provided he can prove the negligence of the driver b. First is true, second is false
c. No, the acquittal means that the guilt of the accused was not proven by proof c. Both statements are false
beyond reasonable doubt d. Both are true
d. Yes, it is sufficient for him to prove the existence of the contract of carriage
and the injuries suffered 75. A thing is not considered lost when it:
a. Deteriorated
b. Stolen or robbed

17

0 0
c. Goes out of commerce d. Fraud in obtaining consent
d. Perishes
85. Which of the following is an accessory thing?
76. In a joint obligation, joint means any of the following, except: a. Land
a. Proportionate b. Video camera
b. Pro-rata c. Milk from cow
c. Individually and collectively d. Pendant of a necklace
d. Mancomunada simple
86. Alternative obligation and facultative obligation are similar in which of the
77. Consignation alone without tender of payment is sufficient in the following following respects?
cases, except: a. The right of choice may be given either to the debtor or creditor
a. When two or more persons claim the same right to collect b. The obligation becomes a simple obligation once the choice of the prestation
b. When the creditor presents the title to the obligation for collection is made and communicated
c. When without just cause, the creditor refuses to give a receipt c. Several prestations are due
d. When the creditor is absent or unknown or does not appear at the place of d. Only one prestation is due but the debtor may render another in substitution
payment
87. An obligation where only one prestation is due with no period or condition
78. A owes B P11.000 due on July 2, 2010. B owes A P6,000 due on July 3, needed is called:
2010 and P4,000 due on July 10, 2010. B owes C P11,000 due on July 3, a. Indivisible obligation
2010. On July 2, 2010 B cannot pay C so B assigns to C her credit of Pll,000 b. Conjoint obligation
against A, without the knowledge of A. On July 10, 2010 C tries to collect c. Pure or simple obligation
from A the P11,000. How much can C compel A to pay? d. Alternative obligation
a. 5,000
b. 11,000 88. Sidfrey was employed as a professional driver of IlanNaKayaMaliKo Bus
c. 1,000 Company owned by Rhea. In the course of his work, Sidfrey hit a pedestrian
d. 9,000 who was seriously injured and later died in the hospital as a result of the
accident. The victim's heirs sued the driver and the owner of the bus for
79. Which of the following is an indivisible obligation? damages. Which of the following statements is not correct?
a. To pay a loan obligation of P 100,000 in 10 equal monthly installments a. There is a presumption of negligence on the part of the employer Rhea if
b. To construct a pavement measuring 2 meters wide and TO meters long passenger is injured
c. To lecture for 5 Saturdays in a tax seminar b. The liability of Sidfrey shall cease when he proves that he observed ordinary
d. To give a rendition to night of Handel's "Messiah" in a concert at the Folk diligence to prevent the damage to passengers and pedestrian
Arts Theater c. The conviction of Sidfrey in a criminal case makes Rhea liable for damages
arising from criminal act
d. The presumption that Sidfrey is negligent is rebuttable by proof of
observance of ordinary diligence

80. Fernando, Ricky and Maximo solidary owe Romel, Leo, Jayson and John, 89. Whenever a period is designated in an obligation, the said period shall be
P30,000. How much can, Romel collect from Ricky? presumed to have been established for the benefit of:
a. P5,000 only a. The debtor
b. P10,000 Romel in turn has to give P2,000 each to Leo, Jayson and John b. Both creditor and debtor
c. P7,500 only c. neither of the parties
d. P30,000 Romel in turn has to give P5,000 each to Leo, Jayson, and John d. the creditor

81. In an obligation to give a determinate thing, what rights are available to the 90. An obligation subject to the happening of a future and certain, event is
creditor? a. Group of answer choices
First Answer-to compel specific performance b. Conditional obligation
Second Answer - to recover damage in case of breach of the obligation c. Resolutory conditional obligation
Third Answer - To ask that the obligation be Complied with at-the expense d. Suspensive conditional obligation
of the debtor e. Obligation with a period
a. Only the first and second answers are correct
b. Only the first answer is correct 91. If the obligation is determinate, one of the duties imposed upon the debtor
c. All answers are correct is?
d. All answers are wrong a. To deliver all accession and accessories
b. To ask performance
82. Gemma, May Ann, and Christian executed a promissory note worded as c. All of the above
follows: "We promise to pay Daryl, Lea, and Joyce the sum of P90,000 Sgd. d. To deliver the thing of the same kind
Gemma, May Ann, Christian". Which is correct?
a. Gemma is obliged to pay to Daryl P60,000 92. Bhavik, a manufacturer of locks, hired Weshley for 2 years, on the condition
b. Gemma is obliged to pay Daryl, Lea and Joyce P90,000 that for 5 years, Weshley should not engage in competitive locks business.
c. Gemma is obliged to pay to Joyce P10,000 After one year, Weshley left for reasons of health. Shortly afterwards, after
d. Gemma is obliged to pay to Lea P30,000 regaining his health, Weshley competed with Bhavik, who now seeks to
restrain him from such competition. Will the action prosper?
83. In tender of payment and consignation, if after consignation is made and the a. Yes, if Bhavik will allow Weshley to complete the 2 years services agreed
creditor allowed the debtor to withdraw the thing deposited court, which of upon.
the following statements is incorrect? b. No, the restriction is void, because it is an unreasonable restraint of trade
a. None of the above c. No, the contract of employment for 2 years was not completed because of a
b. The obligation remain to subsist justifiable reason
c. Co-debtors, guarantors, and sureties are release from .the obligation unless d. Yes, it’s a reasonable restraint, considering that it was only for 5 years
they consented
d. The obligation is extinguished 93. In 2016, Merlyn, Eden, and Dana bound themselves in solidum to give
Trisha P9,000 subject to the following conditions. Merlyn will pay in 2017,
84. It is a type of fraud which is employed after the constitution of an obligation. Eden, if Trisha passes the 2018 CPA board exam and’ Dana will pay in
a. Accidental Fraud 2019. In 2017, how much can Trisha demand from Dana?
b. Incidental Fraud a. P0
c. Causal Fraud b. P6,000

18

0 0
c. P9,000
d. P3,000

94. Baron is obliged to give Jessa P10,000 if X dies. This is an example of:
a. An obligation with a period
b. An obligation with a suspensive condition
c. A pure obligation
d. An obligation with a resolutory condition

95. Change of persons or objects


a. Solutio indebiti
b. None of the above
c. Confusion
d. Novation

96. A, B, and C are liable to X to deliver the P90,000 sack of sugar in Y


warehouse on January 5, 2008. On January 6, 2008, X demanded from& to
deliver the sugar. However, on January 8, 2008, a fire broke in Y warehouse
because of lightning and the entire 90,000 sacks of sugar was totally loss
a. X can hold of A, B, and C liable for 90,000 sacks of sugar because they are
already in delay before the object was loss by fortuitous event
b. X can hold each of them (A, B, C) liable for 30,000 sacks of Sugar because
they are already in delay before the object was loss by fortuitous event
c. A is liable for the 30,000 sack of sugar because he was already in delay but
B and C's obligations are extinguished because they are not in legal delay
and the loss was due to fortuitous event
d. A, B, and C is not liable because the loss was due to fortuitous event

97. Under a building contract, Avedon bound himself to build a house for Jervin
for Pl,000,000. In this type of obligation, who is the passive subject?
a. Either Avedon or Jervin
b. Neither Avedon nor Jervin
c. Avedon
d. Jervin

98. Riyah owned a house rented by Rhae. Riyah sold the house to Rhea where
Rhea agreed to the balance of the purchase price as soon as Rhae leaves the
premises. It was further agreed that Rhea will take care of seeing to it that
Rhae vacates the house. Which is correct?
a. The contract is void because the consent of Rhae was not obtained
b. The contract is valid because the condition is mixed
c. The contract is void because it is potestative on the part of Rhea
d. The contract is valid if Rhae is willing to vacate the premises

99. In an obligation worded as follows: "I promise to continue allowing X to


stay in my condo until he marries", the obligation is subject to:
a. Suspensive period
b. Resolutory period
c. Suspensive condition
d. Resolutory condition

100.The consequential damages suffered by the injured person and those


suffered by his family or third Person by reason of the act
a. Reparation
b. Indemnification
c. Restitution
d. Starvation

19

0 0
c. Mutual promise to marry.
1. S sold one of his lot to B for P100,000.00 The lot, located beside a busy d. Contract not to be performed within a year from the making thereof.
street, consisted of 1,200 square meters and was covered by Transfer
Certificate Title (TCT) No. 12345 After the registered the sale with the
Register of Deeds and the issuance to him by the said office of a new transfer
certificate of title in his name, B discovered that the deed of sale and the 10. These persons are bound by contracts
new title described S’s adjoining lot which, although also consist of 1,200 a. All of them
square meters, was covered by TCT No. 12345 and located farther from the b. Assigns or assignees
street. B learned that C, the secretary of S typed the wrong description on c. Contracting parties
the deed of sale. B now approaches you for advice on which course of action d. Heirs
take What advice will you give to B?
a. Reformation because the deed of sale failed to show the true intention of the 11. If the obligation is payable in foreign currency:
parties to the contract. a. the stipulation and the obligation are void.
b. Rescission because the erroneous description has caused him damages. b. the creditor can compel the debtor to pay in foreign currency as per
c. You Answered agreement.
d. Declaration of nullity of the contract because it is void ab initio by reason c. the obligation is valid, but the stipulation is void.
of the wrong object. d. the obligation is void.
e. Annulment of the contract because he did not consent to the purchase of the
lot covered by TCT No. 12345 12. If there is a concurrence of two or more creditors and/ or two or more debtors
in one and the same obligation, the obligation is presumed to be:
2. Must be in writing to be enforceable a lease of Land for 12 months a. joint
a. Both of the choices are true b. solidary
b. Lease of Land for 12 months c. individual and collective
c. None of the choices d. joint and several
d. Lease of land for 18 months
13. Amazed by the skill of X, who can paint portraits using paint brushes
3. The statement “Contracts shall be obligatory in whatever form they have inserted in his right armpit, Y entered unto a contract with the former to have
been entered into provided all the requisites for their validity are present” his own portrait done and gave X P50,000 as full payment therefor. After he
refers to has started painting the portrait but before its completion, X was sideswiped
a. Formal contracts by a speeding car while he was drunk and walking beside the railway
b. Solemn Contracts causing the doctors to have his right arm amputated. In this case, what is the
c. Consensual contracts remedy of Y?
d. Real contracts
a. Equivalent performance in terms of damages.
4. Requisites of application of payment, except: b. Substitute performance by allowing a third person to do his portrait with
expenses therefor charged to X.
a. One debtor and one creditor c. Nothing, as the service became impossible by reason of an act independent
b. Two or more debts of the will of the debtor.
c. Debts are all due d. Specific performance by compelling X to go on painting Y’s portrait.
d. Debts are different kinds
14.Reformation is not available in the following cases except:
5. One which is perfected by the delivery of the thing which is the subject a. Simple donations inter vivos where no condition is imposed.
matter of the contract b. Wills
a. Consensual contract c. When through the ignorance, lack of skill, negligence or bad faith on the
b. Contract part of the person drafting the instrument or of the clerk or typist, the
c. Real contract instrument does net express the intention of the parties
d. Rescissible contracts d. When the real agreement is void.

6. Extinguishment of debt may be made by creditor either verbally or in 15. Daisy borrowed P15,000 from Candy. On due date, Daisy was not able to
writing: pay but she promised to give Candy a specific ring, a specific bracelet, or a
a. Implied remission specific necklace, in payment of the debt. Candy accepted the offer from
b. Suspensive condition Daisy.
c. Express remission How was the obligation of Daisy to pay P15,000 extinguished?
d. Partial payment
a. By novation
7. A obliged to give B his only car on September 1, 2022. On the said date, A b. By confusion
did not deliver. On September 2, 2022, an earthquake completely destroyed c. By condonation
the car. Is A still liable? d. By compensation
a. Yes. A is already in legal delay, thus, the obligation to deliver the lost
specific thing is converted into monetary claim for damages. 16. A sold his cow to B for P25,000. No date was stipulated for the delivery of
b. No. The obligation is extinguished, even if the debtor is already in default the cow. While still in the possession of A, the cow gave birth to a calf.
because the debtor can plead impossibility of performance. a. B, in order to be entitled to the calf, should pay additional cost for the calf
c. No. Considering that no demand to deliver was made by B and the specific to be agreed upon by both parties.
thing was lost due to fortuitous event, the obligation is extinguished. b. A is entitled to the calf as B has not paid the price.
d. Yes, The creditor can instead demand for a substitute of equivalent value c. B is entitled to the calf which was born after the perfection of the contract.
from the debtor. d. A is entitled to the calf because it was born before his obligation to deliver
the cow arises.
8. In the execution of obligation, liability for malice or bad faith:
a. Extends to natural consequences even if they exceed the debtor’s 17. Reformation of instrument has the following requisites, except
exceptions. a. The failure of the instrument to express the true intention of the parties is
b. Is demandable in all obligations but may be renounced in advance. due to mistake, fraud, inequitable conduct or accent.
c. Extends not only to results intended but also to their foreseen consequences. b. The contract must be in a public instrument.
d. Extends only to results intended but excluding exemplary damages. c. There must be a meeting of minds of the parties to the contract.
d. The true intention of the parties is not expressed in the instrument.
9. Statute of Fraud is applicable to
a. Oral contract of loan 18. Contract that is made for a valuable consideration is
b. Executed contract a. Onerous

20

0 0
b. Gratuitous d. Both statements are true.
c. Consensual
d. Real

19. A obtained a loan of Php 300,000 from B payable on March 25, 2022. As
security for the payment of his loan, A constituted a mortgage on his
residential house and lot in B’s favor. C,. a good friend of A, guaranteed and
obligated himself to pay B in case A fails to pay his loan at maturity. 27. A obtained a loan of Php 300,000 from B payable on March 25, 2022. As
If A sells his residential house and lot to D, can B foreclose the real estate security for the payment of his loan, A constituted a mortgage on his
mortgage? residential house and lot in B’s favor. C,. a good friend of A, guaranteed and
a. No, B cannot foreclose the real estate mortgage. The sale confers ownership obligated himself to pay B in case A fails to pay his loan at maturity.
on the buyer, D who must therefore consent. If A fails to pay his loan on March 25, 2022, can B compel C to pay?
b. No, B cannot foreclose the real estate mortgage. To deprive the new owner a. No, B cannot compel C to pay because as guarantor, C can invoke the
of ownership and possession is unjust and inequitable. principle of excussion, i.e., all assets of B must first be exhausted.
c. Yes, B can foreclose the real estate mortgage. It is binding upon D as the b. No, B cannot compel C to pay because B has not exhausted the available
mortgage is embodied in a public instrument. remedies against A.
d. Yes, B can foreclose the real estate mortgage because real estate mortgage c. Yes, B can compel C to pay because the nature of C’s undertaking indicates
creates a real right that attaches to the property. that he has bound himself solidarily with A.
d. Yes, B can compel C who bound himself to unconditionally pay in case A
20. Not an essential requisite of a contract fails to pay, thus the benefit of excussion will not apply.
a. Prestation
b. Meeting of the minds 28. A offered 20 Tamiya cars to B for P1,000.00 each. B answered by letter that
c. Formalities he was willing to purchase not 20 but 30 pieces at said price of P1,000.00
d. Consideration each. Is the contract perfected?
a. No, because there is qualified acceptance which constitutes an offer by the
21. When the debtor binds himself to pay when his means permit him to do so, buyer
the obligation is: b. No, because there is no offer yet of A that is certain
a. Simple c. Yes, there is perfected sale because the number of pieces is only incidental
b. Conditional matter in the sale
c. Pure d. Yes, because the letter of acceptance was already sent by the offeree
d. With a period
29. The following statements pertain to either payment by cession or dacion en
22. An obligation where only one prestation is due with no period or condition pago.
needed is called: I. The debtor is insolvent
a. Conjoint obligation II. Ownership of the thing/s is transferred to the creditor/s
b. Pure or simple obligation III. Plurality of creditors is required
c. Alternative obligation IV. Obligations are totally extinguished
d. Indivisible obligation a. Statements I and IV pertain to payment by cession.
b. Statements III and IV pertain to payment by cession.
23. S sold a parcel of land to B for 100,000.00 with B paying the said amount c. Statements II and IV pertain to dacion en pago.
immediately. Although S delivered the transfer certificate of title of the land d. Statements I and III pertain to dacion en pago.
to B, the parties did not execute any document at all for the sale except the
receipt of the payment which S issued to B. B now wants to register the sale 30. The first step in the different stages of a contract is
with the Register of Deeds. a. Perfection
a. The contract between S and B rescissble; hence B can sue damages. b. Preparation
b. The contract between S and B is void because it was not in the form required c. Performance
by law. d. Consummation
c. B can compel S to execute a Deed of Sale which is duly notarized so that B
can register the Sale. 31. D obliged himself to give a specific car to C on December 25, 2022,
d. B cannot compel S to execute a Deed of Sale because the contract is stipulating that D is liable even if the thing is lost due to fortuitous event,
unenforceable. and without the need of a demand. On the due date, the car got lost to
fortuitous event. Which of the following is correct?
24. When there is concurrence of offer and acceptance, there is
a. Cause a. Obligation is totally extinguished.
b. Subject matter b. C can compel D to deliver another car.
c. Prestation c. Obligation remains to subsist, but converted into monetary consideration.
d. Consent d. C can require another person to deliver a car, expenses chargeable to D.

25. Which of the following statements is false? 32. D has an obligation to give a BMW car model 2020 to C on January 17,
a. Proof of actual damages suffered by the creditor is not necessary in order 2020. D failed to perform his obligation on maturity date and on January 20,
that the penalty may be demanded. 2020; the only car of D (also BMW model 2020) was totally destroyed due
b. Condonation is essentially gratuitous. to fortuitous event. Therefore:
c. The nullity of the principal carries with it that of the penal clause. a. D’s obligation is not extinguished because he is already in delay.
d. I will pay you P10,000 “soonest” is for the benefit of the debtor. b. D’s obligation is extinguished because of impossibility of performance.
c. D’s obligation is not extinguished because his obligation is a generic thing,
26. Statement 1: Payment by cession does not totally extinguish the whole d. D’s obligation is extinguished because his car is destroyed due to fortuitous
obligation. event.
Statement 2: In obligation with penal clauses, it is necessary for the creditor
to show proof of actual damages suffered by him on account of the non- 33. The stage of death of a contract is
performance of the obligation by the debtor. a. Where there is a meeting of the partners minds
a. Statement 1 is false while statement 2 is true. b. When negotiation are in progress
b. Both statements are false. c. When the parties come to an agreement
c. Statement 1 is true while statement 2 is false. d. When the contract is fully executed

21

0 0
b. Those contrary to law
34. In alternative obligation, the debtor loss the right of choice when: c. Those contrary to public order
a. Among the prestations whereby he is alternatively, only one is d. Those contrary to morals
practicable.
b. Some of the prestations in alternative obligation is/ are impossible, 42. Tina signed a letter addressed and delivered to Rica. The terms of the former
unlawful or which could not have been the object of the obligation. are:
c. The debtor already communicated his choice to the creditor. An offer to sell a 400 sq m lot for P400, 000
a. A, B and C An option time up to July 31, 2020 for Rica to raise the P400,000.
b. B and C Upon payment of the P400, 000 Tina will execute and sign a Deed of Sale
c. A and C On July 31, 2020, Tina sent letter to Rica asking for a new price of P500,000.
d. A and B Can Rica compel Tina to accept the P400,000 and make her sign and execute
a Deed of Sale?
a. Yes, there was actual meeting of the minds
b. Yes, the contract is perfected.
c. Yes, Tina is already estopped by her signed letter
35. This question has been regraded. d. No, the offer was not accepted
The essential or more proximate purpose which the contracting parties have
in view at the time of entering into the contract is
a. object
b. motive
c. cause
d. consideration 43. The kind of compensation which may only be raised by the creditor and not
by the debtor in the obligation to give gratuitous support:
36. S sold his lot consisting of 1000 square meter to B. However,the deed of a. Judicial compensation
sale signed by the parties showed a total area of 1200 square meter due to b. Facultative compensation
the secretary's mistake. Later, S discovered the mistake. What remedy is c. Legal compensation
available to either party? d. Conventional compensation
a. Declaration of nullity of the contract.
b. Reformation 44. The return of what has been paid by mistake is known as:
c. Annulment a. natural obligation
d. Rescission b. negotiorum gestio
c. solutio indebiti
37. Daisy borrowed P15,000 from Candy. On due date, Daisy was not able to d. quasi-delict
pay but she promised to give Candy a specific ring, a specific bracelet, or a
specific necklace, in payment of the debt. Candy accepted the offer from 45. S and B entered into a contract where they made it appear that S sold to B a
Daisy. parcel of land worth P50,000.00. In reality, however, S borrowed from B,
Assume that before Daisy could deliver any of the ring, bracelet or necklace, P50,000.00 with S mortgaging the parcel of land as security for the debt.
the ring and the bracelet were lost successively through the fault of Daisy. a. S and B are bound by the contract of loan and mortgage if third persons are
In this case: affected.
a. Daisy may deliver the necklace to Candy without any obligation to pay b. S and B are bound by the contract of loan and mortgage
damages. c. S and B are not bound at all
b. Consuelo may demand the payment of the price of the bracelet which was d. S and B are bound by the contract of sale.
the last item that was lost, plus damages.
c. Daisy may deliver the necklace to Candy with the obligation to pay damages 46. Not a requisite for a valid consent
because the ring and the necklace were lost through her fault. a. Person must be capacitated
d. Candy may demand payment of the price of the ring or the price of the b. Consent must be free
bracelet plus damages, or the delivery of the necklace. c. Parties are physically present
d. Consent must be intelligent
38. Daisy borrowed P15,000 from Candy. On due date, Daisy was not able to
pay but she promised to give Candy a specific ring, a specific bracelet, or a 47. The Statute of fraud applies only to:
specific necklace, in payment of the debt. Candy accepted the offer from a. Wholly executed contracts.
Daisy. b. Contracts wholly or partially executed on the part of the debtor.
What kind of obligation is the new obligation of Daisy? c. Wholly executory contracts.
a. Facultative obligation d. Contracts wholly or partially executed on the part of the creditor.
b. Compound obligation
c. Simple obligation 48. Like incapacity, force or violence, the following cause vitiate consent or
d. Alternative obligation render it defective so as to make the contract voidable except
a. Threat
39. D obliged himself to give C 100 cavans of rice on December 25, 2022. On b. Undue influence
said date, D failed to make delivery, despite repeated demands from C. c. Fraud
a. D can rescind the contract because the object is indeterminate. d. Reverential fear
b. C has no remedy under the law.
c. C may ask a third person to deliver 100 cavans of rice to him, the value 49. Fraud in the perfection of pre existing obligation
recoverable from D plus damages. a. Dolo incidente
d. C can compel D to deliver 100 cavans of rice plus damages. b. Dolo dicente
c. Dolo cesante
40. I. The principle of autonomy of contracts means that the contracting parties d. Dolo causante
as rule may agree upon any stipulation, clause, term and condition
II. Relativity of contracts means that contracts take effect not only between 50. Three of the following are essential elements of a contract except
the parties but also their heirs and assigns a. Consent of the contracting parties
a. True, False b. Motive of the parties
b. True, true c. Object certain which is the subject matter of the contract
c. False, True d. Cause of the obligation
d. False, false
51. Which of the following can be a valid object of a contract?
41. Which of the following is not a limitation upon the right to contract? a. Bay walk in Manila
a. Those contrary to third person b. Right of suffrage under the constitution

22

0 0
c. Mount Everest a. Conjoint obligation
d. Creditor's right to demand payment b. Simple obligation
c. Facultative obligation
52. Mr. Debtor owes Mr. Creditor who has two (2) legitimate children, P50,000 d. Alternative obligation
payable on December 31, 2022.
a. If both Mr. Debtor and Creditor die, the heirs of Mr. Creditor can collect 62. The following contracts are required to appear in a public document for the
from the heirs of Mr. Debtor. convenience of the parties so that they may be rented to the proper recording
b. If Mr. Debtor dies before December 31, 2022, Mr. Creditor can collect from office, except
the heirs of Mr. Debtor. a. The acceptance of an inheritance
c. If Mr. Debtor dies before December 31, 2022, Mr. Creditor cannot collect b. The cession of actions or rights proceeding from an act appearing in a public
from the heirs of Mr. Debtor. document.
d. If Mr. Creditor dies, his two (2) legitimate emancipated children cannot c. The power to administer property.
recover from Mr. Debtor his obligation. d. Countries which have for their object the creation ot real rights over
immovable property
53. By this principle contracts take effect only upon the contracting parties, their
assigns or successors in interest 63. Which of the following obligations is not subject to a period?
a. Relativity of contract a. payable “little by little”
b. Obligatoriness of contract b. payable “when my means permits me to”
c. Freedom of contract c. payable soonest
d. Mutuality of contract d. payable whenever “I like it”

54. The following except one are characteristics of a contract


a. Consensual
b. Liberty
c. Relativity 64. On January 1, 2025, D borrowed P500,000 from C with interest at 10% per
d. Obligatoriness annum. The loan obligation and the interest are due on December 31, 2025.
I. D may compel C to accept payment before December 31, 2025.
55. A solemn or formal contract has the following essential elements: II. C may compel D to make payment before December 31, 2025.
a. Consent of the contracting parties, object certain cause or consideration and In your evaluation of the foregoing statements:
formalities required by law. a. Both statements are false.
b. Consent of the contracting parties, object certain, delivery of the object, and b. Both statements are true.
formalities required by law. c. Only Statement I is true.
c. Consent of the contracting parties, object certain, cause or consideration and d. Only Statement I is false.
delivery of the object.
d. Consent, of the contracting parties, object certain, and cause or 65. The contract must bind both contracting parties, its validity or compliance
consideration. cannot be left to the of one of them, and this is
a. Obligatoriness of contract
56. Reformation of an instrument is available in the following cases except b. Relativity of contract
a. When a mutual mistakes of the parties causes the failure of the instrument c. Freedom of contract
to disclose their agreement. d. Mutuality of contract
b. When one party was mistaken and the other party acted fraudulently or
inequitable in such way that the instrument does not show the true intention 66. The reason why the obligation exists.
c. When party was mistaken and the other knew or believed that the instrument a. Prestation
did not state their real agreement. b. Passive subject
d. When one of the parties has brought an action to enforce the contract c. Vinculum juris
d. Active subject
57. The following cannot give consent to a contract, except
a. Minors who represent themselves as of legal age to one in good faith 67. When one of the parties to a contract is compelled to give his consent by a
b. Minors, except sale of necessaries in life reasonable and well grounded fear of an imminent and grave evil upon his
c. Insane persons person or property, or upon the person or property of his spouse,
d. Demented descendants or ascendants, there is
a. Violence
58. When in order to wrest consent, serious or irresistible force is employed b. Intimidation
there is c. Force
a. Violence d. Undue influence
b. Intimidation
c. Mistake 68. A appointed B to sell his land.
d. Fraud I. If the authority of B is oral and the sale is in public instrument, the sale is
void,
59. This is the accord or coincidence of the desires and intent of the parties to II. IF the authority of B is in writing and the sale is oral, the contract is
the contract upon the subject matter and the cause. unenforceable
a. Consent a. Both are true
b. Reformation b. The best statement is true; second statement is false
c. Payment c. The first statement is false: second statement is true
d. Ratification d. Both are false

60. L entered into a contract of mortgage with X. T, the clerk of typed the 69. Under the Statute of Fraud, certain contracts must be in writing to be
document. Due to T's negligence, the document made was that of sale enforceable. One of the following contracts is enforceable although not in
instead of mortgage. writing. Which is it?
a. The remedy is annulment a. A representation as to the credit of a third person
b. Parties may enforce their right because it is enforceable b. An agreement made in consideration of marriage other than mutual promise
c. Parties may go to court for interpretation. to marry
d. Reformation of instrument is proper. c. Sale of immovable at a price of not less than P500.00
d. An agreement for the leasing for a longer period than one year of a personal
61. An obligation wherein various things are due, but the payment of one of property.
them is sufficient to extinguish the obligation is called:

23

0 0
70. A, B and C executed a promissory note worded as follows:
“We promised to pay X, Y and Z the sum of P90,000. (SGD) A, B, and C”.
a. A is obliged to pay to X, Y and Z, P90,000.
b. A is obliged to pay to X, P60,000.
c. A is obliged to pay to X, P10,000
d. A is obliged to pay to X, P30,000.

24

0 0
1. Three of the following contracts are inexistent and void from the beginning.
Which is the exception?
a. That whose purpose is contrary to law or public order
b. That which is absolutely simulated or fictitious 11. Mr. Monterola owes Mr. Cera P100, 000. Mr. Monterola knows that on
c. That which contemplates an impossible service maturity date, he will not be able to pay Mr. Cera, and in order to prevent
d. That where both parties are incapable of giving consent to a contract attachment of his property by Mr. Cera, Mr. Monterola, before maturity of
his debt, executed a contract pretending to sell to Mr. Canave his property.
2. An absolutely simulated contract is Which of the following statements is not correct?
a. Unenforceable a. The contract is not valid for lack of consideration
b. Voidable b. The contract being simulated and executed to defraud Mr. Cara is void
c. Void c. The contract is binding between Mr. Monterola and Mr Canave
d. Rescissible d. Mr. Cera can seek rescission of the fictitious contract

3. A contract where both parties are incapable of giving consent is 12. Which is the least defective contract?
a. Unenforceable a. Voidable contract
b. Voidable b. Rescissible contract
c. Void c. Unenforceable contract
d. Voidable d. Inexistent contract

4. Contracts entered into during lucid interval. 13. Contract which have no effect at all and cannot be rated is a/an
a. Voidable a. Inexistent contract
b. Unenforceable b. Unenforceable contract
c. Void c. Voidable contract
d. Valid d. Rescissible contract

5. A contract where one of the contracting parties is incapable of giving 14. Failure of cause will render a contract
consent is a. Unenforceable
a. Rescissible b. Rescissible
b. Void c. Voidable
c. Voidable d. Void
d. Unenforceable
15. In a contract a written, D promises to pay C P10, 000 on March 5, 2020. The
6. Which of the following contracts is valid? consideration received by D is not stated in the contract
a. Oral partnership agreement where immovable property is contributed a. The contract is valid because the cause is always presumed to exist
b. Oral contract of sale of an immovable property entered into by an agent who b. The contract is valid because cause is not essential to a contract
was given authority orally by the principal c. The contract is void because the cause is not stated
c. Oral agreement of lease of a house within six months d. The contract is valid so long as it is in writing
d. Oral contracts of agency giving authority to an agent to sell the land
belonging to the principal 16. A kind of contract which is defective because it contains all the essential
requisites prescribed by law, but it causes injury or damage to the
7. Contract entered into during a state of a hypnotic spell is contracting parties or to a third person.
a. Voidable a. Void contract
b. Legal b. Voidable contract
c. Void c. Unenforceable contract
d. Valid d. Rescissible contract

8. Mark, against his good sense and judgment entered into a contract for the 17. Which of the following is void?
delivery of 500 sacks of rice to Lester for a price of P500, 000. The contract a. Those whose object is outside the commerce of men
is b. Those which are entered into to defraud creditors
a. Void c. Those which are contrary to the statute of fraud
b. Voidable d. Those where one of the parties is incapacitated to give his consent
c. Valid
d. Unenforceable 18. Contract which cannot be sued upon unless there has been a ratification
a. Unenforceable contract
9. A intimidated B to marry A's daughter After a year, B would like to file an b. Voidable contract
action for annulment but could not do so because A was around to intimidate c. Void contract
him The marriage contract is d. Rescissible contract
a. Rescissible
b. Void 19. Contract that is valid unless annulled
c. Voidable a. Inexistent
d. Unenforceable b. Unenforceable
c. Rescissible
10. Which of the following is not a characteristic of a void or inexistent d. Voidable
contract?
a. The action or defense for declaration of the nullity or inexistence of the 20. The action to annul a voidable contract is extinguished by
contract does not prescribe a. Novation
b. They are not subject to ratification b. Ratification
c. The right to raise the defense of illegality cannot be waived c. Estoppel
d. The defense of illegality of the contract is available to the third person whose d. Rescission
interest is not directly affected
21. A sold in writing to B his stereo set for Php600.00. There is no delivery from
A and no payment of the price from B. Contract is
a. Void
b. Voidable
c. Enforceable
d. Unenforceable

25

0 0
c. Voidable
d. Void

22. S sold his parcel of land for only Php 1M although the value of the same is
Php 2M. He thus suffered damage or lesion in the sale due to the inadequacy
of the price. Therefore, the contract is
a. Rescissible 32. A defective contract because it is entered into in the name of another without
b. Unenforceable or in excess of authority, or it is verbal is
c. Valid, not defective a. Voidable
d. Voidable b. Void
c. Unenforceable
23. Because A wants to sell his land to B but the latter does not want to buy the d. Rescissible
same, A forced B to buy his land. The contract is
a. Not binding upon B since his consent was vitiated 33. A sold his land to B verbally. The sale is
b. It is valid, binding and enforceable a. Voidable
c. It is unenforceable as against B but not as against A b. Unenforceable
d. May be ratified expressly or tacitly by A c. Rescissible
d. Void
24. A promised to give B a car as reward after B killed C. Later, after the killing,
the contract was changed to a lease of a big house for a certain period. The 34. A, with oral authority from P, sold the latter's house in writing The sale is
contract of lease is a. Valid
a. Void b. Voidable
b. Unenforceable c. Void
c. voidable d. Rescissible
d. Rescissible e. Unenforceable

25. While his father was still alive, A sold to the property he (A) expected to 35. S sold to B his land valued at P1M only for the price of P0.7M thus resulting
receive from his father. Is the contract defective? to inadequate price the sale is:
a. It is void for future inheritance cannot be sold a. Rescissible
b. It is voidable sale if he fails to receive the property he expected to receive b. Valid
from his father c. Voidable
c. It is valid for there can be sale of future things and what A sold is future d. Unenforceable
property
d. It is completely a valid contract because the seller is compulsory heir 36. In order that fraud may make a contract voidable
a. It should be serious and should have not have been employed by both
26. S orally leased to R his parcel of land for a term of two years The contract contracting parties
is b. It may be serious and the parties must be in pari delicto
a. Rescissible c. It may be incidental but both parties should not be in pari delicto
b. Void d. It may be incidental but should have been employed by both parties
c. Unenforceable
d. Voidable 37. On December 1, 2022, M entered into a contract with A. On September 1,
2024, M discovered the fraud committed by A at the time the contract was
27. A verbal agreement was made between A and B whereby A agreed to sell perfected. In this case an annulment may be filed:
and B agreed to buy A's farm for P100, 000 00. The price was unpaid. a. No more because the right has already prescribed
Possession was not given nor was the deed delivered, both being refused. b. Within four years from December 1, 2022
The contract is c. Until December 1, 2024
a. unenforceable d. Until September 1, 2028
b. Void
c. Rescissible 38. If the illegal contract between the parties is a criminal offense and both
d. Voidable parties are guity (in pari delicto), such illegal contract shall produce the
following effects, except:
28. A was forced by B to sign a contract, C a creditor of A wants to annul the a. The parties shall have no right of action against each other
contract. Is C allowed by law to do so? b. The effects and instruments of the crime will be confiscated in favor of the
a. No because a third person cannot assail a void contract government
b. Yes, a third person can annul a rescissible contract c. Both parties shall be criminally prosecuted
c. No, because a third person cannot assail a voidable contract d. The parties may recover what they have given if it is not illegal in itself
d. Yes, because the contract is voidable and C is damaged
39. The Statute of Frauds apples only to
29. If the obligation of the debtor is “I will pay you my debt after I have arrived a. Contracts wholly or partially executed on the part of the debtor
from abroad”, the obligation is b. Wholly executory contracts
a. Unenforceable c. Wholly executed contracts
b. Void d. Contracts wholly or partially executed on the part of the creditor
c. Valid
d. Voidable 40. The following are characteristics of rescissible contracts, except
a. The action to rescind them prescribes
30. I. A contract entered into by a minor is void b. The action to rescind them are not available to third persons even if the
II. A contract where the amount involved exceeds P500.00 must be in interests are directly affected
writing to be valid c. They may be set aside for equitable reasons
a. True, true d. They are valid until rescinded
b. True false
c. False, false 41. It takes place when the parties do not intend to be bound at all by their
d. False, true agreement.
a. Absolutely simulated contract
31. A defective contract because it is prohibited by law is b. Deed of assignment
a. Rescissible c. Apparent contract
b. Unenforceable d. Relatively simulated contract

26

0 0
c. No, unless the mortgagor is a co-debtor.
d. Yes, provided it is in writing and registered.

42. No. 1: When one of the parties brought an action to enforce the instrument,
he cannot subsequently ask for its reformation.
No. 2: The injured party may seek rescission, even after he has chosen
fulfillment if the latter should become impossible.
a. True, true
b. False, true
c. True, false
d. False, false

43. A defective contract where damage or lesion is essential is:


a. Unenforceable
b. Void
c. Rescissible
d. Voidable

44. Anako, insane, sold his land to Bentong 21 years old for P200,000. Anako
spent P120,000 in buying another piece of land, P15,000 for clothing and
medicines, P25,000 for food and P40,000 he lost in gambling. After
annulment of the contract, how much must Anako return to Bentong?
a. None
b. P120,000
c. P160,000
d. P200,000

45. A and B who are both unemancipated minors entered into a contract. The
contract entered into by and between them is:
a. Void
b. Voidable
c. Unenforceable
d. Rescissible

46. This contract is without effect unless ratified:


a. Donation between husband and wife.
b. Marriage between first degree cousins.
c. Contract of sale between two insane persons.
d. Contract of sale between husband and wife.

47. Which of the following contracts is not void ab initio?


a. That which is undertaken in fraud of creditors.
b. Those whose object is outside the commerce of man.
c. That whose object did not exist at the time of transaction.
d. That which contemplates an impossible service.

48. Which phrase best completes the statement- The affidavit of good faith in a
Deed of Chattel Mortgage is:
a. A certification form from the mortgagor that he is the mortgagor of the
chattel.
b. An affidavit, the absence of which will vitiate the mortgage between the
parties.
c. Necessary only if the chattels being mortgaged are growing crops.
d. An oath where the parties swear that the mortgage is made for the purpose
of securing the obligations specified and that the obligation is just and valid.

49. A borrowed P1M from B. The loan was secured by a mortgage of A’s land.
Without the knowledge of A, C paid B the sum of P1M for A’s debt. As a
result:
a. The obligation of A to B was extinguished but A should reimburse C the
amount of P1M.
b. C may foreclose the mortgage on A’s land if A cannot pay.
c. C cannot claim reimbursement from A in as much as the payment was made
without the knowledge of A.
d. C can recover the amount from B in case A refuses the reimbursement to C.

50. D owes C a sum of money with M as mortgagor of his land to secure the
loan. Is the mortgage valid even if the mortgagor is not the debtor?
a. No, the mortgagor/ pledgor must be the debtor himself.
b. Yes, provided the mortgagor or pledgor in case of pledge is the absolute
owner of the property mortgaged or pledged.

27

0 0
1. Three of the following statements pertain to natural obligation. Which one
does not?
a. Obligation exists by reason of equity and moral justice
b. If performed voluntarily, recovery can no longer be made.
c. There is juridical necessity to perform it. 9. I. Obligations with a resolutory period take effect at once, but terminate
d. It is not enforceable in a court of justice. upon arrival of the day certain.
II. Obligations with a resolutory period take effect at once, but terminate
2. Example No. 1: G, guardian of W, sold W’s house valued at P50,000 for upon happening of the condition
P37,500 or a lesion by one – fourth of the value. a. true, true
Example No. 2: S sold his house valued at P50,000 for only P10,000 because b. true, false
S did not know the true value of the house c. false, true
a. Only No. 1 is rescissible. d. false, false
b. Only No. 1 is rescissible.
c. Both contracts are rescissible. 10. Colossus Corporation published an advertisement in the Manila Bulletin
d. Both contracts are valid and enforceable. which reads as follows “INVITATION TO BID:” Construction of a 3 storey
building in Lucao District, Dagupan City.” The advertisement includes the
3. A, B and C are solidary debtors of X, Y and Z, solidary creditors, in the specifications of the building to be constructed. The following companies
amount of P2, 700.00. X renounces the whole obligation without the consent submitted heir bids: Cris Construction Corporation with a bid price of P10.2
of Y and Z. The debtors accepted the renunciation. million; Norhan Company, P10.4 M; and Frank Builders Corporation, P10.5
a. Only P300.00 is extinguished. M. Colossus Corp. considered also the financial capability, reputation and
b. The whole obligation is extinguished. experience of the bidders, the kind and quality of materials to be used and
c. Only P900.00 is extinguished. other factors
d. No part of the obligation is extinguished because not all the creditors a. Colossus Corporation is required to declare Cris Corporation as the winner
consented to the renunciation. having declared the lowest bid.
b. An award given to either Norhan or Frank Builders is voidable because they
4. S and B orally agree that S would sell and B would buy S’s radio for P400, are not the lowest bidder.
two years from the date of the agreement. At the end of the two – year c. Colossus Corporation cannot refuse to accept a winner even if none of the
period, S refused to deliver the radio although B was willing to pay. bidders is qualified because the offer is definite.
a. No Statute of Frauds because the price is less than P500. d. An award to Frank Builders is valid because Colossus Corporation is not
b. The contract falls under the Statue of Frauds, therefore unenforceable. bound to accept the lowest bidder.
c. The object is movable, oral contract is enforceable.
d. B can compel S to deliver because B is willing to pay the price. 11. Avedon sold Bato’s car to Caridad in the name of Bato without the authority
of the latter. The sale is:
5. Statements No. 1: Obligation for whose fulfillment a day certain has been a. Unenforceable
fixed, shall be demandable only when that day comes. Obligation with a b. Void
resolutory period take effect at once, but terminate upon arrival of the day c. Voidable
certain. d. Rescissible
Statement No. 2: D binds himself to pay “little by little”. The obligation is
for the benefit of the debtor. 12. The following are primary modes of extinguishing obligations. Which is the
a. No. 1 is false, no. 2 is true exception?
b. Both are true a. Confusion
c. Both are false b. Condonation
d. No. 1 is true, No. 2 is false c. Compensation
d. Compromise
6. Thru some mistake on the part of a bank personnel, A was given One Million
Pesos instead of One Thousand pesos. This situation will be governed by 13. The following, except one, are the characteristics of void or inexistent
the provisions on”: contract. Which is the exception?
a. Contracts a. The defense of illegality of the contract is available to third persons whose
b. Law interests are not directly affected.
c. Quasi-delicts b. They are not subject to ratification.
d. Quasi-contracts c. The right to raise defense of illegality cannot be waived.
d. The action or defense for declaration of their nullity or inexistence of the
7. S, a store owner, hired the services of E as a clerk in the store for P250.00 contract does not prescribe.
per day. E began to work immediately although no employment papers were
signed by S and E. After 1 year and 2 months without E receiving any 14. There shall be no reformation of contract in the following cases, except:
compensation except free meals and lodging. E demanded payment from S. a. A contract of sale of a condominium unit wherein the seller offered for sale
S refused to pay Eon the ground that their contract was unenforceable not Suit 17-J of Tower 2 while the buyer bought Suite 17-G of Tower 2.
being in writing and that the employment already lasted more than one year. b. In testamentary wills.
a. The contract is rescissible because E suffered damages by reason of non- c. Simple donation inter vivos when no condition is imposed.
payment of his compensation. d. A contract of sale agreed upon by the parties but the document signed was
b. The contract is valid and enforceable although not in writing. a contract of mortgage.
c. The contract is unenforceable because it is not in writing.
d. The contract is void because an employment contract must be in writing to 15. S owns an oil painting. Being in need of money, S sold the painting to B for
be valid. P1,000. After the sale it was discovered that the painting was valuable and
worth P5,000.
8. An insurance policy which, in practice, is prepared by an insurance company a. S may rescind the contract on ground of lesion or inadequacy of cause.
and all the insured has to do is to sign thereon if he agrees with its terms, is b. S may rescind the contract on ground of fraud.
an example of: c. B is entitled to the benefit of the contract because it is valid and binding.
a. A gratuitous contract d. S may annul the contract on the ground error.
b. A commutative contract
c. An auto-contract 16. On July 15, 2018, X entered into a contract with Y. On February 10, 2019,
d. A contract of adhesion X discovered that fraud was committed at the time he entered into the
contract, a fraud that vitiated his consent. The action for annulment shall be
brought:
a. Within four years from X entered into the contract.
b. Within three years from the time of the fraud.

28

0 0
c. Within four years from February 10, 2019. d. None, because the assignment made by C was without the knowledge of D.
d. On February 10, 2019.

17. D, fearing that his creditor C, would go after his only parcel of land to satisfy
his claim for payment of D’s debt, sold his said land to X who did not know
of D’s intention. Decide. 26. Salas and Bersamin entered into a contract whereby Salas would deliver 5
a. C can file an action for rescission of the sale because the sale was undertaken pieces of genuine Rolex wristwatches to Bersamin. Salas proposed to
to defraud him. Bersamin that should Salas deliver 5 units of fake pieces of Rolex
b. C may ask for damages against X since he was damaged by the sale. wristwatches by reason of financial difficulties on his part, Bersamin would
c. C cannot ask for the rescission of the sale. not sue him for damages on the ground of fraud. Bersamin accepted the
d. C can ask for annulment of the sale as this is voidable contract. proposal. On due date, Salas delivered 5 pieces of fake Rolex wristwatches.
Upon discovery of the fraud, Bersamin sues Salas for damages. Salas
18. D owes C P10,000 with G as guarantor. Third person X, pay the obligation contends that he cannot be held liable for damages because Bersamin
without the consent of the debtor. X asks for reimbursement from D but D waived his right to hold him liable on the ground of fraud if the reason
is insolvent. thereof is the financial difficulties of Salas.
a. X can compel C to return the payment. a. Bersamin can sue Salas for the annulment of the contract since it is voidable.
b. X is by right subrogated to the rights of C. b. Bersamin cannot sue Salas for damages because the reason for the waiver is
c. The obligation of D to C is not extinguished. valid, i.e., financial difficulties of Salas, which is a valid reason for the
d. X cannot compel C to subrogate him in his rights. waiver.
c. Bersamin can sue Salas for damages because the waiver he made is void.
19. Which statement is true? Nonetheless, the contract is valid.
a. Physical force will vitiate consent. d. Bersamin can sue Sales for the rescission of the contract since it is
b. Mistake as to accounting will never vitiate consent. rescissible.
c. Incidental fraud will vitiate consent provided it is employed by only one
party. 27. In order that mistake may vitiate consent:
d. Mistake as to identity of the party will never vitiate consent. a. Mistake must refer to the principal conditions of the contract.
b. Letter A and letter B are false.
20. The following contracts are required to appear in a public document for the c. Letter A and letter B are true.
convenience of the parties so that they may be registered into the proper d. Mistake must refer to the substance of the thing, object of the contract.
recording office, except:
a. The cession of actions or rights proceeding from an act appearing in a public 28. Tadeo owned a vacant lot adjacent to a bigger lot where Star Circus planned
document. to put up a two-week performance as provided in the lease contract, which
b. Contracts which have for their object the creation of real rights over it executed, with Legaspi, the owner of the bigger lot. In expectation of using
immovable property. his property for a parking lot, Tadeo leveled his lot and built a small shed,
c. The power to administer property. which cost him P20, 000.00, an amount he had yet to pay the contractor.
d. The acceptance of an inheritance. Later, however, the circus cancelled the performance, so Tadeo brought a
court action to recover P20, 000.00 from the circus and Legaspi. Based on
21. In an obligation worded as follows: “I promise to continue allowing X to the foregoing information, which of the following is incorrect?
stay in my condo unit until he marries,” the obligation is subject to: a. Tadeo can recover damages from Star Circus and Legaspi by reason of
a. suspensive condition stipulation pour autrui.
b. suspensive period b. The circus contract took effect only between Star Circus and Legaspi, the
c. resolutory condition parties to the contract of lease.
c. Tadeo, as an incidental beneficiary, has no rights under the circus contract.
22. The obligation remains valid up to a time certain but terminates upon the d. Tadeo must pay his own contractor the amount of P20, 000.00 because that
arrival of said period. is the obligation he alone contracted to pay.
a. conditional
b. with a period 29. This obligation is demandable at once
c. ex die a. When it has a resolutory condition
d. in diem b. When it is with a term ex die
c. When it has a suspensive condition
23. In delicts and quasi-delicts, the defendant shall be liable for: d. When it has a period
a. Damages intended but excluding exemplary damages.
b. Damages which may be renounced in advanced. 30. D owes C P10,000. But the debt soon prescribes. Later X, against the
c. Not only for results intended but also for their foreseen consequences. consent of D, pays C P10,000.
d. Damages which are natural and probable consequences of the act or a. X has no right.
omission complained of whether or not such damages have been foreseen b. X cannot recover from D what he pays C.
or could have reasonably been foreseen by the offender. c. X can recover from D P10,000 because the latter was enriched at the
expense of X.
24. This is an obligation with a resolutory condition d. Both D and C are liable to X.
a. I’ll give you P10,000 if you pass the October 2019 CPA board Examination
b. I’ll give you P10,000 on December 31, 2019 31. Perfecto offered to sell his house and lot to Reynaldo for P1, 500, 000.00.
c. I’ll give you my car now, but should you fail in any of your subjects, your Perfecto told Reynaldo that he was giving Reynaldo thirty (30) days to
ownership will cease and it will be mine again decide whether to buy or not the house and lot. Reynaldo accepted the option
d. I’ll give you P10,000 if A dies of TB but did not give anything to Perfecto to support the option given to him.
Twenty days later, Perfecto found another buyer who was ready to buy the
25. D owes C P20, 000.00 due on March1, P3, 000.00 due on March 8, P5, house and lot for P2, 000,000.00. Perfecto wants to ask you whether he can
000.00 due on March 14, and P2, 000.00 due on March 24. On March 18, C still withdraw the offer he made to Reynaldo. Decide.
assigned his credit right to T without informing D who learned of the a. No, Perfecto may not withdraw the offer because the option was accepted
assignment on March 20. On March 25, T may collect from D: by Reynaldo although Reynaldo did not give anything in support thereof.
a. P20, 000.00 b. Yes, Perfecto may withdraw the offer because he stands to gain an added
b. P2, 000.00 profit of P500, 000.00 and this will be more than enough to pay any damages
c. P4, 000.00 to Reynaldo.

29

0 0
c. No, Perfecto may not withdraw his offer until the lapse of the period given 39. X, Y and Z solidarily bound themselves to pay to solidary creditors A, B
to Reynaldo to exercise his option. and C the amount of P75,000.00. The loan was secured by a mortgage on
d. Yes, Perfecto may withdraw the offer by just informing Reynaldo of such B’s land. Out of gratuity, A, in a public instrument, renounced the obligation
fact. in favor of X, with the formality required by law. In this case, which
statement is correct?
32. The obligation begins only from a day certain or upon the arrival of the a. Both the principal obligation and the mortgage are extinguished by
period remission.
a. ex die b. Both the principal and the accessory obligations subsist.
b. conditional c. The principal obligation is extinguished.
c. in diem d. The real estate mortgage is extinguished.
d. with a period
40. A1, A2 and A3 oblige themselves solidarily to give C a specific car valued
P12,000. On the due date, C demanded delivery but the debtors failed to
deliver. The next day, while A1 still in possession of the car, it got lost due
to fortuitous event. The right of C is
a. Proceed against any of the debtors for the value and damages.
b. Proceed against A2 or A3 but only P4,000 each.
c. Proceed against A1 only, because he is the one is possession at the time it
33. On May 31, 2018, D promised to give a specific house and lot to C if C was lost.
passes the Bar Examination. On September 24, 2018, C took the Bar d. None, obligation is totally extinguished.
Examination. The result of the Bar Examination, which C took, was released
on April 14, 2019 and C passed it. C shall be entitled to the house and lot:
a. on May 31, 2018
b. on April 14, 2019
c. when he receives his rating
d. on September 24, 2018
41. A promised to give B his Volvo car if B passes the CPA board exams.
34. Consignation alone, as a special form of payment, may extinguish an Pending the results of the exams, the car is destroyed by a fortuitous event
obligation under any of the following instances. Which is the exception? without any fault on the part of A. As a result,
a. When the creditor is absent, or is unknown or does not appear in the place a. The obligation of A is extinguished
of payment. b. The obligation of A will be equitably reduced
b. When there are two or more persons claiming title to one and the same c. A will have to give B another car of equivalent value
obligation. d. The obligation of A is converted into monetary obligation
c. When the creditor refuses to issue receipts.
d. When the creditor is capacitated to receive payment. 42. An obligation where each of the debtors is liable only for a proportionate
part of debt, and each creditor is entitled only to a proportionate part of the
35. “I will give you this book provided that if I like to have it back, you will credit is called:
return the same to me”. a. Divisible obligation
a. C. The obligation is valid because the condition merely causes the loss of b. Solidary obligation
rights already acquired. c. Joint obligation
b. The obligation is void, because the fulfillment depends upon the will of the d. Conjoint obligation
debtor.
c. The obligation is void, because the fulfillment depends upon the will of the 43. The following contracts are void or inexistent, except:
creditor. a. Those whose cause or object did not exist at the time of the transaction.
d. Combination of A and B. b. Those whose cause, object or purpose is contrary to law, morals, good
customs, public order or public policy.
36. On October 4, 2018, A borrowed from B P50,000, to be paid 20 days c. Those, which are absolutely simulated or fictitious.
thereafter. A proposed to B that X will pay A’s debt, and that A will be free d. Those where one of the parties employed fraud to obtain the consent of the
from all liabilities. B and X agreed to the proposal. On October 25, 2018, other.
when B tries to collect from X, he finds that X is insolvent. At the time of
delegation, X was already insolvent but this was not known to A. The 44. An insane entered into a contract with a minor.
insolvency is not of public knowledge. So B sues A on the ground that it a. The contract is voidable
was A who made the proposal and that A really guaranteed X’s solvency. b. The contract is void.
Decide. c. The contract is rescissible.
a. A is liable because he is presumed to have guaranteed X’s solvency. d. The contract is unenforceable.
b. A is not liable, because A does not know the insolvency of X at the time of
delegation and neither was the insolvency of public knowledge. 45. Which of the following contracts is rescissible?
c. A is not liable, because A does not know the insolvency of X at the time of a. Contracts entered into defraud creditors when the latter cannot collect the
delegation and neither was the insolvency of public knowledge. claims due them.
d. A is liable because X agreed to the proposal to make himself solidary liable b. Contracts entered into during a hypnotic spell.
for the obligation. c. Contracts entered into in a state of drunkenness.
d. Contracts where both parties are incapable of giving consent.
37. The following are real contracts. Which is not?
a. Contract of deposit 46. Items 47, 48, and 49 are based on the following information:
b. Contract of loan A and B are indebted to X and Y for 10, 000.00. A and B share in the debt
c. Contract of guaranty in the ration of 1:3; while X and Y share in the credit in the ratio of 2:3.
d. Contract of donation How much may X collect from A if there is active solidarity?
a. P2, 500.00
38. The act of abandoning all his properties in favor of his creditors so that the b. P4, 000.00
latter may cause their sale and apply the proceeds thereof to their claims c. P10, 000.00
proportionately is called: d. P1, 000.00
a. payment by cession
b. set off 47. A, B and C are liable in solidum to X for P12, 000.00. X renounced the share
c. tender of payment with consignation of A who accepted it. Later, B becomes insolvent.
d. dacion en pago a. X can collect nothing from C.
b. X can collect from C P4, 000.00
c. X can collect from C P8, 000.00.

30

0 0
d. X can collect from C P12, 000.00 55. One of the distinctions between a facultative obligation and an alternative
obligation is that in an alternative obligation:
48. Guardian sells a property of Ward valued P100,000 to X for P75,000. a. The right of choice belongs to the debtor alone.
a. The contract is unenforceable. b. Only on thing is principally due.
b. The contract is rescissible. c. If the obligation to hive a principal thing is void, the obligation to give the
c. The contract is valid substitute is also void.
d. The contract is voidable. d. If all prestations except one are impossible, that which is possible must still
be given.
49. Rescission of contract can take place in this case:
a. When the seller cannot return the installment paid to him by the buyer. 56. In order that fraud may make a contract voidable:
b. When he who demands rescission can return whatever he may obliged to a. It may be incidental but both parties should not be in “pari delicto”
restore. b. It may be serious and the parties must be in “pari delicto”
c. When the party seeking resolution can perform only as to part and rescind c. It may be incidental but should have been employed by both parties.
as to remainder. d. It should be serious and should have not have been employed by both
d. When the things which are the object of the contract are legally in the contracting parties.
possession of third persons who acted in good faith.
57. Anything paid or delivered before the arrival of the period, the obligor being
50. Choose the contract which is voidable: unaware of the period or believing that the obligation has become due and
a. Those whose object is outside the commerce of man. demandable
b. Those where consent of one contracting party was vitiated by violence or a. may be recovered with fruits and interests
mistake. b. may not be recovered
c. Those where both contracting parties cannot give consent. c. may be recovered with interests
d. Those undertaken in fraud of creditors where the latter cannot in any manner d. may be recovered without interests
cannot collect his credit.
58. The following except one are characteristics of a contract:
a. liberty
b. obligatoriness
c. relativity
d. consensual

51. A was badly in need of money. He offered to sell his parcel of land to B for
P100,000.00. B agreed and paid A the P100,000.00 and A signed a receipt.
When B wanted to register the sale, he needed a Deed of Absolute Sale.
What can B do?
a. B may sue A to return the P100,000.00 under the legal maxim “No one shall
enrich himself at the expense of another”.
b. B may posses and utilize A’s land as a buyer in good faith.
c. B may compel A to execute the Deed of Absolute Sale because the contract 59. In contracts and quasi – contracts, the liability of the debtor who acted in
is valid. good faith:
d. B cannot get back the P100,000.00 because the contract is not enforceable. a. Extends to all damages which may be renounced in advanced.
b. Extends only to results intended but excluding exemplary damages.
52. A father promised to give his son a car if the son will marry B this year. If c. Extends to all natural and probable consequences of the breach of the
by the end of the year, B is already dead or the son has not married B, the obligation, and which the parties have foreseen at the time the obligation
obligation to give a car is effective and demandable. was constituted.
A father promised to give his daughter a car if the daughter will not marry d. Extends to all damages which may be reasonably attributable to the non
her boyfriend earlier than December 31, 2019. If by December 31, 2019, the performance of the obligation.
daughter has not yet married her boyfriend, or if prior thereto, her boyfriend
had died, the obligation is extinguished. 60. Items 47, 48, and 49 are based on the following information:
a. true, true A and B are indebted to X and Y for 10, 000.00. A and B share in the debt
b. true, false in the ration of 1:3; while X and Y share in the credit in the ratio of 2:3.
c. false, true How much may X collect from A if the debtors are joint debtors, while the
d. false, true creditors are joint creditors?
a. P10, 000.00
53. Reformation of an instrument is available in the following cases, except: b. P2, 500.00
a. When one of the parties has brought an action to enforce the contract. c. P1, 000.00
b. When a party was mistaken and the other knew or believed that the d. P4, 000.00
instrument did not state their real agreement.
c. When a mutual mistake of the parties causes the failure of the instrument to 61. If the illegal contract between the parties is a criminal offense but only one
disclose their agreement. party is guilty, such illegal contract shall produce the following effects,
d. When one party was mistaken and the other party acted fraudulently or except:
inequitably in such a way that the instrument does not show their true a. The instruments shall be confiscated in favor of the government.
intention. b. The innocent party cannot recover what he has given.
c. The guilty party will be criminally prosecuted.
54. Six years ago, Marcelo, 17 years old, sold his one-hectare agricultural lot to d. Neither party may compel the other to comply with his undertaking.
Betanio for P100, 000.00. Marcelo invested the proceeds of the sale in a
business, which was successful at the start but had gone through difficult 62. An incidental element of a contract.
times as a result of the economic crisis. He thought of the agricultural lot a. Implied warranty.
that he had sold to Betanio, which had been enjoying bumper crops since b. Payment of interest in a loan.
Betanio bought it from him. Marcelo now approaches you to seek your help c. Delivery of the object in a contract of pledge.
on whether it would still be possible to annul the sale of the agricultural lot d. All of the above.
to Betanio. What advice will you give to Marcelo?
a. Both Marcelo and Betanio may annul the sale. 63. The passage of time extinguishes the obligation:
b. Betanio may annul the sale but not Marcelo a. The passage of time extinguishes the obligation
c. Marcelo may annul the sale but not Betanio. b. Fulfillment of resolutory condition
d. Neither Marcelo nor Betanio may annul the sale. c. Arrival of resolutory period
d. Remission
e. Rescission

31

0 0
71. An obligation wherein various things are due but the complete performance
64. W, a woman, agreed to live with M’s wife without the benefit of marriage of all of them is necessary to extinguish the obligation.
in exchange for M’s promise to give a monthly support of P10, 000.00. Both a. Conjoint obligation
parties are single. After 3 years, M found another woman and stopped giving b. Pure or simple obligation
support to W. c. Facultative obligation
a. M can demand the return of the support that he had given to W for the past d. Alternative obligation
3 years.
b. The agreement between M and W is illegal but the illegality does not 72. Items 47, 48, and 49 are based on the following information:
constitute crime. A and B are indebted to X and Y for 10, 000.00. A and B share in the debt
c. W can demand the continuation of the monthly support from M pursuant to in the ration of 1:3; while X and Y share in the credit in the ratio of 2:3.
their agreement. How much may X collect from A if there is passive solidarity?
d. Both M and W will criminally prosecuted. a. P2, 500.00
b. P10, 000.00
65. The following are obligations with a term or period, except: c. P4, 000.00
a. D to give C P 50,000.00 if C’s father dies within 2 years. d. P1, 000.00
b. D to give C P 50,000.00 upon death of C’s father dies within 2 years.
c. D to give C P 50,000.00 on Christmas day next year. 73. A owned a house rented by B. A sold the house to C where C agreed to pay
d. D to give C P50, 000.00 on December 1, 2018. the balance of the purchase price as soon as B leaves the premises. It was
further agreed that C will take care of seeing to it that B vacates the house.
66. Leveriza, the owner of an apartment, leased the premises to Toribio. The Which is correct?
terms of the lease provide for a monthly rental of P6, 000.00 which shall be a. The contract is valid if B is willing to vacate the premises
paid by Toribio at the residence of Leviriza about two blocks from the b. The contract is valid because the condition is mixed
apartment. For the month of December, Toribio went to the residence of c. The contract is void because it is potestative on the part of C
Leveriza but the latter was not around to receive the payment. Not wanting d. The contract is void because the consent of B was not obtained
to go back, Toribio left the payment with Nograles, a neighbor of Leveriza,
who promised to give the payment to Leveriza. However, Nograles spent 74. A obliges himself to pay X P100,000 in 30 days plus a penalty of P20,000
the amount he received for himself. Based on the foregoing facts, which of if A fails to pay the obligation in due time. A failed to pay the obligation in
the following statements is incorrect? 30 days. X can demand from A.
a. Toribio can go after Nograles for the amount misspent by Nograles. a. The principal of P100,000 plus P20,000 penalty.
b. The payment by Toribio is not valid because Nograles was not authorized b. The principal of P100,000 plus legal interest.
to receive the payment. c. The principal of P100,000 plus P20,000 penalty, plus legal interest.
c. Leveriza can demand payment from Toribio for the December rental. d. The principal of P100,000 plus P20,000 penalty, plus legal interest, plus
d. The payment by Toribio to Nograles is valid since the latter is a neighbor of damages.
Leveriza.
75. Prime Engineering Review Center (PERC) stated in the leaflets it distributed
last January that any reviewee who places first in the licensure examinations
for engineers this year will receive a cash prize of P150,000.00
a. The obligation of PERC is a pure obligation.
b. The obligation of PERC is subject to a resolutory condition.
c. The obligation of PERC is an obligation with a suspensive period.
d. The obligation of PERC is subject to a suspensive condition.

67. Which of the following contracts is not rescissible?


a. Those executed in representation of an absentee, if the latter suffer a lesion
of more than ¼ of the value of the object of the contract.
b. Those where one of the parties is incapable of giving consent to a contract. 76. X Company bought out a competitor, Y, with a stipulation that Y should
c. Those undertaken in fraud of creditors when the latter cannot in any other thereafter not engage in any business in the Philippines without the consent
manner collect the claims due them. of X Company.
d. Those which are entered into by guardians whenever the wards whom they a. The stipulation is not valid because this is against public policy.
represent suffer a lesion of more than ¼ of the value of the object of the b. The stipulation is valid, only it will create a monopoly in trade.
contract. c. The Stipulation is not valid because the parties as a rule have no absolute
e. Those which refer to things under litigation if they have been entered into freedom to stipulate terms and conditions in a contract.
by the defendant without the k nowledge and approval of the litigants or of d. The stipulation if ratified is valid.
competent judicial authority.
77. I. The condition that some event happen at a determinate time shall
68. The following, except one, are secondary modes of extinguishing extinguish the obligation as soon as the time expires or it has become
obligations. Which is that exception? indubitable that the event will not take place.
a. compromise II. The condition that some event will not happen at a determinable time
b. changing the object of the obligation with the consent of the parties shall render the obligation effective from the moment the time indicated has
c. death of both parties elapsed, or it has become evident that the event cannot occur.
d. prescription a. true, false
b. false, false
69. Meeting in one person of the characteristics of both debtor and creditor in c. false, true
one and the same obligation extinguishes the obligation by way of: d. true, true
a. Novation
b. Condonation or remission 78. Which of the following contracts is valid and enforceable?
c. Compensation or set – off a. An oral contract for the sale of a piece of land for P50, 000.00 with the buyer
d. Merger of confusion giving a down payment of P20, 000.00
b. An oral contract where the fruits of an immovable belonging to the debtor
70. Not a ground for damages: are to be applied to the interest and principal of his obligation.
a. bad faith c. A written contract for the purchase of a newborn baby who is the illegitimate
b. culpa child of the buyer.
c. mora d. A written contract of employment where the employee agreed voluntarily
d. dolo causante not to demand overtime payment for work rendered in excess of the regular
hours of work per day.

32

0 0
b. D can ask for return of the book because it should be considered to have
79. In a contract, as written, D promises to pay C P10,000 on September 15, been lent only by L to S, i.e., the contract should be one of commodatum
2018. The consideration received by D is not stated in the contract. only.
a. The contract is void because the cause is not stated. c. D cannot ask for the return of the book although she is the only heir because
b. The contract is valid so long as it in writing. L’s right over the book was not transmitted to her.
c. The contract is valid because cause is not essential to a contract. d. The contract involving the book is void since the intent of L and S was
d. The contract is valid because the cause is always presumed to exist. ambiguous. Accordingly, D can ask for the return of the book on the ground
of the nullity of the contract.
80. A who owes B P100,000, entered into a fictitious sale of his car with C, so
that B cannot attach it in case A cannot pay his debt. 87. D stole the carabao of C. D was arrested, tried in court and convicted. Aside
a. The contract between A and C is absolutely simulated. from being sentenced to a prison term, D was also ordered by the court to
b. The contract between A and C is relatively simulated. return the carabao. However, the carabao died before D could deliver it to
c. The contract between A and C is rescissible. C.
d. The contract between A and C is void as against B. a. D is not liable to C if the cause of the death is a fortuitous event.
b. D is liable to C whatever may be the cause of death of the carabao.
81. The receipt of a later installment of a debt without reservation as to prior c. D has no liability to C whatever may be the cause of the death of the carabao
installments, shall give rise to a rebuttable presumption that such because his obligation to return the carabao was due to an order of the court
installments have been paid. and not on his own volition.
If a taxpayer pays his income tax liability for the current year, there is a d. D is liable to C if the carabao died because of D’s fault.
presumption that income tax liability for the previous year has been paid.
a. true, true 88. The following is considered fraud or fraudulent:
b. true, false a. The usual exaggeration in trade where the other party had opportunity to
c. true, false know the facts.
d. false, false b. Misrepresentation by a third person.
c. Failure to disclose facts when there is a duty to reveal them.
82. S sold his only horse to B for P30, 000.00. The parties agreed that S shall d. Misrepresentation made not in bad faith.
deliver the horse one week from the execution of their agreement. B,
however, is required to pay the price immediately in certified check. In the 89. If the condition is potestative on the part of the debtor, the obligation is void.
place of S and B, it was the custom that anyone selling a horse should place If the condition is potestative on the part of the creditor, the obligation is valid.
a horseshoe on its hooves. However, this was not stated in the agreement a. false, true
including how the horse would be cared for before delivery. b. true, false
a. S is obliged to place a horseshoe on the horse because part of the contract. c. true, true
b. S is not obliged to place a horseshoe on the horse although it was the custom d. false, false
in the place because S and B did not stipulate about it
c. S and B are bound only by the terms stated in their agreement. 90. Which of the following is a secondary classification of obligations under the
d. S is not obliged to take care of the horse delivery because the caring of the Civil Code?
horse was not stipulated. a. Determinate and generic obligations.
b. Pure and conditional obligations.
83. These are factors vitiating consent. Which is not? c. Alternative and facultative obligations.
a. Intimidation d. Obligations with a period or obligations with a term.
b. Due influence
c. Fraud 91. The following cannot give consent to a contract. Which is the exception?
d. Violence a. Un-emancipated minors
b. Deaf-mute who does not now how to write.
c. Insane person during his lucid interval.
d. Person under civil interdiction.

84. W, a woman, agreed to live with H, a man, as the wife of H without the
benefit of marriage in exchange for the monthly support of P10, 000.00 that
H would give to W. Both H and W are single, of legal age, and there is no
legal impediment for them to get married. Based on the foregoing
information, which of the following statements is true? 92. Stanley, 17 years old, suffered a deep cut on his eyebrow during a basketball
a. W may legally demand that H give her the monthly support of P10, 000.00 game. He requested one of his teammates to call Dr. Ramos whose clinic
that he had promised. was just a stone’s throw from the basketball court. Later, Dr. Ramos sent a
b. The agreement between H and W is valid because they can legally get bill to Stanley amounting to P500.00 for the services he rendered. Stanley
married if they want to. refused to pay bill, claiming that, as a minor, he has not liable. The charge
c. The agreement between H and W is void for being contrary to morals. was a reasonable amount.
d. H may legally demand that W live with him as his wife. a. Stanley is not liable because he is a minor and thus incapable of giving
consent to engage the services of the doctor.
85. Which of the following is not a ground for the extinguishment of an b. Stanley is liable only when he reaches the age of majority.
obligation? c. Stanley is liable only when he reaches the age of majority.
a. Merger d. Stanley is liable although he is a minor because the contract involved a
b. Remission necessary.
c. Death of creditor
d. Compensation 93. Which is the least defective contract?
a. Mutuality of contract
86. D, the only daughter and heir of L, a lawyer, discovered shortly after L’s b. Void contract
death that her father’s book, Legal Eagle, a rare collection, was missing from c. Rescissible contract
L’s library. After going through L’s records, D traced the book to S, a law d. Voidable contract
student. When D informed S that she wanted the book returned, S refused
claiming that L donated the book to him. D, however, protested saying that 94. When the debtor binds himself to pay when his means permit him to do so,
L donated the book to him. D, however, protested saying that the book was the obligation shall be deemed to be
merely lent to him by L. a. With a resolutory condition
a. D cannot ask for the return of the book because the same should be b. With a period
considered as having been donated by L to S. c. With a potestative condition

33

0 0
d. With a suspensive condition Representation as the credit of a third person

95. Contract with a false cause is Lease of real property longer than one year
a. void
b. voidable Guaranty
c. gratuity
d. unenforceable Lease of personal property longer than one year

96. Statement No. 1. A stipulation in a contract of lease, which provides that the
tenant shall have the right to continue in possession of the leased premises
so long as he pays the monthly rentals thereon, is a valid stipulation. 3. A and b executed a contract on January 12, 2022 where A agreed to sell and
Statement No. 2. The seller sold to a minor some necessaries in the amount B agreed to buy A’s only fighting cock. The agreement provides that the
of P600. The goods were delivered to the minor who, in turn paid the delivery of the cock is to be made on June 24, 2022 at which time B would pay
purchase price therefore. The contract is unenforceable. the agreed price of P2,000. On June 9, 2022, A sold the same fighting cock to
a. No. 1 is true; No. 2 is false C, B asks for your advice, assuming there is no delivery yet of the cock to C.
b. Both are true
c. No. 1 is false; No. 2 is true First Advice: B may sue for damages on June 9, 2022 without the need of
d. Both are true demanding delivery because it is useless, the object was sold to a third person
acting in good faith
97. It was X a third party who employed violence upon A, wherefore, by reason
thereof, A was compelled to sell his car to B. Second Advice: B may sue for damages after the arrival of the stipulated period,
a. Either A or B can annul the contract. that is June 24, 2022, because the right of B to sue will only accrue on June 24,
b. The contract is valid because X is not a party to the contract. 2022
c. The contract is void.
d. The contract is voidable. First is wrong, second is correct

98. A – 1 and A -2 are solidary debtors of B – 1, B – 2 and B – 3, joint creditors Both advices are correct
in the amount of P90,000. How much can B -3 collect from A – 2?
a. B – 3 could collect P90,000 from A – 2 and give P30,000, each to B – 1 and Both advices are wrong
B – 2.
b. B – 3 could collect P15,000 from A -2 and P13,000 from A – 1. First is correct, second is wrong
c. B – 3 could collect P45,000 from A -2 and give P13,000, each to B – 1 and
B – 2.
d. B – 3 could collect P30,000 from A – 2.
4. Simulation of a contract maybe absolute or relative. It is relative when
99. An absolutely simulated contract is:
a. Rescissible The parties are related within the 4th degree of consanguinity
b. Voidable
c. Void The contract is void
d. Unenforceable
The parties conceal their true agreement
100.Statement 1 – An offer becomes ineffective upon the death, civil
interdiction, insolvency of the offeror before acceptance is conveyed to him. The parties do not intend to be bounded at all
Statement 2 – An offer becomes ineffective upon the death, civil
interdiction, insolvency or insanity of the offeree before acceptance is
conveyed to him.
a. Both statements are false 5. D is obliged to give Object 1, Object 2, or Object 3 to C. The obligation
b. Statement 1 is false but statement 2 is true referred is:
c. Statement 1 is true but statement 2 is false
d. Both statements are true a conjunctive obligation

a simple obligation

an alternative obligation

a facultative obligation

MIDTERM EXAM IN REG 1


6. P took a public bus in going to his office. Although P paid his fare, the bus
did not issue him a ticket. Along the way, the bus met an accident causing a
1. A and B agreed on a certain contract, but A fraudulently made a document slight injury to P and other passengers. If P is to recover damages from the bus
reciting another kind of contract. Later both A and B died. Which is correct? owner, the source of the bus owner’s liability is

Neither the son of A nor the son of B may bring an action to reform the Contract
instrument
P cannot recover any amount because no ticket was issued
Only the son of A may bring an action to reform the instrument
Law
Either the son of A or the son of B may bring an action to reform the instrument
Quasi-delict
Only the son of B may bring an action to reform the instrument

7. D, out of his love and affection for C, donated a parcel of land to the latter
2. The following contracts should observe the Statute of Frauds, except: who accepted it. The formalities required by law were complied with. The cause
of the contract

34

0 0
The generosity of D The obligation is valid because the impossible condition is negative.

The contract has no cause because D did not give any consideration to D. The obligation is unenforceable because the condition is immoral.

The parcel of land The condition is void because of the positive impossible condition.

The acceptance of the donation by C

13. "I will give you this book provided that if I like to have it back, you will
return the same to me”.
8. I. When what is to be delivered is a determinate thing, the creditor may compel
the debtor to make the delivery and if the debtor refuses, the creditor may ask Combination of A and B.
that the obligation be complied with at the expense of the debtor.
The obligation is void, because the fulfillment depends upon the will of the
II. The obligation to give a determinate thing includes that off delivering all its creditor.
accessions and accessories, even though they may not have been mentioned
The obligation is void, because the fulfillment depends upon the will of the
true, false debtor.

false, false The obligation is valid, because the condition merely causes the loss of rights
already acquired.
false, true

true, true
14. I. Responsibility arising from fraud is demandable in all obligations. Any
waiver of an action for fraud is void.

9. Not a requisite of stipulation pour autri: II. If the law or contract does not state the diligence which is to be observed in
the performance of an obligation, that which is expected of a father of good
The stipulation in favor of a third person must be part of the contract, and not
family shall be required.
the contract itself.
true, true
The favorable stipulation should not be conditioned or compensated by any
kind of obligation. true, false
Neither of the contracting parties bears the legal representation or authorization false, true
of the third party.
false, false
The third person is incidentally benefitted by the stipulation.

15. D owes C P6,000. No date for payment was stipulated by the parties. Which
10. The assessment of the amount of damages is left to the discretion of the is correct?
courts, according to the circumstances of each case, except
C can require D to pay at anytime
Moral
D is not liable to C because the obligation is void there being no date of payment
Liquidated
C can require D to pay when the period arrives
Exemplary
D is not required to pay unless C goes to court and asks the court to require D
Temperate to pay

11. Angel dela Gurdia, guardian of Edurado Menor, a minor, sold the palay 16. The following statements are presented to you:
harvested from the land belonging to Eduardo Menor for P42, 000.00. The palay
had a value of P50, 000.00. At the time of the sale seven months ago, Eduardo I. The illegality of the motive of a party to a contract renders the contract void.
was 17 ½ years old. Eduardo wants to recover the damages he suffered under the
contract entered into by his guardian. What is the remedy available to Eduardo II. The motive of one party to a contract is always known by the other party.
Menor?
In your evaluation of the foregoing statements:
Rescission because Eduardo Menor suffered lesion of P8, 000.00
Only II is true
Annulment because Eduardo menor was only 17 ½ years old at the time the
Both are false
sale was made by Angel dela Guardia.
Both are true
Both rescission and annulment.
Only I is true
Neither rescission nor annulment.

17. This is a type of defective contract because it is entered into in the name of
12. X obliged himself to give Y his brand new Rolex watch if the latter cannot
another without or in excess of authority, or it is verbal:
make a circle that is at the same time a square.
Void
The obligation is void, not enforceable.

35

0 0
Voidable The true intention of the contracting parties is not expressed in the instrument
purporting to embody the agreement by reason of mistake, fraud, inequitable
Rescissible conduct or accident.

Unenforceable

23. Fraud in the perfection of pre-existing obligation

18. Which of the following is not correct in rescission? Dolo causante

The person who seeks rescission must be able to return what he ought to return Dolo dicente

Can be demanded by any of the contracting parties Dolo incidente


The object of the contract is not in the possession of third persons who have Dolo cesante
acquired in good faith

Cannot be availed when the party who has been damaged has other legal
remedy 24. S sold his lot consisting of 1,000 square meters to b. However, the deed of
sale signed by the parties showed a total area of 1,200 square meters due to the
secretary’s mistake. Later, S discovered the mistake. What remedy is available
to either party?
19. A, B and C owe W and Y P6,000. A, B and C share in the debt at 2:3:5 while
W and Y share in the credit at 1:2. If there is active solidarity (creditors), how Rescission
much can Y collect from B?
Reformation
P2,000
Annulment
P6,000
Declaration of nullity of the contract
P4,000

P1,800
25. On June 1, 2024, Decker orally guaranteed the payment of a P5,000 note
Decker’s cousing owed Baker. Decker’s agreement with Baker provided that
Decker’s guaranty would terminate in eighteen months. On June 3, 2024, Baker
20. (ART 1169) A is obliged to deliver his only car to B on November 20, 2018.
wrote Decker confirming Decker’s guaranty. Decker did not object to the
If A does not deliver on the said date, and on November 22, 2018, a typhoon
confirmation. On August 23, 204, Decker’s cousin defaulted on the note and
destroys the car. Is A still liable? Baker demanded that Decker honor the guaranty. Decker refused. Which of the
following statements is correct?
No, The obligation is extinguished, even if the debtor is already default because
the debtor can plead impossibility of performance.
Deck er is liable under the oral contract of guaranty because Bak er demanded
payment within one year of the date the guaranty was given
No, Considering that no demand to deliver was made by B and the specific
thing was lost due to fortuitous event, the obligation is extinguished.
Deck er is not liable under the oral guaranty because it expired more than one
year after June 1
Yes. The creditor can instead demand for a substitute of equivalent value from
the debtor. Decker is not liable under the oral guaranty because Decker’s promise was not
in writing
Yes. A is already in legal delay, thus the obligation to deliver the lost specific
thing is converted into monetary claim for damages.
Deck er is liable under the oral guaranty because Decker did not object to
Baker’s June 3 letter

21. In three of the following defective contracts, ratification cleanses the defects.
Which is the exception? 26. This kind of compensation which arises by way of proved counterclaim in a
case is called:
Sale of immovable property or interest orally entered into.
Judicial compensation
Both parties are incapable of giving consent.
Voluntary compensation
Contracts entered into by a person who has been given no authority.
Legal compensation
Sale of piece of land thru an agent the authority is oral.
Facultative compensation

22. Reformation is not the proper remedy if


27. The following instances will render an offer ineffective before acceptance is
The mutual mistake of the parties causes the failure of the instrument to disclose
conveyed. Which one will not?
their real agreement
All of the above
There was mistake, fraud, inequitable conduct or accident which prevented the
meeting of the mind of the contracting parties
Insanity of either party
One party was mistaken and the other acted fraudulently or inequitably in such
Insolvency of either party
a way that the instrument does not show their true intention

36

0 0
Civil interdiction or either party Either Y or Z can demand P90,000 from W

A’s obligation is extinguished up to P30,000

28. S, a supplier of fresh fish from Lucena City hired T, the owner of a trucking Both obligations are extinguished by compensation
company, for a fee of P3,000 to bring the fish of S to the Dampa Market in
Paranaque City which ordered the fish for a price of P20,000. In so far as S is Either Y or Z can demand P60,000 from A
concerned, his prestation in his contract with T is

The transport of the fish


33. Heavy rain from severe tropical storm Maring, with international name
The payment of P20,000 Kompasu, triggered floods and landslides in the Philippines, according to the
national disaster management council. During the flood, the properties of D were
The sale of the fish saved from the destruction by C without the knowledge of D who was then away.
C incurred necessary and useful expenses in the act of saving D’s properties. For
The payment of P3,000 such expenses:

D must reimburse C because there was an implied contract between D and C.

29. A, a duly authorized agent of P, wrote a letter to B on March 1, 2022 offering D is not required to reimburse C because C acted without D’s consent.
to sell P’s only Mercedes Benz car for P200, 000.00 cash. On March 3, 2018, B
wrote a letter to A stating that he was accepting all the terms of the offer, which D must reimburse C although C acted without the consent of D.
letter was received by A on March 5, 2022, before A could relay such acceptance
to P, P died in a vehicular accident on March 6, 2022. D need not reimburse C because C’s act of saving D’s properties is a gratuitous
act that requires no compensation.
The contract was perfected on March 1, 2022.

The contract was perfected on March 5, 2022.


34. Analyn advertised in the newspaper her parcel of land wanting to sell the
The contract was perfected on March 3, 2022. same for P1M. Banny personally went to the former with cash in hand to buy
the subject parcel of land. In this case:
The contract was not perfected because P, the real party to the sale, died before
the acceptance came to his knowledge. Analyn cannot evade her obligation as seller to Banny

Analyn can still reject Banny as offered in the purchase of land

30. In novation, which of the following is incorrect? Analyn can reject the offer of Banny unless she properly consigns with the court
her payment for the land
In expromission, the insolvency of the new debtor shall not revive the action of
the creditor against the original debtor. Analyn cannot anymore reject Banny as buyer of her land

If the original obligation is subject to a suspensive condition, the new obligation


is not subject to the same condition unless stated.
35. Statement 1: The action for annulment on the ground of fraud shall be filed
While of the condition is resolutory, the new is also resolutory condition, unless within 4 years from the time of the perfection of the contract.
otherwise stated.
Statement 2: If a valid contract is novated to a void contract, both contracts are
If the new obligation is void, original obligation shall subsist. considered void.

No.1 is true; No.2 is false

31. Sometime in 2012, Tomas donated a parcel of land to his stepdaughter Irene, No.1 is false; No.2 is true
subject to the condition that she may not sell, transfer or cede the same for twenty
years. Shortly thereafter, he died. In 2022, because she needs money for medical Both are true
expenses, Nene sold the land to Conrado. The following year, Irene died, leaving
as her sole heir a son by the name of Armando. When Armando learned that the Both are false
land which he expected to inherit had been sold by Irene to Conrado, he filed an
action against the latter for annulment of the sale, on the ground that it violated
the restriction imposed by Tomas. Conrado filed a motion to dismiss, on the 36. Which of the following must be in writing to be enforceable as required by
ground that Armando did not have the legal capacity to sue. Is Conrado correct? the Statute of Frauds?
Choose the best answer.
A contract whereby one agrees to pay another’s debt if the latter defaults in his
Yes, only one of them is an aggrieved party on account of their own violation payment.
of the condition of, or restriction on, their ownership imposed by the donation.
A contract for the lease of an agricultural lot for a period of 8 months.
Yes
A subscription for 100 shares of stock of a corporation at P100.00 per share.
No
A contract for the construction of a building scheduled to begin 3 months after
No, only an aggrieved party to the contract may bring the action for annulment the execution of the contract.
thereof

37. A donation of a gold ring worth P5,000 made by D to C. The donation of the
32. A is indebted to solidary creditors W, Y and Z for P90,000 due on August 1, gold ring by D and its acceptance by C were made orally with D simultaneously
2021, while W owes A P90,000 also due on August 1, 2021. On August 1, 2 021, delivering the ring to C.

37

0 0
State the status of the above contract from among the following choices: of the telephone company. The latter maintained that Baldomero remained as
his customer as far as their service contract was concerned, notwithstanding the
Rescissible lease between Baldomero and Jose. Who is correct, Baldomero or the telephone
company? Choose the best answer.
Voidable
Telephone company, because Baldomero cannot substitute Jose in his stead
Unenforceable
without the consent of the telephone company
Valid and enforceable
Baldomero
Void
Telephone company

Baldomero, since it was already stipulated in the contract that Jose will be the
one to pay the telephone services
38. On June 1, 2022, S and B entered into a loan worth P501. The agreement
was merely an oral contract. The parties agreed that S shall deliver the money to
B on June 10, 2022 at the place of B. On June 10, 2022 the money was not
delivered and S refused to do. The contract is? 43. A owes B P2,000. A, knowing that the debt has already prescribed,
nevertheless still pays B. Which is correct?
The contract between S and B is valid.
A can recover the P2,000 without interest
The contract between S and B is unenforceable.
A cannot recover anything
The contract between S and B is void.
A may invoke solution indebiti
The contract is voidable.
A can recover the 2,000 plus interest

39. Reformation of the instrument is the proper remedy where


44. One of the following is not a requisite of the object of a contract.
The true intention of the parties cannot be ascertained
It must be determinate as to its kind or capable of being made determinate
The failure of the instrument to express the true intention of the parties is due
without the need of the parties entering into a new agreement.
to mistake, fraud, inequitable conduct or accident
It must not be contrary to law, morals, good customs, public order or public
There is no meeting of minds of the parties to the contract policy.
Consent of the parties was secured because of mistake, fraud, accident or
It must be within the commerce of men.
inequitable conduct
It is a right, it must be transmissible.

40. One of the following is an obligation with a period with a suspensive effect.
Which is it? 45. Merle offered to sell her automobile to Violy for P60,000. After inspecting
the automobile, Violy offered to buy it for P50,000. This offer was accepted by
“I will give you P5,000 per month as support beginning January 1 of next year.” Merle. The next day, Merle offered to deliver the automobile, but Violy being
short of funds, secured postponement of the delivery, promising to pay the price
“I will give you P5,000 per month as support until you finish your law course.”
“upon arrival of the steamer, Helena”. The steamer however never arrived
“I will give you P5,000 per month as support until you are able to find work.” because it was wrecked by a typhoon and sank somewhere off the coast of
Samar.
“I will give you P5,000 per month for your medicines if your physical
examination says you are sick.” Question 1: is there a perfected contract in this case?

Question 2: is the promise to pay made by Violy conditional or with a term?

41. Statement 1: An oral sale of land made by its owner is unenforceable Question 3: can Merle compel Violy to pay the purchase price and to accept the
automobile?
Statement 2: Sale of land made by an agent without authority from the owner
thereof is void No, with a term, no

False, tru e Yes, with a term, yes

True, true No, conditional, no

False, false Yes, conditional, yes

True, false

46. Contracts that cannot be sued upon unless ratified:

42. Baldomero leased his house with a telephone to Jose. The lease contract Unenforceable
provided that Jose shall pay for all electricity, water and telephone services in
Consensual
the leased premises during the period of the lease. Six months later, Jose
surreptitiously vacated the premises. He left behind unpaid telephone bills for
Rescissible
overseas telephone calls amounting to over P20,000. Baldomero refused to pay
the said bills on the ground that Jose had already substituted him as the customer

38

0 0
Voidable 52. D is obliged to give Object 1, Object 2, or Object 3 to C. Assume the
following:

I. Object 1 and Object 2 are lost through D’s fault, and later Object 3 is lost
47. There is novation of the obligation if through fortuitous event.

1. The period for payments is shortened from 5 years to3 years II. Object 1 and Object 2 are lost through fortuitous event, and later Object 3 is
lost through D’s fault.
2. The period for payment is extended from 3 years to 5 years
In your evaluation of the foregoing statements, D’s obligation is extinguished in:
True, true
I only
True, false
Both I and II
False, false
II only
False, true
Neither I nor II

48. A entered into a contract with B whereby B agreed to testify in a criminal


case filed against A in exchange for P5, 000.00 to be given by A. The contract 53. A leased a property to B part of the premises was subleased by B to C. B
between A and B is violated the conditions of the lease, so A wanted to rescind the lease contract.
Which is correct?
Void for being contrary to public policy.
A can rescind the lease contract provided he respects the sublease
Is valid because there is nothing wrong with it.
A cannot rescind the lease contract as the sublease would naturally be affected
Void for being contrary to law
A may rescind the lease contract and is not bound to respect the sublease
Void for being contrary to good customs.
B may object to the cancellation of the contract of the contract because he is
not a party to the violation
49. Geely Auto Philippines engaged the services of Wilan Construction
Corporation for the construction of its showroom. For this purpose, Wilan
bought construction materials on a 60-day credit from Narra Commercial 54. Which of the following contracts is valid and enforceable?
Corporation. On due date, Narra demanded payment from Wilan but the latter
contended that it could not may any payment because it has not yet collected A contract where a party gave his consent because the other party threatened to
from Geely. In this case: sue him for an unpaid debt.

Narra can hold liable both Wilan and Geely. A contract where a party gave his consent in a moment of drunkenness.

Narra cannot hold liable both Wilan and Geely. A contract where there was inadequate cause attended by mistake.

Narra can hold liable Wilan only. A contract where a party gave his consent because the other party threatened to
kill the first party’s spouse.
Narra can hold liable Geely only.

55. As a rule, a contract of sale is perfected:


50. The following contracts are void except:
Upon demand.
Contracts which are absolutely simulated.
Upon compliance with the requirements of the law as to form.
Contracts where parties acted in pari delicto.
Upon the meeting of the minds of the thing which is the object of the contract
Contracts where both parties in incapacitated to give consent. and upon the price.

Contracts whose objects as illegal; Upon delivery of the object of the contract.

51. Which of the following contracts is void, except? 56. S sold 100 bottles of imported “Fundador” brandy to B who paid
immediately the price thereof amounting to P20, 000.00. S promised to deliver
All of the foregoing. the brandy to B within one week from their agreement. On the agreed date of
Those where the intention of the parties relative to the principal object of the delivery, S delivered to B 100 bottles of fake “Fundador brandy”. The contract
between S and B is:
contract cannot be ascertained.
Rescissible
Those which object is outside the commerce of men.
Valid
Those expressly prohibited or declared void by law.
Voidable
Those which contemplate a possible service.
Void

39

0 0
57. A, B and C owe W and Y P6,000. A, B and C share in the debt at 2:3:5 while
W and Y share in the credit at 1:2. How much can W collect from A?
63. D forced C to lend him P10,000. The promissory note is in writing.
P1,000
Contract remains to be valid.
P1,200
The contract is void.
P400
C cannot demand payment from D because the contract is unenforceable .
P667
The contract is rescissble because the contract is fraudulent.

58. Where demand by the creditor shall be necessary in order that delay may
exist 64. The following obligations are demandable at once, except:

When time is of the essence of the contract An obligation with a condition antecedent.

When demand would be useless An obligation with an in diem period.

When the obligor requested for an extension of time An obligation with a condition not to do an impossible thing.

When the obligor has expressly acknowledged that is in default An obligation without any term or condition.

59. A wrong committed independent of contract and without criminal intent 65. A and B entered into a verbal contract whereby A agreed to sell to B his only
parcel of land for P20,000, and B agreed to buy at the aforementioned price. B
Delict went to the bank, withdrew the necessary amount, and returned to A for the
consummation of the contract. A however, had changed his mind and refused to
Culpa contractual go through with the sale

Quasi-contract Statement 1: the agreement is valid

Correct! Statement 2: an action by B against A for specific performance will prosper

Culpa aquiliana True, true

False, false
60. I. A defendant who is acquitted in a criminal case is no longer liable civilly. False, true

II. A quasi-contract is an implied contract True, false

false, true

true, false 66. Consent is manifested by the meeting of the offer and the acceptance upon
the thing and the cause which are to constitute the contract. Which of the
true, true
following constitute the contract. Which of the following constitutes a definite
offer?
false, false
All the foregoing.

Business advertisement of things for sale.


61. A, B and C owe W and Y P6,000. A, B and C share in the debt at 2:3:5 while
W and Y share in the credit at 1:2. If there is passive solidarity (debtors), how
An offer made through an agent.
much can W collect from C?
Advertisement for bidders.
P1,000

P3,000
67. In order to defraud C, his creditor, D by means of force compelled B to buy
P2,000 his (D’s) land. B has no knowledge of such fact.
P6,000
As to B, the contract is voidable.

As to C, the contract is rescissible.


62. Elements that accompany certain contracts unless set aside or suppressed by
Both of a and b
the parties are known as
None of a and b
Essential elements

Natural elements
68. A borrowed P10,000 from B. On due date, A was not able to pay but she
Moral elements
promised to give B a specific ring, a specific bracelet, or a specific necklace, in
payment of the debt. B accepted the offer of A. What kind of obligation is the
Accidental elements
new obligation of A?

40

0 0
Facultative obligation 74. A lost in gambling and as payment executed a promissory note in favor of
the winner, B. Later, B assigned the note to C. May C successfully recover from
Compound obligation A?

Simple obligation Yes, because B can recover , so also the assignee can

Alternative obligation No, because B cannot recover, so also cannot the assignee

Yes, if C is not aware that it is in payment of a gambling debt

69. The waiver of an action for fraud is valid in the case of: No, because promissory notes can only be negotiated but not assigned

in neither I nor II

I. Future fraud 75. Thru some errors on the part of a bank personnel, A was mistakenly given
P10,000 instead of only P1,000. This situation will be governed by the provision
Both I and II
of the law on
II. Past fraud
Quasi-contract

Contracts
70. O is the owner of a car which was sold by S without authorization in favor
Quasi-delicts
of B. The contract between S and B in so far as O is concerned is:
Delicts
Valid
Delicts
Voidable

Void
76. X was the owner of a P10,000 square meter property. X married Y and out
Unenforceable their union, A, B, and C were born. After the death of Y, X married Z and they
begot as children D, E, and F. After the death of X, the children of the first and
Rescissible
second marriages executed an extrajudicial partition of the afforested property
on May 1, 1970. D, E, and F were given a one thousand square meter portion of
the property. They were minors at the time of the execution of the document. D
71. X, who was abroad, phoned his brother Y, authorizing him to sell X’s parcel was 17 years old, E was 14, and F was 12; and they were made to believe by A,
of land in Pasay. X sent the title to Y by courier service. Acting for his brother, B, and C that unless they sign the document they will not get any share. Z was
Y executed a notarized deed of absolute sale of the land to Z after receiving not present then. In January 1975, D, E, and F filed an action in court to nullify
payment. What is the status of the sale? the suit alleging they discovered the fraud only in 1973. Which of the following
statements is true?
Void, since X should have authorized agent Y in writing to sell land
Since fraud can be the basis of nullity, Minority is not anymore a basis because
Valid, since the buyer could file an action to compel X to execute a deed of sale four years had passed since the constitution of the contract

Valid, since Y was truly his brother X’s agent and entrusted with the title A, B, and C may ratify the defect of the contract since they are the one who
needed to effect the sale executed

Valid, since a notarized deed of absolute sale covered the transaction and full In the case of fraud, when through insidious words or machinations of one party
payment was made the other is induced to enter into the contract without which he would have still
agreed to, the action still prosper

Minority can be a basis to nullify the partition because D, E and F were not
72. D is indebted to C in the sum of P10, 000. For the purpose of avoiding the properly represented by their parents or guardians at the time they contracted the
claims of C, D donated all his properties worth the same amount to X.
extra-judicial partition
The contract is merely voidable.

The contract is void 77. One of the following obligations is void. Which is it?
The contract is rescissible. D to give C P50, 000.00 if C does not run 100 miles without stopping.
The contract is absolutely simulated. D to give C P50, 000.00 when D has the means.

D to give C P50, 000.00 if D buys a brand new car.


73. DO UT FACIAS: D to give C P50, 000.00 if C runs for barangay chairman next year.
I do that you may give

I give that you may do


78. Roland, a basketball star, was under contract for one year to play-for play
exclusively for Lady Love, Inc. however, even before the basketball season
I do that you may do
could open, he was offered a more attractive pay plus fringes benefits by Sweet
I give that you may give Taste, Inc. Roland accepted the offer and transferred to Sweet Taste. Lady Love
sues Roland and Sweet Taste for breach of contract. Defendants claim that the
restriction to play for Lady Love alone is void, hence, unenforceable, as it

41

0 0
constitutes an undue interference with the right of Roland any payment at all. 81. In December 2022, Salvador and the Star Semiconductor Company (SSC)
Printado has also a standing contract to enter into contracts and the impairment executed a Deed of Conditional Sale wherein the former agreed to sell his 2,000
of his freedom to play and enjoy basketball. Can Roland be bound by the square meter lot in Cainta, Rizal, to the latter for the price of P1,000,000 payable
contract he entered into with Lady Love or can he disregard the same? Is he P100,000 down, and the balance 60 days after the squatters in the property have
liable at all? How about Sweet Taste? Is it liable to Lady Love? been removed. If the squatters are not removed within six months, the P100,000
down payment shall be returned by the vendor to the vendee, Salvador filed
Yes,Roland is liable under the contract as far as Lady Love is concerned. He is ejectment suits against the squatters, but in spite of the decisions in his favor, the
liable for damages under Article 1170 of the Civil Code since he contravened squatters still would not leave. In August, 2023, Salvador offered to return the
the tenor of his obligation. Not being a contracting party, Sweet Taste is not P100,000 down payment to the vendee, on the ground that he is unable to remove
bound by the contract but it can be held liable under Art. 1314. The basis of its the squatters on the property. SSC refused to accept the money and demanded
liability is not prescribed by contract but is founded on quasi-delict, assuming that Salvador execute a deed of absolute sale of the property in its favor, at which
that Sweet Taste knew of the contract. Article 1314 of the Civil Code provides time it will pay the balance of the price. Incidentally, the value of the land had
that any third person who induces another to violate his contract shall be liable doubled by the time. Salvador consigned the P100,0000 in court, and filed an
for damages to the other contracting party. action for rescission of the deed of conditional sale, plus damages. Will the
action prosper?
No, but Roland is liable under the contract as far as Lady Love is concerned.
He is liable for damages under Article 1170 of the Civil Code since he No
contravened the tenor of his obligation. Not being a contracting party, Sweet
Taste is not bound by the contract but it can be held liable under Art. 1314. The Yes, the action will prosper. The action for rescission may be brought only by
basis of its liability is not prescribed by contract but is founded on quasi-delict, the aggrieved party to the contract. Since it was Salvador who failed to comply
assuming that Sweet Taste knew of the contract. Article 1314 of the Civil Code with his conditional obligation, he is not the aggrieved party who may file the
provides that any third person who induces another to violate his contract shall action for rescission but the SSC. The company, however, is not opting to
be liable for damages to the other contracting party. rescind the contract but has chosen to waive Salvador’s compliance with the
condition
No, but Roland is liable under the contract as far as Lady Love is concerned.
He is liable for damages under Article 1170 of the Civil Code since he Yes
contravened the tenor of his obligation. Not being a contracting party, Sweet
Taste is not bound by the contract but it can be held liable under Art. 1314. The No, the action will not prosper. The action for rescission may be brought only
basis of its liability is not prescribed by contract but is founded on quasi-delict, by the aggrieved party to the contract. Since it was Salvador who failed to
assuming that Sweet Taste do not knew the contract. Article 1314 of the Civil comply with his conditional obligation, he is not the aggrieved party who may
Code provides that any third person who induces another to violate his contract file the action for rescission but the SSC. The company’s silence, however,
shall be liable for damages to the other contracting party. means that it will, in no time, rescind the sale

Yes,Roland is liable under the contract as far as Lady Love is concerned. He is


liable for damages under Article 1170 of the Civil Code since he contravened
82. Which of the following is not a special mode of payment?
the tenor of his obligation. Not being a contracting party, Sweet Taste is not
bound by the contract but it can be held liable under Art. 1314. The basis of its
Dacion en pago
liability is not prescribed by contract but is founded on quasi-delict, assuming
that Sweet Taste knew of the contract. Article 1314 of the Civil Code provides Payment by cession
that any third person who induces another to violate his contract shall be liable
for damages to the other contracting party. G Cash payment

Tender of payment

79. The mere liberality of a benefactor is the cause in:

83. These statements concerning legal compensation are presented to you:

I. Legal compensation cannot take place if the debts have different due date s.

II. Legal compensation may take place although the debts are payable at different
places.
All of the above
Only statement II is true
Onerous contract
Only statement I is true
Gratuitous contract
Both statements are false
Remuneratory contract
Both statements are true

80. D’s wife was about to deliver a child. Her parents brought her to the hospital.
Who should pay the expenses for medical attendance ? 84. Which of the following contracts involving real property is valid and
enforceable although not in writing?
the wife’s parents
Sale of piece of land for P50, 000.00
the wife
Donation and acceptance of a residential lot.
the husband
Mortgage of a commercial lot to secure a loan of P50, 000.00
½ wife’s parents, ½ husband
Lease of an agricultural lot for a period of 2 years at a monthly rental of P3,
000.00

42

0 0
Yes, Edward may withdraw the offer because he stands to gain an added profit
of P500, 000.00 and this will be more than enough to pay any damages to Leon.
85. A contract of sale is not a

Consensual contract
90. A contract of sale is, except:
Real contract
Consensual contract
Nominate contract
Nominal contract
Principal contract
Real contract

Principal contract
86. S and B agreed on the sale of a certain amount of opium for the total price of
P1, 000,000. Which of these statements is correct?

All of the above 91. On June 1, 2022, Kevin and Louie entered into an oral contract of sale
whereby Kevin sold his computer to Louie for P50, 000.00. The parties agreed
If S had delivered the opium, and B does not pay him, he cannot ask for the that Kevin shall deliver the computer to Louie on June 10, 2022 at the place of
return of the opium. Louie. Louie, however, is required to pay the price upon agreement. Louie
complied with his obligation by paying the price to Kevin in cash on June 1,
If B had paid the P1, 000,000 and S does not deliver the opium, he cannot
2022. As of June 1, 2022,
recover the price paid.
The contract of sale between Kevin and Louie is partly executed.
The opium and the consideration of P1, 000,000 shall be confiscated by the
government. The contract of sale between Kevin and Louie is wholly executory.

The contract of sale between Kevin and Louie is wholly executed.

87. A is obliged to give his bracelet to B on October 30, 2021. However, on The contract of sale should be in writing before it can be determined whether it
August 1, 2021, B borrowed the ring of A with an agreement that B will return is executed or executory.
the ring on September 30, 2021. On September 30, 2021 A demand the return of
the ring but B refused claiming compensation, which of the following is correct?

B is entitled to claim compensation 92. Statement 1: In reformation of instruments, it is necessary that there is
meeting of minds of the parties to the contract; otherwise, annulment of contract
A may oppose the compensation being claimed by B shall be the remedy if fraud, accident, mistake or inequitable conduct prevented
the meetings of minds.
Either A or B can claim compensation
Statement 2: In the interpretation of contract, doubts on the incidental
A is entitled to claim compensation
circumstances of onerous contracts shall be resolved in favor of greatest
reciprocity of interest and if gratuitous, least transmission of rights and interest.

First is false, second is true


88. P, the owner of a business, appointed A as his agent to manage it. P
authorized A to borrow money for the business. A himself is the one who lends
First is true, second is false
the money at the current rate of interest. Thus, in the said contract of loan, A
enters into a contract in his own behalf as lender and represents P as borrower. Both statements are true
The contract of loan in such a case is considered as:
Both are false
an auto-contract

an aleatory contract
93. Andie entered into a contract with Bessy to teach Accounting 1 topics
a remuneratory contract starting January 2, 2022. Andie died of COVID 19 on November 27, 2021. Is
Andelito, the son of Andie obliged to teach Bessy Accounting 1 topics?
a gratuitous contract
No, if Andelito did not inherit the accounting knowledge and teaching acumen
of his father.
89. Edward offered to sell his house and lot to Leon for P1, 500, 000.00. Edward
No, the contract between Andie and Bessy is personal in nature and are not
told Leon that he was giving Leon thirty (30) days to de cide whether to buy or
transmissible.
not the house and lot. Leon accepted the option and give P500 to support the
option given to him. Twenty days later, Edward found another buyer who was Yes, because contracts take effect between the contracting parties, their heirs
ready to buy the house and lot for P2, 000,000.00. Edward wants to ask you and assigns.
whether he can still withdraw the offer he made to Leon. Decide.
Yes, because Andelito inherited the properties as well as obligations of his
Yes, Edward may withdraw the offer by just informing Leon of such fact. father.

No, Edward may not withdraw the offer because the option was accepted by
Leon and gave something in support thereof.
94. Fraud exists in three of the following. Which is the exception?
No, Edward may not withdraw his offer until the lapse of the period given to
Leon to exercise his option. When the misrepresentation refers to the usual exaggerations in trade, and the
other party had an opportunity to know the facts.

43

0 0
When through the insidious words or machinations of one of the contracting is damage or prejudice to one of the parties or to a third person. Its enforcement
parties, the other is induced to enter into a contract, which, without them, he would cause injustice by reason of some external facts.
would not have agreed to.
Void or Inexistence Contract
When there is a failure to disclose facts, when there is a duty to reveal them, as
when the parties are bound by confidential relations. Unenforceable Contract

When there is an expression of an opinion by an expert which turned out to be Rescissible Contract
wrong, and the other party relied upon such expert knowledge.
Voidable Contract

95. D was driving on his way to Manila from the province when he suffered a
100. The pre-nuptial agreement of H and W, husband and wife, provides for
busted tire. Not having any spare tire, he went to a nearby car spare parts store
complete separation of property. Later, H, with violence and intimidation forced
to buy a new tire. However, he did not have sufficient money with him so he
phoned G, his friend, who happen to know S, the store owner. G then instructed W to sell him (H), W’s pieces of jewelry. The sale is
D to give to S the telephone through where G told S, “Don’t worry, if D cannot Void, because husband and wife are not allowed to sell property to each other.
pay, just charge me”. D was thus able to buy a new tire for P6,000 for which S
issued D a sales invoice. Which is correct? Unenforceable, if the value is at least P500

S can enforce G’s promise because there was a writing of some kind, the Rescissible, if W, suffered a lesion of more than ¼ of the value of the property
invoice for the sale of the tire.
Voidable, because the consent of W is vitiated
If D cannot pay, S can proceed against G to make good his promise to pay D’s
debt.

S can enforce G’s promise to answer for D’s debt since the guaranty was
witnessed by D.

G’s promised cannot be enforced against him because he did not execute any
writing for the guaranty he made.

96. Which of these contracts is consensual?

partnership

deposit

commodatum

mutuum

97. Not a requisite of stipulation pour autrui

The stipulation must be part of the contract

The third person communicated his acceptance to the obligor before its
revocation

There must be an agency between either of the parties and the third person

The contracting parties must have clear and delivery conferred a favor upon
third person

98. A conferment of a direct benefit in a contract between two persons in favor


of a third person who must accept such benefit before the same is withdrawn is
known as:

Polocitacion

Counter – offer

Stipulation por autrui

Donation propter nuptias

99. This kind of defective contract refers to that contract which is validly agreed
upon because all the essential element exists, but courts can nullify it when there

44

0 0
e. Indivisible - It creates a lien on the whole or all of the properties mortgaged,
1. A and B jointly borrowed P10,000 from C. In order to secure their respective which lien continues until the obligation is secures has been fully paid.
obligations, A pledged his cellphone while B pledged his laptop. At the
maturity date of the loan, A paid P5,000 of the loan. Which of the following 9. What is the nature of a contract of pledge, of chattel mortgage, of real estate
statements is correct? mortgage or of antichresis?
a. A cannot demand the release of his cellphone because a contract of pledge a. It is indivisible if the principal contract is solidary.
is indivisible. b. It is divisible whether the principal contract is joint or solidary.
b. The contract of pledge on the laptop is extinguished. c. It is indivisible whether the principal contract is joint or solidary.
c. The contract of pledge on the cellphone is extinguished. d. If is divisible if the principal contract is joint.
d. The obligation of B is extinguished.
10. When is the pledgor or mortgagor required to be the owner of the thing
2. If the thing pledged is not sold in the first and second public auctions, which pledged or mortgaged for the validity of contracts of pledge, of real estate
of the following statements is incorrect in case of appropriation by the mortgage or of chattel mortgage?
pledgee of the thing pledged? a. At the time the contract of pledge or mortgage is constituted.
a. The creditor may appropriate the thing pledged validly. b. At the time the principal obligation is constituted.
b. No answer text provided. c. At the time of the failure to pay the principal obligation.
c. The contract of pledge is extinguished if the creditor decided to appropriate d. At the time the thing to be pledged or mortgaged is to be delivered.
the thing pledged.
d. The principal obligation/contract of loan is not extinguished if the creditor 11. DLSU pledged its notes receivable form a parent to a factor on January 1,
decided to appropriate the thing pledged. 2016 with remaining term of 30 days or maturity date of January 31, 2016.
The face value of the notes receivable is P1,000,000 with maturity value of
3. In case the thing pledged is alienated by the debtor-pledgor to third person, P1,200,000. The notes receivable is pledged for a note payable of
what is the effect of the consent of the pledgee to the alienation made by the P1,100,000 inclusive of interest. If the factor-pledgee collects the
pledgor of the thing pledged to third person? P1,200,000 from the parent on January 31, 2016, who shall be entitled for
a. The contract of pledge is already extinguished. the excess of P100,000?
b. The thing pledged remains to be owned by the debtor-pledgor a. None of the above
c. The ownership of the thing pledged is transmitted to the buyer. b. DLSU, the pledgor-debtor
d. The creditor-pledgee loses possession of the thing pledged. c. Factor, the pledgee-creditor
d. Parent
4. ABC Inc. borrowed P2,000,000 from BPI. ABC Inc. is under receivership.
To secure the fulfillment of the loan, ABC mortgaged its administrative 12. Using the same data in preceding number, suppose it is C who dies leaving
building. Which of the following statements is correct? X and Y as heirs. If D pays X P50,000, which of the following statements
a. The contract of mortgage is valid because the mortgagor is the absolute is correct?
owner of the property mortgaged at the time the mortgage is constituted a. The contract of mortgage is extinguished.
b. The contract of mortgage is null and void because the mortgagor has no free b. X cannot cancel the mortgage to the prejudice of Y.
disposal of the thing. c. X may cancel the mortgage to the prejudice of Y.
c. The contract of mortgage is voidable. d. The contract of loan is extinguished.
d. The contract of mortgage is null and void because only natural person may
enter in a contract of mortgage. 13. The following persons may bid at the public auction, except
a. The pledgee if there are other bidders
5. If the proceeds of the sale of the thing pledged in conventional pledge is b. The pledgor or owner
more than the amount of the obligation, which of the following statements c. The pledgee even if he is the only bidder
is true? d. Third persons
a. The creditor-pledgee shall be entitled to the excess unless there is an
agreement to the contrary. 14. The following are the rights of the debtor-pledgor, except
b. The creditor-pledgee shall always be entitled to the excess a. To ask that the thing pledged be judicially or extra-judicially deposited if it
c. The debtor-pledgor shall always be entitled to the excess. is used without authority or for a purposes other than for its preservation.
d. The debtor-pledgor shall be entitled to the excess unless there is an b. To become the owner of the offspring of the animal pledged if there is no
agreement to the contrary. stipulation to the contrary although such offspring but shall be subject to the
pledge.
6. The following may become the object of a contract of pledge, except c. To require that the thing be deposited with a third person if it is in danger of
a. Incorporeal rights which are evidenced by negotiable instruments, bill of being lost or impaired through the negligence or willful act of the pledgee.
lading, shares of stocks, bonds, warehouse receipts and similar documents. d. To demand the return of the thing pledged, upon offering another thing in
b. Real or immovable properties and rights thereon. pledge, provided the latter is of the same kind and quality, if there are
c. Personal property susceptible of possession. reasonable grounds to fear the destruction or impairment of the thing
d. All movable pledged without the fault of the pledgee. This right is without prejudice to
the right of the pledgee to have the thing sold at a public sale.
7. What is the status of contract of real estate mortgage if the property covered e. To demand the return of the thing pledged despite defaulting on the payment
as collateral is a personal property instead of real property? of the secured principal obligation.
a. Third persons who are not directly affected by the real estate mortgage may f. To continue to be the owner of the thing pledged unless it is expropriated.
ask for the declaration of nullity of the real estate mortgage contract. g. To alienate, with the consent of the pledgee, the thing pledged.
b. The contract of real estate mortgage is always void because the subject h. To ask for the return of the thing pledged after he has paid the debt and its
matter of real estate mortgage is immovable property. interest, with expenses in a proper case.
c. The contract of real estate mortgage may be valid as to the contracting
parties on the basis of doctrine of estoppel but the third person directly 15. If the proceeds of the sale of the thing pledged in conventional pledge is less
affected by said void contract may ask for its declaration of nullity. than the amount of the obligation, which of the following statements is true?
d. The contract of real estate mortgage will be valid to both contracting parties a. The creditor-pledgee can recover deficiency only if stipulated.
and even third persons. b. The creditor-pledgee cannot recover the deficiency even if stipulated.
c. The debtor-pledgor shall pay for the deficiency if stipulated.
8. The following are the important characteristics of contract of real estate d. The debtor-pledgor shall always pay for the deficiency.
mortgage, except
a. Inseparable - It subjects the property upon which it is imposed, whoever the 16. The following are the obligations of the debtor-pledgor, except
possessor may be, to the fulfillment of the obligation for whose security it a. To pay damages that the pledgee may suffer by reason of the flaws of the
was constituted. thing pledged, if he was aware of such flaws but he did not advise the
b. Real right - It creates a lien on the property mortgaged. pledgee of the same.
c. Real contract - It is perfected by the delivery of the thing mortgaged. b. To warrant the thing pledged for its quality and merchantability.
d. Accessory - It cannot exist without a principal obligation.

45

0 0
c. To pay for the expenses which are necessary for the preservation of the thing d. No even if the stipulated because any stipulation for recovery of deficiency
pledged. in of pledge is void.
d. To pay the debt and its interest, with expenses, in a property case, when they
are due.

17. D borrowed from C P10,000 secured by a mortgage on D’s two lots (lot 1
and lot 2). D dies leaving E and F as heirs with E inheriting lot 1 and F lot
2. F pays P50,000 of the loan. Which of the following statements is correct? 26. What is the effect of sale at public auction of the thing pledged?
a. F may ask for the extinguishment of the mortgage on lot 2. a. The contract of pledge and principal obligation/contract of loan shall be
b. The contract of loan is extinguished. extinguished if the proceeds of the sale exceed the amount of the principal
c. F cannot ask for the extinguishment of the mortgage on lot 2. obligation, interest and expenses in a proper case.
d. The contract of mortgage is extinguished. b. The contract of pledge and principal obligation/contract of loan shall be
extinguished whether or not the proceeds of the sale are equal to the amount
18. What is the nature of a contract of pledge? of the principal obligation, interest and expenses in a proper case.
a. Formal contract c. The contract of pledge and principal/contract of loan obligation shall be
b. Consensual contract extinguished if the proceeds of the sale are equal to the amount of the
c. Solemn contract principal obligation, interest and expenses in a proper case.
d. Real contract d. It does not extinguish the contract of pledge.

19. The following are the formalities required for the sale of the thing pledged 27. The following are the rights of a third person who pledges his own movable
in case of failure of the debtor to fail the principal obligation, except property to secure debt of another except
a. It must be sold at the first auction. a. To be indemnified by the debtor if he pays the creditor.
b. There must be a notice to the debtor and the owner of the thing pledged, b. To be subrogated to all the rights of the creditor against the debtor if he pays
stating the amount for which the public sale is to be held. the creditor.
c. It must be through a notary public. c. To become principally liable.
d. It must be by public auction. d. To be released from liability in the cases provided by law.

20. D borrowed P1,000,000 from C. G, a third person, mortgaged his land to 28. It is a contract by virtue of which the debtor delivers to the creditor or to a
secure the fulfillment of D’s loan. Is the contract of mortgage valid? third person a movable, or instrument evidencing incorporeal rights for the
a. Yes provided G will deliver the land to C. purpose of securing the fulfillment of a principal obligation with the
b. Yes because third persons who are not parties to the principal obligation understanding that when the obligation is fulfilled, the thing delivered shall
may secure the latter by pledging or mortgaging their own property. be returned with all its fruits and accessions.
c. No because D must be the owner of the mortgaged land. a. Contract of Real estate mortgage
d. No because G is no privy to the contract of loan. b. Contract of Antichresis
c. Contract of Pledge
21. It refers to the remedy available to the mortgage by which he subjects the d. Contract of Chattel mortgage
property mortgaged to the satisfaction of the obligation secured when the
principal obligation is not paid when due or when there is any violation of 29. Which of the following direct modes of extinguishing contract of pledge
any condition, stipulation or warranty by the mortgagor. impliedly extinguish the principal obligation or contract of loan?
a. Compensation I. Return by the pledgee of the thing pledged to the pledgor or owner.
b. Foreclosure II. Renunciation or abandonment in writing by the pledgee of the contract
c. Dation en pago of pledge.
d. Novation III. Sale of the thing pledged in public auction in case of default by debtor
regardless of the amount of the net proceeds of sale.
22. The following are the essential requisites of a contract of real estate IV. Appropriation of the thing pledged by pledgee in case the thing pledged
mortgage for its validity, except is not solid in the first and second public auctions.
a. That the document in which the mortgage appears be recorded in the a. III and IV
Registry of Property. b. I and III
b. That the mortgagor be the absolute owner of the thing pledged. c. I and II
c. That the person constituting the mortgage must have the free disposal of his d. II and IV
property, and in the absence thereof, that he be legally authorized for the
purpose.
d. That it be construed to secure the fulfillment of a principal obligation. 30. D borrowed P10.000 from C and pledged his ring and watch with P4,000
and P6,000 value respectively. After several days. D pays P4,000 to C.
23. In case of legal pledge or pledge by operation of law, if the thing held in Which of the following statements is correct?
pledged by operation of law is sold by the creditor in public auction resulting a. The contract of pledge is extinguished.
to excess, who shall be entitled to the excess? b. D may compel C to return the ring because P4,000 of the loan is already
a. The creditor or foreclosing party even if not stipulated. paid.
b. The debtor/owner of the thing sold even if not stipulated. c. D cannot demand the release of his ring because a contract of pledge is
c. The debtor/owner of the thing sold only if stipulated. indivisible.
d. The creditor or foreclosing party if stipulated. d. The contract of loan is extinguished.

24. The following may become object of contract of real estate mortgage, except 31. The following are the kinds of principal obligations that may be secured by
a. Immovable property a contracts of pledge, of real estate mortgage or of chattel mortgage, except
b. Personal property a. Pure obligations
c. Rights on immovable property b. Conditional obligations whether suspensive condition or resolutory
d. Real property condition
c. Unenforceable obligations
25. In case of legal pledge or pledge by operation of law, if the thing held in d. Correct!
pledged by operation of law is sold by the creditor in public auction resulting e. Null and void obligations
to deficiency, may there be recovery of deficiency by the creditor? f. Voidable obligations
a. Yes but it must be stipulated by the contracting parties. g. Rescissible obligations
b. No because it is pledged by operation of law. h. Natural obligations
c. Yes even if not stipulated. i. Obligations with a period whether suspensive period (ex die) or resolutory
period (in diem)

46

0 0
a. The pledgor-owner may ask for the return of the thing, upon offering another
thing in pledged, provided the latter is of the same kind as the former and
not of inferior quality even it is already sold in public auction by the
pledgee-creditor.
b. The right of the pledgor-owner is superior to the right of the pledgee-
creditor.
c. The pledgee-creditor may cause the sale of the tiling pledged at a public sale
and the proceeds of the auction shall be a security for the principal obligation
in the same manner as the thing originally pledged. However, the pledgee is
bound to advise the pledgor, without delay, of any danger to the thing
pledged.
d. The contract of pledge is extinguished by the sale of the thing pledged in the
public auction.
32. D borrowed P10,000 from C. To secure the fulfillment of the loan, D
pledged his laptop. The contract of pledge provides that the creditor-pledgee 39. Grace obtained a loan from Jojo in the amount of P1M with Mar serving as
may appropriate the laptop upon failure of the debtor-pledgor to pay the guarantor. Rody pledged his cellphone to secure the debt of Grace. If Rody
loan. On the date of maturity of the loan. D failed to pay the loan. Which of pays P1M loan of Grace, which is correct?
the following statements is correct? a. There will be no legal subgrogation if Grace does not consent to the payment
a. C does not become the automatic owner of the laptop upon D’s failure to of Rody.
pay the loan because that provision is considered pactum commissorium b. Rody cannot collect the P1M from Mar if Grace will not be able to pay
which is contrary to law and public policy. Rody.
b. The contract of pledge is null and void because of pactum commissorium c. Rody can collect the P1M from Mar if Grace will not be able to pay Rody.
provision. d. Mar is not liable to Rody.
c. The laptop cannot be alienated for the payment of the loan.
d. C becomes the owner of the laptop. 40. Which of the following statements is incorrect in case the pledgee renounces
or abandons in writing the contract of pledge?
33. What is the remedy of the pledgor-owner if there are reasonable grounds to a. The pledgee becomes a depository upon renunciation if in the meantime, the
fear the destruction or impairment of the thing pledged, without the fault of thing pledged is not returned to the owner.
the pledgee? b. The contract of pledge is extinguished but the contract of loan remains.
a. To ask for the extinguishment of the contract of loan. c. The return of the thing pledged is necessary for extinguishing the contract
b. To require that it be deposited with a third person. of pledge for this mode of contract of pledge extinguishment.
c. To ask for the extinguishment of the contract of pledge. d. The acceptance by the pledgor is not necessary for extinguishing contract of
d. To ask for the return of the thing, upon offering another thing in pledge, pledge.
provided the latter is of the same kind as the former and not of inferior
quality but subject to the right of the pledgee to sell the said pledged item in 41. D borrowed P20.000 from C. To secure the fulfillment of the loan, D
public auction. mortgaged a land owned by his ailing father. Which of the following
statements is correct?
34. A third person who pledges his own movable property to secure the debt of a. The contract of mortgage will become valid upon the death of D’s father.
another shall be released from liability in the following cases and may ask b. The contract of loan is null and void because the contract of mortgage is null
for the return of the thing pledge from the pledgee, except and void.
a. If through some act of the creditor, the pledgor cannot be subrogated to the c. The contract of mortgage is valid because future property may be pledged
rights, mortgages and preferences of the creditor. or mortgaged.
b. If the creditor voluntarily accepts immovable or other property in payment d. The contract of mortgage is null and void because the mortgagor must be
of the debt even if the creditor thereafter loses the same eviction. the owner of the property mortgaged at the time it is constituted but the
c. If an extension of time is granted to the debtor by the creditor without contract of loan remains to be valid.
pledgor’s consent.
d. If the thing pledged is deteriorated on the fault of the pledgee. 42. In case the bids of the pledgor-owner, the pledgee and a third person are
e. If the debtor defaults in the payment of principal obligation on the maturity equal and considered the highest bid, who shall be preferred among them?
date. a. Third person
b. No answer text provided.
35. The contract of pledge shall cover the following, except c. Pledgor-owner
a. The fruits, income, dividends or interests earned or produced by the thing d. Pledgee
pledged, unless there is stipulation excluding them.
b. The thing pledged. 43. The following are the essential requisites of conventional pledge or contract
c. The offspring, when the thing pledged is an animal, unless there is a of pledge, except
stipulation excluding them. a. That the pledgor be the absolute owner of the thing pledged.
d. The future inheritance of the pledgor. b. That person constituting the pledge has the free disposal of his property, and
in the absence thereof, that he be legally authorized for the purpose.
36. What is the form of contract of pledge to bind the contracting parties? c. That the contract of pledge be constituted in public document.
a. It must be registered in the registry of deeds. d. That it be constituted to secure the fulfillment of a principal obligation or
b. It may be in any form because it is a real contract. contract of loan.
c. It must be in a public instrument. e. That the thing pledged be placed in the possession of the creditor, or a third
d. It must be in a private instrument. person by common agreement or there must be delivery of the thing pledged.

37. A and B jointly borrowed P10,000 from C. In order to secure the obligation, 44. If the thing is found in the possession of the pledgor or owner or if the thing
A pledged his cellphone while B pledged his laptop. At the maturity date of is in the possession of a third person who has received it from the pledgor
the loan, A paid P5,000 of the loan. Which of the following statements is or owner, which of the following is incorrect?
correct? a. There is prima facie presumption that pledge returned the thing pledged.
a. A cannot demand the release of his cellphone because a contract of pledge b. There is prima facie presumption that the contract of pledge is extinguished.
is indivisible. c. There is prima facie presumption that the contract of loan is extinguished.
b. The contract of pledge on the cellphone is extinguished.
c. The contract of pledge on the laptop is extinguished. 45. Which of the following direct modes of extinguishing contract of pledge do
d. The obligation of B is extinguished. not impliedly extinguish the principal obligation or contract of loan?
I. Return by the pledgee of the thing pledged to the pledgor or owner.
38. If there are reasonable grounds to fear the destruction or impairment of the II. Renunciation or abandonment in writing by the pledgee of the contract
thing pledged, without the fault of the pledgee, which is correct? of pledge.

47

0 0
III. Sale of the thing pledged in public auction in case of default by debtor c. Pactum debitarium
regardless of the amount of the net proceeds of sale. d. Pactum crematorium
IV. Appropriation of the thing pledged by the pledgee in case the thing
pledged is not sold in the first and second public auctions. 53. What is the nature of a contract to constitute a pledge?
a. I and II a. Real contract
b. III and IV b. Formal contract
c. II and IV c. Consensual contract
d. I and III d. Solemn contract

46. What is the legal effect of a promise or contract to constitute a pledge? 54. What is the remedy of the pledgor-owner if the thing pledged is in danger
a. It is perfected by mere consent and gives rise only to a personal action of being lost or impaired through the negligent or willful act of the pledgee?
between the contracting parties. a. To ask for the extinguishment of the contract of loan.
b. It gives rise to a real action over the thing pledged. b. To ask for the extinguishment of the contract c f pledge.
c. It is perfected through the notarization of the promise. c. To require that it be deposited with a third person.
d. It is perfected by delivery of the thing pledge. d. To ask for the return of the thing pledged.

47. Which of the following statements concerning formalities of real estate 55. It is a contract whereby the debtor or third person secures to the creditor the
mortgage is incorrect? fulfillment of a principal obligation, specially subjecting to such security
a. The real mortgage must be registered in the Registry of Property to bind immovable property or real rights over immovable property in case the
third persons. principal obligation is not complied with at the time stipulated.
b. The real mortgage must be in a public instrument for the convenience of the a. Antichresis
parties but not for validity. b. Pledge
c. The real mortgage may be in any form to be valid since it is a consensual c. Correct!
contract. d. Real estate mortgage
d. The real mortgage must be in writing to be valid since it is a formal contract. e. Chattel mortgage

48. The provisions on conventional pledge on the possession, care and sale of
the thing as well as on the termination of the pledge shall be applicable to
legal pledge except with respect to the sale of the thing as follows. The 56. The following are the characteristics of a contract of pledge, except
following are the rules applicable to legal pledge on the sale of the thing a. Subsidiary - The obligation incurred does not arise until the fulfillment of
pledged, except the principal obligation which is secured.
a. If without just grounds, the creditor dues not cause the public sale to be held b. Accessory - It has no independent existence of its own because there must
within such period, the debtor may require the return of the thing. be contract of loan.
b. After payment of debt and expenses, the remainder of the price of sale shall c. Consensual - It is perfected by mere consent.
be retained by the pledgee-creditor. d. Unilateral - It creates an obligation on the part of the creditor to return the
c. In case there is deficiency from the public sale, the pledgee-creditor may thing upon the fulfillment of the principal obligation.
still recover the deficiency even if not stipulated.
d. The thing may be sold only after demand of the amount for which the thing 57. The following are examples of legal pledge, except
is retained. a. A possessor in good faith may retain the movable upon which he has
e. The public auction shall take place within one month after such demand. incurred necessary and useful expenses until he has been reimbursed
therefore.
49. Which of the following stipulations in a contract of pledge is null and void? b. The depositary may retain the thing deposited until the full payment of what
I. A stipulation which provides that the contract of pledge is not may have been due from him by reason of the deposit
extinguished by the return of the thing pledged. c. He who has executed work upon movable has a right to retain it by way of
III. A stipulation allowing the appropriation by the pledgee of the thing pledge until he is paid.
pledged in case the same is not sold in the first and second auctions. d. A contract by virtue of which the debtor delivers to the creditor or to a third
III. A stipulation for the recovery of deficiency in case the proceeds from person a movable, or instrument evidencing incorporeal rights for the
the sale of the thing pledged is less than the amount of the obligation. purpose of securing the fulfillment of a principal obligation with the
a. I and II understanding that when the obligation is fulfilled, the thing delivered shall
b. I and III be returned with all its fruits and accessions.
c. I, II and III
d. II and III 58. The following are the rights of the creditor-pledgee, except
a. If the pledge earns or produces fruits, income, dividends, or interests, the
50. Which of the following statements pertains to equitable mortgage? creditor shall compensate what he receives with those which are owing him;
a. No answer text provided. but if none are owing him, or insofar as the amount may exceed that which
b. It is one which is created by the agreement of the parties. is due, he shall apply it to the principal. Unless there is a stipulation to the
c. It is one which although lacks certain formality, form or words or other contrary, the pledge shall extend to the interest and earnings of the right
requisites provided by statute, shows the intention of the parties to charge pledged.
the real property as a security for a debt and contains nothing contrary to b. To appropriate the thing pledged in case the thing pledged is not sold in first
law. and second public auctions.
d. It is one executed pursuant, to an express requirement of a provision of law. c. To cause the sale of the thing pledged at a public sale, if there is a danger of
destruction, impairment or diminution of value of the thing pledged without
51. If two or more things are pledged, who has the right to choose which thing his fault. The pledgee is bound to advise the pledgor, without delay, of any
will be sold in the absence of stipulation? danger to the thing pledged.
a. Debtor d. To bring actions which pertain to the owner of the tiling pledged in order to
b. Government recover it from, or defend it against third person.
c. Pledgor e. To collect and receive the amount due if the tiling pledged is a credit which
d. Pledgee becomes due before it is redeemed, and to apply the same to the payment of
his claim.
52. It is a stipulation whereby the thing pledged or mortgaged shall f. To use the thing pledged if he is authorized to do so, or when its use is
automatically become the property of the creditor in the event of non- necessary of the preservation of the thing.
payment of the debt within the term fixed. g. To sell the thing pledged upon default of the debtor.
a. Pactum creditarium h. To demand reimbursement of the expenses made for the preservation of the
b. Pactum commissorium thing pledged.

48

0 0
i. To become the automatic owner the thing pledged upon first default of
debtor to pay the principal obligation. 66. The following stipulations concerning real estate mortgage are null and
j. To retain in his possession the thing pledged until the debt is paid void, except
a. A stipulation prohibiting a second mortgage with respect to property
59. It is a type of pledge which refers to the right of a person to retain a thing registered under the Torrens System.
until he receives payment of his claim. b. Tipo or upset price which refers to price set by the parties as the amount at
a. Legal pledge or pledge by operation of law which the property at which the property will be sold at public auction.
b. Agreed pledge c. A stipulation providing that the mortgage shall become the automatic owner
c. Voluntary pledge of the property mortgaged upon failure of the debtor to pay the principal
d. Conventional pledge obligation.
d. A stipulation forbidding the owner form alienating the immovable
60. What is the remedy of the pledgor-debtor if the pledgee-creditor use the mortgaged.
thing pledged, without the authority of the owner or should misuse the thing
in any other way? 67. It is a contract by virtue of which the debtor delivers to the creditor or to a
a. To ask for the extinguishment of the contract of pledge. third person a movable, or instrument evidencing incorporeal rights for the
b. To ask for the return of the tiling pledged. purpose of securing the fulfillment of a principal obligation with the
c. To ask that it be judicially or extra-judicially deposited. understanding that when the obligation is fulfilled, the thing delivered shall
d. To ask for the extinguishment of the contract of loan. be returned with all its fruits and accessions.
a. The cause is the service remenerated.
61. Pledge may be extinguished directly or indirectly. The following are the b. The cause or consideration of the principal obligation or contract of loan.
modes of extinguishing the contract of pledge directly, except c. The cause or consideration is the liberality of the pledgor.
a. Sale of the thing pledged regardless of the net proceeds. d. It has no cause or consideration.
b. Appropriation of the thing pledged by the pledgee if the thing pledged is not
sold in the first and second auctions. 68. What is the form of contract of pledge to bind or to affect third persons?
c. turn by the pledgee of the thing pledged to the pledgor or owner. a. It must be recorded in the Intellectual Property Office.
d. Renunciation or abandonment in writing by the pledgee of the pledge. b. It must be registered in the registry of deeds.
e. When the principal obligation secured by the pledged is extinguished. c. It must be registered in the chattel mortgage registry.
d. It must be in a public instrument showing a description of the thing pledged
and the date of the pledge.

62. The following statements pertaining to a promise to constitute a pledge or


mortgage are correct, except
a. It creates no real right in the property but only a right to compel the
fulfillment of the promise but there is no contract of pledge or contract of
mortgage yet.
b. The contract perfected is a real contract.
c. It gives rise only to a personal action between the contracting parties.
d. The promissor will be criminally liable if he mortgages or pledges as 69. The following requisites are essential to the contracts of pledge, real estate
unencumbered things which he know were subject to some burden. mortgage and chattel mortgage, except
a. That when the principal obligation becomes due, the things in which the
63. The following are the instances where the thing pledged or mortgaged may pledge or mortgage consists may be alienated for the payment of the
be sold or alienated to pay the principal obligation, except creditor.
a. If the debtor has lost the right to make use of the period or where there is an b. That the persons constituting the pledge or mortgage have the free disposal
acceleration clause in the payment of installment. of their property, and in the absence thereof, that they be legally authorized
b. Upon default to pay the obligation at maturity. for the purpose.
c. If the pledgor or mortgagor fails to fulfill certain conditions and such c. That the pledgor or mortgagor be the absolute owner of the thing pledged or
violation would make the debt due and demandable mortgaged.
d. Before maturity of the principal obligation. d. That the thing pledged or mortgaged be placed in the possession of the
creditor, or of a third person by common agreement.
64. The following are the obligations of creditor-pledgee, except e. That they be constituted to secure the fulfillment of a principal obligation
a. To deposit the thing pledged with a third person even not authorized by the such as contract of loan.
court.
b. To deliver to the debtor the surplus after paying his claim from what he has 70. D borrowed P10,000 from C and pledged his ring and watch with P4,000
collected on a credit that was pledged and which has become due before it and P6,000 value respectively. They agreed that the ring will secure P4,000
is redeemed. of the loan and the watch will secure the balance of the loan. After several
c. To be liable for the loss or deterioration of the thing pledged unless it is due days, D pays P4,000 to C. Which of the following statements is correct?
to a fortuitous event. a. The contract of pledge on the watch is extinguished.
d. To take care of the thing pledged with the diligence of a good father of a b. The contract of loan is fully extinguished.
family. c. D may compel C to return the ring because contract of pledge on the ring is
e. To be responsible for the acts of his agents or employees with respect to the extinguished.
thing pledged. d. D cannot demand the release of his ring because a contract of pledge is
f. Not to use the thing pledged except when he is authorized by the owner or indivisible.
when the use of the thing is necessary for its preservation.

65. The real estate mortgage shall cover the following (R-I-N-G-I-R)
a. Real property mortgaged
b. Improvements
c. Growing fruits
d. Rents and income not yet received when the obligation becomes due
e. Indemnity granted or owing to the proprietor from the insurers of the
property mortgaged or in virtue of expropriations for public use.
f. Natural accessions

49

0 0
d. No, only if there is stipulation
1. The following are instances of pledge created by operation of law, except
a. Hotelkeeper retains the things brought into the hotel by the guest who cannot
pay his hotel bill
b. A mechanic retains the car he repaired until he is paid
c. A depository retains the thing in pledge until full payment of what is due
him 10. A owes C P5,000. The debt is secured by a pledge of G's watch. If G pays
d. An agent retains in pledge the thing which is the object of the agency until C, which of the following statements is correct?
he paid his commission a. G is subrogated with the rights of the creditor
b. G cannot recover from A.
2. If the thing is found in the possession of the pledgor or owner or if the thing c. The contract of pledge is not extinguished because the contract of loan is
is in the possession of a third person who has received it from the pledgor subsisting
or owner, which of the following is incorrect? d. The contract of loan cannot be extinguished by the payment of a thirs person
a. There is prima facie presumption that pledge returned the thing pledged
b. None of the choices 11. D borrowed P10,000 from C. To secure the fulfillment of the loan, D
c. There is a prima facie presumption that the contract of loan is extinguished pledged his laptop. The contract of pledge provides that the creditor pledgee
d. There is prima facie presumption that the contract of pledge is extinguished may appropriate the laptop upon failure of the debtor-pledgor to pay the
loan. On the date of maturity of the loan, D failed to pay the loan. Which of
3. Pledge may be extinguished directly or indirectly. The following are the the following statements is correct?
modes of extinguishing the contract of pledge directly, except a. The contract of pledge is null and void because of pactum commissorium
a. When the principal obligation secured by the pledged is extinguished provision.
b. Return by the pledgee of the thing pledged to the pledgor or owner b. C does not become the automatic owner of the laptop upon failure to pay
c. Appropriation of the thing pledged if the thing pledged is not sold in the first the loan because that provision is considered pactum commissorium which
and second auctions is contrary to law and public policy
d. Renunciation or abandonment in writing by the pledge of the pledge c. The laptop cannot be alienated for the payment of the loan
d. D becomes the owner of the laptop
4. X borrowed money from Y and gave a piece of land as security by way of
mortgage. It was agreed between the parties that upon non-payment of the 12. D borrowed P1M from C. G, a third person, mortgaged his land to secure
loan, the land would already belong to Y. If X failed to pay the debt, would the fulfillment of D's loan. Is the contract of mortgage valid?
I now become the owner of the land? a. Yes provided that G will deliver the land to C
a. Y would not become the owner because the agreement that he would b. No because G is no privy to the contract of loan
become the owner default of X is against the law c. No because D must be the owner of the mortgaged land
b. Y would become the owner but with right of redemption by X d. Yes because third persons who are not parties to the principal obligation
c. Y would become the owner because it was agreed upon by them based on may secure the latter by pledging or mortgaging their own property
the principle of autonomy of contracts
d. No, provided the stipulation is in writing 13. If there are reasonable grounds to fear the destruction or impairment of the
thing pledged, without the fault of the pledge e, which is correct?
5. Which of these is not an essential requisite common to both pledge and a. The contract of pledge is extinguished by the sale of the thing pledged in the
mortgage? public auction
a. That the pledgor or mortgagor has the free disposal of the properties b. The pledge creditor may cause the sale of the thing pledged at a public sale
b. That the pledge or mortgage be constituted to secure the fulfillment of a and the proceeds of the auction shall be a security for the principal obligation
principal obligation in the same manner as the thing originally pledged
c. That the pledgor or mortgagor be the principal debtor c. The right of the pledgor-owner is superior to the right of the pledge creditor
d. That it is the essence of the pledge or mortgage that if the obligation is not d. The pledgor-owner may ask for the return of the thing, upon offering another
paid the things in which the pledge or mortgage consist may be alienated for thing in pledged, provided the latter is of the same kind as the former and
the payment of the debt not of inferior quality even it is already sold in public auction by the pledgee
creditor
6. What is the remedy of the pledgor-owner if there are reasonable grounds to
fear the destruction or impairment of the thing pledged, without the fault of 14. The following are the kinds of principal obligations that may be secured by
the pledgee? a pledge or mortgage, except
a. To ask for extinguishment of the contract of loan. a. Pure or conditional obligations whether suspensive or resolutory
b. To require that it be deposited with a third person b. Rescissible, voidable or unenforceable obligations
c. To ask for extinguishment of the contract of pledge c. Null and void obligations
d. To ask for the return of the thing upon offering another thing in pledge d. Natural obligations
provided the latter is of the same kind as the former and not of inferior
quality 15. A mortgaged his Rolex watch to B for a loan of P.2M. Due to the failure of
A to redeem the security, B sold the same at public auction for P.15M to the
7. D is indebted to C in the amount of P200,000 and delivers to C his diamond highest bidder.
ring by way of pledge. If D sells the same diamond ring to T, when will T a. B can recover the deficiency of P.05M from A
acquire ownership of the same? b. B can recover the deficiency if there is no contrary agreement
a. From the time T pays the price to D c. B cannot recover the deficiency even if there is stipulation
b. From the time C consents to the sale between D and T d. B can recover the deficiency from A if so stipulated
c. From the time the sale is perfected between D and T
d. From the time T obtains actual possession of the diamond ring 16. A pledged his Toyota car to 8 for a loan of P1M. A was unable to pay the
loan and therefore B sold the car in a public auction but it was sold only for
8. Element of contracts of pledge and mortgage, except P.5M. Can B recover the deficiency from A?
a. Pledge and mortgage are accessory contracts a. Yes, he can provided that it was agreed upon by the parties
b. The thing pledged or mortgaged may be appropriated if the debtor cannot b. No, he cannot recover the deficiency in the absence of contrary stipulation
pay c. No, he cannot even if there is an agreement that he can
c. Pledgor or mortgagor must be the absolute owner d. Yes, he can even in the absence of stipulation allowing him
d. Pledgor or mortgagor must have the free disposal of the thing pledged
17. The following requisites are essential to the contracts of pledge and
9. A pledged his watch to B for P20,000. A failed to pay his obligation. B sold mortgage, except
it at public auction for P18,000. Can B recover the deficiency? a. That they are constituted to secure the fulfillment of a principal obligation
a. Yes, if there is stipulation b. That the persons constituting the pledge or mortgage have the free disposal
b. Yes, even without stipulation of their property, and in the absence thereof, that they be legally authorized
c. No, even if there is stipulation for the purpose

50

0 0
c. That the pledgor or mortgagor be the absolute owner of the thing pledged or sale of the bracelet in favor of D for P10,000 only. Which of the following
mortgaged statements is correct?
d. That the thing pledged or mortgaged be placed in the possession of the a. If D was a purchaser in bad faith as he knew of the defective title of B over
creditor, or of a third person by common agreement the bracelet from S, ownership will not pass to him (D).
e. That when the principal obligation becomes due, the things in which the b. The deficiency of P5,000 may still be recovered by C from B if there is a
pledge or mortgage consists may be alienated for the payment of the creditor stipulation to this effect.
c. C can no longer recover the deficiency of P5,000 from B. the pledge,
together with the sale is valid. The voidable title of B is valid because it is
not yet annulled.
d. The title of B over the bracelet is not valid, hence the pledge, as well as the
sale of said bracelet is likewise defective. The pledgor must be the owner of
18. A pledge his wristwatch to B for a loan of P.1M. Due to A's failure to pay the thing pledged.
his debt, B auctioned the watch and was sold to C for P.15M. Can A recover
the excess?
a. A can recover the excess even if not so stipulated
b. A stipulation allowing the excess to go to the pledgor is void
c. A cannot recover the excess even if so stipulated by the parties
d. A can recover the excess if so agreed upon by the parties

19. What is the nature of contract of pledge or mortgage?


a. A pledge or mortgage is divisible if the principal contract is joint
b. A pledge or mortgage is indivisible if the principal contract is solidary 26. The following are examples of legal pledge, except
c. A pledge or mortgage is indivisible whether the principal contract is joint or a. The depositary may retain the thing deposited until the full payment of what
solidary may have been due from him by reason of the deposit
d. A pledge or mortgage is divisible whether the principal contract is joint or b. A contract by virtue of which the debtor delivers to the creditor or to third
solidary person a movable, or instrument evidencing incorporeal rights for the
purpose of securing the fulfillment of a principal obligation with the
20. D borrowed P10,000 from C and pledged his ring and watch with P4,000 understanding that when the obligation is fulfilled, the thing delivered shall
and P6,000 value respectively. After several days, D pays P4,000 to C. be returned with all the its fruits and accessions
Which of the following statements is correct? c. He who has executed work upon movable has a right to retain it by way of
a. D may compel C to return the ring because P4,000 of the loan is already pledge until he is paid
paid d. A possessor in good faith may retain the movable upon which he has
b. The contract of pledge is extinguished incurred necessary and useful expenses until he has been reimbursed
c. D cannot demand the release of his ring because a contract of pledge is therefore
indivisible
d. The contract of loan is extinguished 27. It is a stipulation whereby the thing pledged or mortgaged shall be
automatically become the property of creditor in the event of non-payment
21. The contract of pledge shall cover the following, except of the debt within the term fixed
a. The offspring, when the thing pledged is an animal, unless there is a. Pactum debitarium
stipulation excluding them b. Pactum crematorium
b. The fruits, income, dividends or interest earned or produced by the thing c. Pactum commissorium
pledged, unless there is stipulation excluding them d. Pactum creditarium
c. The inheritance of the pledgor
d. The thing pledged 28. What is the remedy of the pledgor-debtor if the pledgee creditor use the
thing pledged, without the authority of the owner or should misuse the thing
22. The following are instances of pledge created by operation of law, except: in any other way?
a. A mechanic retains the car he repaired until he is paid a. To ask for the extinguishment of the contract of pledge
b. An agent retains in pledge the thing which is the object of the agency b. To ask that it be judicially or extra judicially deposited
regarding his commission c. To ask for the extinguishment of the contract of loan
c. Hotel keeper retains the things brought into the hotel by the guest who d. To ask for the return of the thing pledge
cannot pay his hotel bills
d. A depository retains the thing deposited until he is paid the charges due him 29. What is the legal effect of a promise or contract to constitute a pledge?
a. It is perfected by mere consent and gives rise only to a personal action
23. The provisions on conventional pledge on possession, care and sale of the between the contracting parties
thing as well as on the termination of the pledge shall be applicable to legal b. It is perfected be delivery of the thing pledge
pledge except with respect to the sale of the thing as follows. The following c. It is perfected through the notarization of the promise
are the rules applicable to legal pledge on the sale of the thing pledged, d. It gives rise to a real action over the thing pledge
except
a. After payment debt and expenses, the remainder of the price of sale shall be 30. Onyok pledged his motor bike to Pepito for P5,000.00. When Onyok failed
retained by the pledgee-creditor to pay his obligation, Pepito sold the motor bike at public auction to the
b. The thing may be sold only after demand of the amount of which the thing highest bidder for P4,000.00. Therefore
is retained a. Pepito can recover P1,000.00 from Onyok even without stipulation
c. If without just grounds, the creditor does not cause the public sale to be held b. Pepito cannot recover P1,000.00 from Onyok even if there is stipulation
within such period, the debtor may require the return of the thing c. Pepito cannot recover P1,000.00 from Onyok unless stipulated that he can
d. The public auction shall take place within one month after such demand. d. Pepito can recover P1,000.00 from Onyok if it is stipulated

24. What is the remedy of the pledgor-owner of the thing pledged is in danger 31. The following are the instances where the thing pledged or mortgaged may
of being lost or impaired? be sold or alienated to pay the principal obligation, except
a. To ask for the extinguishment of the contract of loan a. If the debtor has lost the right to make use of the period or where there is an
b. To require that it be deposited with a third person acceleration clause in the payment of installment
c. To ask for the extinguishment of the contract of pledge b. If the pledgor or mortgagor fails to fulfill certain conditions and such
d. To ask for the return of the thing pledged violation would make the debt due and demandable
c. Before maturity of the principal obligation
25. S, a minor, sold her bracelet to B for P8,000. Later, B, needing money to d. Upon default to pay the obligation at maturity
pay her daughters tuition fee, borrowed P15,000 from C and as a security,
pledged the bracelet to the latter. B failed to pay C resulting into the auction 32. A pledged his wristwatch to B pawnshop for a loan of P7,000 which he
failed to pay. B sold the watch at public auction for P4,000.

51

0 0
a. it cannot recover the deficiency even if there is stipulation allowing it 1
b. The pawnshop B can recover the P4,000 deficiency from A 3
c. It cannot recover the deficiency if there is stipulation disallowing recovery
d. It cannot recover the deficiency from A unless there is stipulation to the 41. Who chairs the Ease of Doing Business/Anti-Red Tape Advisory
contrary Council?
DILG Secretary
33. The following are the obligations of the debtor pledgor, except DOF Secretary
a. None of the choices DTI Secretary
b. To pay for the expenses which are not necessary for the preservation of the
thing pledged 42. Which of the following is not a function of the Anti-Red Tape
c. To pay the debt and its interest, with expenses, in a property case, when they Authority?
are due Review proposed major regulations of government agencies, upon
d. To pay damages that the pledge may suffer by reason of the flaws of the submitted regulatory impact assessments
thing pledged, if he was aware of such flaws but did not advise the pledge Monitor compliance of agencies and issue notices to erring and non-
of the same compliant government employees and officials
Implement and oversee national policy on anti-red tape and ease of doing
business and implement reforms to improve competitiveness ranking
Initiate investigation motu propio only or file cases for violations

43. In case a government agency fails to approve or disapprove an original


application within the prescribed processing time, the said application shall
be __.
Returned to the filer
Deemed approved
Deemed denied
34. In 2012, D borrowed P.4M from C, collateralized by a pledge of shares of Deemed pending
stock of X corporation worth P.8M. In 2013, because of the economic crisis,
the value of the shares pledged fell to only P.1M. Can C demand that D 44. Who appoints the two (2) representatives from the private sector to sit
surrender the other shares worth P.7M? in the Ease of Doing Business/AntiRed Tape Advisory Council?
a. No, because the only right of C is to sell the shares at the public auction and DTI Secretary
keep the proceeds as security for the loan. Executive Secretary
b. No, because the right of C is to demand payment of the loan. DOF Secretary
c. Yes, because the collateral as security for the loan was worth P.8M and President
therefore the other shares of P.7M must be delivered.
d. No, because the right of C is to demand another thing worth P.8M. 45. Which of the following transactions must be processed within 20
working days?
35. When is the pledgor or mortgagor required to be the owner of the thing Basic transactions
pledged or mortgaged for the validity of contract of pledge or mortgage? Highly technical transactions
a. At the time of the failure to pay the principal obligation Complex transactions
b. At the time the principal obligation is constituted Simple transactions
c. At the time the thing to be pledged or mortgaged is to be delivered
d. At the time the pledge or mortgage is constituted

36. What shall provide NGAs/LGUs access to data and information to verify
the validity and existence of business entities?
a. Philippine Business Databank
b. Philippine Business Portal
c. Philippine Business Database
d. Philippine Business Data Registry

37. How long is the period of suspension for government officials or


employees violating the Ease of Doing Business Act for the first time?
1 year
9 months
3 months
6 months

38. Which government agency is mandated to establish a central business


portal to receive and capture application data on business-related
transactions, and provide links to online registration of national
government agencies?
DOTr
DOF
DICT
DTI

39. The Ease of Doing Business/Anti-Red Tape Advisory Council is


composed of how many persons?
3
9
7
5

40. Simple transactions with government agencies must be processed


within __ working days:
5
2

52

0 0
Cession of conjugal partnership of gain

1. Which of the following is not correct?


6. Seno entered into a contract with Brim by threatening Brim that if Brim does
Group of answer choices not agree to make the contract, Seno will publish defamatory matter concerning
Brim’s wife:
The obligation to give includes that of delivering all its accessories and even
though they may not have been mentioned. Group of answer choices

If a person obliged to do something fails to do it, the same shall be executed at The contract is valid because the defamatory matter to be published does not
his cost. relate to Brim, the contracting party.

When the obligation consists in not doing, and the obligor does what has been The contract is voidable because such publication when carried out whether
forbidden him, it shall be undone at his expense. true or not will cause a serious harm to Brim and his wife.

In reciprocal obligations where no period was agreed upon from the moment The contract is unenforceable but Brim is entitled to damages the moment the
one of the parties fulfills his obligations, delay by other begins. publication is made.

The contract is void.

2. 1st statement – Negotiorum gestio and solutio indebiti are innominate


contracts
2nd statement - Negotiorum gestio and solutio indebiti are implied 7. The following are all factors vitiating consent, except:
contracts
Group of answer choices
Group of answer choices
Mistake
Both statements are false.
Negligence
1st statement is false, 2nd statement is true.
Intimidation
Both statements are true.
Fraud
1st statement is true, 2nd statement is false.

8. The obligation is unenforceable because the condition is immoral.


3. To cancel a contract and restore the parties to their original positions before
the contract, the parties should execute a: The obligation is void because of the positive impossible condition.

Group of answer choices The obligation is valid because the impossible condition is negative.

Novation The obligation is voidable, not enforceable.

Release

Rescission 9. 1st statement – In novation by way of expromission, there is revival of the


original debtor’s obligation if the new debtor’s insolvency is known to the
Revocation original debtor and also known to the public.
2nd statement – In novation by way of delegation, there is revival of the
original debtor’s obligation if the new debtor is insolvent at the time of
delegacion and such insolvency is known to the public even if not known to the
4. Which of the following contracts is not void ad initio? original debtor.

Group of answer choices Group of answer choices

That whose object is outside the commerce of men. Both statements are false.

That whose object did not exist at the time of transaction. Both statements are true.

That which contemplates an impossible service. First false, second true

That which is undertaken is fraud of creditors. First true, second false

5. The following contract must be in a public instrument for purpose of 10. Which of the following statement about cause is not correct?
convenient. Which is the exception?
Group of answer choices
Group of answer choices
Contract without cause is void.
Donation of real property
Statement of a false cause in contract shall render them void.
Repudiation of hereditary right
Although the cause is not stated in the contract, it is presumed that it exists and
Power to administer property is lawful.

53

0 0
When the motive of one contracting party is contrary to law, the contract is Group of answer choices
void.
Least expensive thing

Most expensive thing


11. An Incidental element of a contract
Last thing which disappeared
Implied warranty against eviction
Payment of interest in a loan First thing which disappeared
Delivery of the object in contract of pledge
17. When a condition has been imposed with the intention of suspending the
All of the foregoing
efficacy of an obligation to give, which of the following shall be observed
during the pendency of the condition?

Group of answer choices


12. Which of the following is an accessory thing?
If the thing is lost, the obligation shall be extinguished.

When the thing deteriorates, the impairment is to be borne by the creditor.


Calf of cow
If the thing perished, or goes out of commerce, or disappears in such a way that
Milk of a goat
its existence is unknown or it cannot be recovered, the debtor shall be obliged
Saddies of a horse to pay damages.

Bird feeds If the thing is improved by nature, or by time, the improvement shall inure to
the benefit of the creditor.

18. The following are requisite of fortuitous event, except:


13. Which of the following can be a valid object of a contract?
Group of answer choices
Group of answer choices
Cause is independent of the will of the debtor.
right to vote in a national election
The event is unforeseeable/ unavoidable.
luneta park
Occurrence renders it absolutely impossible for the debtor to fulfill his
wild animals obligation in a normal manner; impossibility must be absolute not partial,
otherwise not force majeure.
creditor’s right to demand payment
Debtor contributed to the aggravation of the injury to the creditor.

14. 1st statement – In civil obligation, the fulfillment of the obligation depends
solely on the conscience of the person. 19. S orally sold to B his house and lot for P5,000,000 where B initially paid
2nd statement – A quasi contract does not require consent of the P50,000. After paying the balance, B now seeks to register the lot in his name,
contracting parties but the Register of Deeds refuses to do so. In this case:

Group of answer choices Group of answer choices

1st statement is true, 2nd statement is false. The transaction involving the oral sale of a house and lot is null and void.

Both statements are false. The contract between S and B is unenforceable under the Statute of Fraud.

1st statement is false, 2nd statement is true. The oral sale of the house and lot although unenforceable under the Statute of
Fraud has been ratified by the receipt of the consideration and, therefore, B can
Both statements are true. compel S to execute the deed of sale in a public document.

If S does not want to execute the public document, he can rescind or cancel his
agreement with B.
15. Three of the requisites of cession in payment. Which is the exception?
20. In facultative obligations, if substitution has been made, which of the
Group of answer choices following is false?
one debtor and one creditor
Group of answer choices
complete or partial insolvency
The obligations is extinguished
more than one debt
The loss of the original prestation is immaterial
abandonment of all debtor’s property not exempt from execution The obligation is converted into a simple obligation

The obligation ceases to be facultative

16. The creditor shall have a right to indemnity for damages when, through the 21. Contracts are effective and binding only between the parties, their assigns
fault of the debtor, all the things which are alternatively the object of the
and their heirs. Three of the following enumerations are exceptions as provided
obligation have been lost or compliance of the obligation have become
by law. Which does not belong to the exception?
impossible. The indemnity shall be fixed taking as a basis the value of the:

54

0 0
Group of answer choices

Where there is a stipulation in favor of a third party. 26. A wrote a letter to B wherein A offered to sell a piece of land to B for
P200,000. B signified his desire to buy the land. In A’s letter, B was given a
Where one of the parties to the contract dies and thereafter a suit is filed on the period of two (2) months within which to produce the P200,000. After 45 days,
basis of the contract. A told B that price of the land is now P250,000. Can B compel A to accept the
P200,000 first offered by A and execute the deed of sale?
Where the obligations arising from contract are not transmissible by their
nature. Group of answer choices

Where the obligations arising from context are not transmissible by stipulation Yes, because there was actual meeting of the minds of the parties.
or by provision of law.
No, for B did not signify his acceptance of A’s offer.
22. 1st statement – There is no delay in obligations not to do.
2nd statement – As a rule, the debtor will be in default if he does not perform Yes, because A is already estopped by his signed letter.
his obligation on due date
Yes, because the period of two (2) months has not expired.
Group of answer choices

1st statement is true, 2nd statement is false


27. Iris bought the car from Joe and delivered a check in payment of the same.
1st statement is false, 2nd statement is true Has Iris paid the obligation, why?

Both statements are false Group of answer choices

Both statements are true No, not yet. The delivery of promissory note payable to order or bills of
exchange or other mercantile documents shall produce the effect of payment
only when they have been cashed or when through the fault of the creditor they
have been impaired.
23. Which of the following is not a personal property?
Yes, because a check is a valid legal tender of payment.
Group of answer choices
It depends, if the check is a manager’s check or a cashier’s check, it will
Certificate of stock produce the effect of payment. If it is an ordinary check, no payment.

Electricity distributed by Decorp or Panelco Yes, because a check is as good as cash.

Animals

Hanging bridge 28. 1st statement – Unenforceable contract is less defective than a voidable
contract.
2nd statement- The action to declare a void contract a nullity pre scribes in
four (4) years.
24. Crisostomo donated a parcel of land to Anthony worth P500,000 in a public
instrument on January 21, 2024. Anthony accepted the same on the same day Group of answer choices
in a separate private instrument. When Anthony now seeks to register the land
in the Register of Deeds, the latter refused the registration because the Both are true
acceptance of the donated property was not in a public instrument. In this case:
Both are false
Group of answer choices
No. 1 is true; No.2 is false
Anthony can compel the Register of Deeds to register the land under his name
because the donation is valid. No. 1 is false; No. 2 is true

Anthony cannot compel the Register of Deeds to register the land under his
name because the donation is void.
29. X obliged himself to pay Y the amount of P30,000 30 days after May 31,
The donation is valid but the registration may be refused on the ground that the 2023 plus a penalty of P3,000 of he fails to pay the obligation on due date.
acceptance was not in a public instrument. After demand for payment by Y, offered to pay on July 1, 2023. Y can demand
from X
The Register of Deeds committed a grave abuse of discretion in refusing the
registration of the donated property in question. Group of answer choices

P30,000 plus P3,000 plus legal interest

25. Which of the following obligations will be extinguished by loss or P30,000 plus legal interest
destruction of a thing due?
P30,000 plus P3,000
Group of answer choices
P30,000 plus P3,000 plus legal interest plus damages
When the thing is lost before the debtor has incurred in delay.

When the obligation is to deliver a car with plate number ABC-123.


30. The creditor acquires this right upon the delivery of the subject matter.
When the thing is lost without the fault of the debtor.
Group of answer choices
When the obligation is to deliver a brand new pink apple I-phone 13 pro max.
Jus in re

55

0 0
Jus ad rem Legal obligations

Jus in personam Natural obligations

Personal right Industrial obligations

36. An incidental element of a contract

31. One of the following statements concerning ratification of a voidable Group of answer choices
contract is false. Which is it?
Implied warranty against eviction
Group of answer choices
Payment of interest in a loan
Ratification extinguishes the action to annul a voidable contract.
Delivery of the object in contract of pledge
Ratification cleanses the contract from all its defects from the moment it was
constituted. All of the foregoing

Ratification requires the conformity of the party who has no right to bring the
action for annulment.
37. The following contracts are not perfected until the delivery of the object of
Ratification may be made by the guardian of the incapacitated person, or the the obligations except:
incapacitated person upon attaining capacity, or the party whose consent was
vitiated. Group of answer choices

Pledge

32. Which of the following illustrates resolutory period? Deposit

Mutuum
Group of answer choices
Barter
A will support B until end of 2023.

A will support B if C will marry B.

A will support B after the death of C’s father. 38. A, B, and C solidarily owe X and Y P30,000. X remitted the entire
obligation in favor of A. The effect is:
A will support B until B pass the CPA Licensure exam.
Group of answer choices

The obligation is not extinguished until A collects from B and C.


33. When the delivery of the specific thing is subject to a suspensive condition
and if it is improved at the expense of the debtor: The obligation is not yet extinguished until Y is paid by X and Y’s share of the
credit.
Group of answer choices
A cannot recover from B and C because remission in his favor extend to the
The creditor shall enjoy the improvement only after payment of just benefit of B and C.
compensation.
A can recover from B and C their respective share of the debt.
The creditor shall have the right to enjoy the thing as well as its fruits.

The debtor shall have no other right than that granted to the usufructuary.
39. Juan ordered James, a ten-year old to climb a slippery tree, and promised to
The debtor shall acquire the right over the thing. share with him part of the fruits. James fell and was killed. Juan is liable for
damages because of his fault or negligence which is referred to as a:

Group of answer choices


34. If the obligation is payable in foreign currency, which is correct?
Contracts
Group of answer choices
Quasi-delicts
The obligation is void.
Law
The obligation is valid, but the stipulation is void.
Quasi-contract
The creditor can compel the debtor to pay in foreign currency as per
agreement.

The stipulation and the obligation are void. 40. Which of the following is correct?

Group of answer choices

35. The duty to pay taxes and to support one’s family are classified as: The nullity of the penalty clause carries with it that of the principal obligation.

Group of answer choices The creditor is entitled only to the penalty and cannot demand for additional
damages although there is refusal to pay on the part of the debtor or fraud in
Civil obligations the performance of the obligation.

56

0 0
Proof of actual damages suffered by the creditor is not necessary in order that required.
the penalty may be demanded. Statement 2 – that which is required by law shall be observed.

The penalty may not be equitably reduced by the courts. Group of answer choices

41. Asyong is a thief who is under obligation now to return the car to the owner Both statements are true.
Bosyo. The car was lost through an accidental fire. The obligation will:
Both statements are false.
Group of answer choices
Statement 1 is true
be extinguished because an accidental fire is always a fortuitous event.
Statement 2 is true
be extinguished because not all persons can avail of the remedies provided by
law.

be extinguished because the value of the car will be converted into the number 46. Ms. Maputi agreed to live with Mr. Hukluban as the wife of the latter
of days of Asyong will serve in prison. without the benefit of marriage in exchange for the monthly support of twenty-
five thousand (P25,000) that Mr. Hukluban would give to Ms. Maputi. Both
not be extinguished because the obligation proceeds from a criminal offense Mr. Hukluban and Ms. Maputi are single, of legal age, and there is no legal
hence Asyong will still have to pay the price of the car. impediment for them to get married. Their parents have no objections to the
two getting married. Based on the foregoing information, which of the
following statements is true?

42. 1st statement – Void contracts in certain cases are subject to ratification. Group of answer choices
2nd statement- The contract does not become voidable if the violence or
The agreement between Mr. Hukluban and Ms. Maputi is void for being
intimidation was done by persons not party to the contract.
contrary to morals.
Group of answer choices
Mr. Hukluban may legally demand that Ms. Maputi live with him as his wife.
Both are true
Ms. Maputi may legally demand that Mr. Hukluban give her the monthly
Both are false support of P25,000 that he had promised.

No. 1 is true; No.2 is false The agreement between Mr. Hukluban and Ms. Maputi is valid because they
can legally get married if they want to.
No. 1 is false; No. 2 is true

47. The statute of frauds:


43. Which of the following is true?
Group of answer choices
Group of answer choices
Prevents the use of oral evidence to contradict the terms of a written contract.
Acquittal in the criminal case will excuse the accused from civil liability.
Applies to all contracts having consideration valued at P500 or more.
If the obligation consists of giving, the creditor is entitled to the fruits and
accessories. Requires the independent promise to pay the debt of another to be in writing.

A quasi-contract does not require consent of the contracting parties. Applies to all real estate leases.

Responsibility arising from fraud is demandable is all obligations and any


waiver of right to sue for damages arising from fraud is void.
48. The absence of an agreement to the contrary what shall be the basis of
payment of an obligation in case there should supervene, an extraordinary
inflation or deflation of the currency stipulated?
44. Apple promised to sell her house and lot in Cebu to Bubbles if the latter
decides to live in Cebu. This obligation is: Group of answer choices

Group of answer choices The absence of an agreement to the contrary what shall be the basis of payment
of an obligation in case there should supervene, an extraordinary inflation or
with casual condition and is therefore valid. deflation of the currency stipulated?

with suspensive potestative condition on the part of the debtor and is therefore The value of the currency at the due date of the obligation.
void.
The value of the currency shall be fixed by courts.
with resolutory potestative condition on the part of the debtor and is therefore
The value of the currency shall be determined by an expert from the Bangko
valid.
Sentral ng Pilipinas.
with suspensive potestative condition on the part of the creditor and is therefore
valid.

49. The following are rescissible contracts, except:

45. If the contract stated the diligence which is to be observed in the Group of answer choices
performance of the obligation;
Statement 1 – that which is expected of a good father of a family shall be Entered into by guardian whenever ward suffers damage more than ¼ of value
of property.

57

0 0
Agreed upon in representation of absentees, if absentee suffers lesion by more 54. Arlene owns s row of apartment houses in Lucao, Dagupan City. She
than ¼ of value of property. agreed to lease Apartment 1 to Janet for a period of 18 months at the rate of
Php 10,000 per month. The lease was not covered by any contract. Janet
Contracts where fraud is committed on creditor (accion pauliana). promptly gave Arlene two months deposit and 18 checks covering rental
payment for 18 months. This show of good faith prompted Arlene to promise
Contracts entered into by minors. Janet that should Arlene decides to sell the property, she would give Janet the
right of first refusal.

Arlene owns s row of apartment houses in Lucao, Dagupan City. She agreed to
50. Dodoy asked Cocoy if the latter can grant him his washer and dryer combo lease Apartment 1 to Janet for a period of 18 months at the rate of Php 10,000
machine with a fair value of P60,000.00 for a term of one year and return the per month. The lease was not covered by any contract. Janet promptly gave
same. Cocoy nodded his head. The contract of commodatum is in the stage of: Arlene two months deposit and 18 checks covering rental payment for 18
months. This show of good faith prompted Arlene to promise Janet that should
Group of answer choices Arlene decides to sell the property, she would give Janet the right of first
refusal.
Preparation
Group of answer choices
Perfection
Yes, under the civil code, a promise to buy and sell a determinate thing is
Consummation reciprocally demandable.
Birth No, the promise to buy and sell a determinate thing was not supported by a
consideration.

Yes, Janet right of first refusal was clearly violated when the property was not
51. Aaron borrowed P100,000 from Jesart. To secure said loan, Aaron pledged
offered for sale to her before it was sold to Jun.
his Panerai watch worth P120,000. When Aaron failed to pay Jesart when the
loan became due, Jesart caused the public sale of the pledged watch, where it No, a right of first refusal involves an interest over real property that must be
was sold to Jessie for P80,000. How much can Jesart still collect from Aaron?
embodied in a written contract to be enforceable.
Nothing Next
P20,000

P40,000
55.
P80,000
is a tobacco trader and also a lending investor. He sold tobacco leaves to
Homer for delivery within a month although the period of delivery was not
guaranteed. Despite Gary’s efforts to deliver on time, transportation problems
and government red tape hindered his efforts and he could only deliver after 30
52. Legal subrogation is presumed in the following, except:
days. Homer refused to accept late delivery and to pay on the ground that the
agreed term had not been complied with. As lending investor, Gary granted a
Group of answer choices
Php 1M loan to Isaac to be paid within two years from the execution of the
contract. As security for the loan, Isaac promised to deliver to Gary his Toyota
When a third person, not interested in the obligation, pays with the approval of
Innova within seven days but Isaac failed to do so. Gary was thus compelled to
the creditor.
demand payment for the loan before the end of the agreed two-year term.
When a third person, not interested in the obligation, pays with the approval of
Can Gary compel Isaac to pay his loan even before the end of the two year
the creditor.
period?
When a third person, not interested in the obligation, pays with the approval of
Group of answer choices
the creditor.
Yes, Gary can compel Isaac to immediately pay the loan. Noncompliance with
When, even without the knowledge of the debtor, a person interested in the
the promised guaranty or security renders the obligation immediately
fulfillment of the obligation pays.
demandable. Isaac lost his right to make use of the period.

Yes, Gary can compel Isaac to immediately pay the loan. The delivery of the
Toyota Innova is a condition for the loan. Isaac’s failure to deliver the car
53. A borrowed P1M from B. The loan was secured by a mortgage of A’s land.
violated the condition upon which the loan was granted. It is but fair for Gary
Without the knowledge of A, C paid B the sum of P1M for A’s debt. As a
result: to demand immediate payment.

Group of answer choices No, Gary cannot compel Isaac to immediately pay the loan. The delivery of the
car as security for the loan is an accessory contract; the principal contract is
C may foreclose the mortgage on A’s land if A cannot pay. still the Php 1M loan. Thus, Isaac can still make use of the period.

C cannot claim reimbursement from A in as much as the payment was made No, Gary cannot compel Isaac to immediately pay the loan. Equity dictates that
without the knowledge of A. Gary should have granted a reasonable extension of time for Isaac to deliver
his Toyota Innova. It would be unfair and burdensome for Isaac to pay the Php
C can recover the amount from B in case A refuses the reimbursement to C. 1M simply because the promised security was not delivered.

The obligation of A to B was extinguished but A should reimburse C the


amount of P1M.
56. A, B, C and D are obliged to pay V, W, X, Y and Z, P20,000. Which is
correct?

Group of answer choices

58

0 0
V may collect from A, P20,000 The instrument may be reformed because it does not express the true
agreement of the parties.
V may collect from A, P5,000

V may collect from A, P1,000


61. D owes C P6,000. No date for payment was stipulated by the parties.
V may collect from A, P4,000 Which is correct?

Group of answer choices

57. A, a bachelor who has sired many children by different women, donated a C can require D to pay when the period arrives.
house and lot to T, his illegitimate son. The deed of donation and its acceptance
were in a public instrument and a new certificate of title to the house and lot C can require D to pay at anytime.
was issued in the name of T. Later, A discovered that T was not his son. Can A
annul the contract? D is not liable to C because the obligation is void there being no date of
payment.
Group of answer choices
D is not required to pay unless C does to court and asks the court to require D
No, because all requisites have been complied with a new title had been issued to pay.
in the name of T.

Yes, A may annul the contract on the ground of mistake as to the identity of T.
62. If the debtor gives a property to his creditor who accepts it in payment of
No, but A can ask for rescission because he suffered damages. an obligation in money, this will be governed by:
Yes, the contract is void ad initio Group of answer choices

Cession

58. The expenses of consignation, when properly made shall be charged Barter
against
Exchange
Group of answer choices
Sales
The debtor

The creditor
63. Arlene owns s row of apartment houses in Lucao, Dagupan City. She
Both the debtor and creditor agreed to lease Apartment 1 to Janet for a period of 18 months at the rate of
Php 10,000 per month. The lease was not covered by any contract. Janet
Neither the debtor nor creditor promptly gave Arlene two months deposit and 18 checks covering rental
payment for 18 months. This show of good faith prompted Arlene to promise
Janet that should Arlene decides to sell the property, she would give Janet the
right of first refusal.
59. D is obliged to give C a specific ring. The parties agreed that D may give a
specific necklace as substitute. Which of the following statements is true? Not long after Janet moved in, she received the news that her application for
Masters in Business Administration scholarship at Stanford School of Business
Group of answer choices in California had been approved. Since her acceptance of the scholarship
entailed a transfer of residence, Janet asked Arlene to return the advance rental
If the ring is lost through a fortuitous event before substitution, the obligation is
payments she made. Arlene refused, prompting Janet to file an action to
extinguished. recover the payments. Arlene filed a motion to dismiss, claiming that the lease
on which the action is based is unenforceable. If you were the judge, would
If the necklace is lost through a fortuitous event before substitution, the you grant Arlene motion?
obligation is extinguished.
Group of answer choices
If the necklace is lost through debtor’s fault before substitution, the debtor shall
pay damages. Yes, I will grant the motion because the lease contract between Arlene and
Janet was not in writing, hence, Janet may not enforce any right arising from
If the ring is lost through debtor’s fault after substitution, the debtor shall pay the same contract.
damages.
No, I will not grant the motion because to allow Arlene to retain the advance
payment would amount to unjust enrichment.

60. A and B entered into a contract of mortgage. However, as written the Yes, I will grant the motion because the action for recovery is premature; Janet
document states it is a contract of sale with right of repurchase, the error due to should first secure a judicial rescission of the contract of lease.
fault of the clerk/typist. Hence:
No, I will not grant the motion because the cause of action does not seek to
Group of answer choices enforce any right under the contract of lease.
The contract of sale must be annulled since it is voidable.

The instrument has to be enforced as is for it is the proof of the agreement


64. Statement 1 – In expromission, the debtor’s insolvency or non-fulfillment
between the parties.
of the obligation shall not give rise to any liability on the part of the original
debtor.
Because of the negligence of the parties in signing without first reading the
Statement 2 – In delegacion, the insolvency of the new debtor, shall not
instrument, they are bound by the contents of the same.
revive the action of the creditor against the original debtor.

59

0 0
Group of answer choices Group of answer choices

True, true Both statements are wrong.

True, false 1st statement is correct, 2nd statement is wrong.

False, true Both statements are correct.

False, false 1st statement is wrong, 2nd statement is correct.

70. In 2022, A, 17 years old sold his ring to B of legal age payable in 3 equal
annual payments. B paid in 2022, 2023 and 2024 as agreed upon. In 2025:
65. The following are the ways by which innominate contacts are regulated,
except: Group of answer choices

Group of answer choices A can ask for annulment

By the stipulation of the parties. B can ask for annulment

By the general principle of quasi contracts and delicts. The contract is without defect

By the rules governing the most analogous nominate contract. The contract is without defect

By the customs of the place.

71. Which phrase best completes the statement – A chattel mortgage can be
constituted to secure:
66. It is understood to be that which must necessarily come although it may not
be known when: Group of answer choices

Group of answer choices obligations both past and future.

Passing the CPA board exam c. A priest becoming a saint obligations existing at the time the mortgage is constituted.

The birth of a child future obligations only.

The birth of a child past obligations only.

A day certain

72. The holding of a conference among people remote from one another by
means of telecommunication devices such as telephone or computer terminals.
67. In culpa aquiliana, a liability of a person who is held liable for the fault or
negligence of another person who is under his care or responsibility shall cease Group of answer choices
if:
Computer Conferencing
Group of answer choices
Teleconferencing
He observed ordinary diligence to prevent the injury.
Audio Conferencing
He was not present at the time of the act.
Videoconferencing
There was contributory negligence on the part of the injured person.

The person who caused the injury is a minor.


73. The debtor shall not lose the right to make use of the period in one of the
following cases:

68. Who is entitled to the fruits and interest if any case of unilateral obligation Group of answer choices
and the condition imposed is fulfilled?
Answer I- The debtor, if the condition is suspensive When he becomes insolvent.
Answer II – The debtor, if the condition is resolutory
When he violates any undertaking in consideration of which the creditor agreed
Group of answer choices to the period.

True, true When the debtor attempts to abscond.

True, false When he does not give any guaranty or security to the creditor.

False, false

False, true 74. A, B, C and D are joint debtors of V, W, X, Y and Z, solidary creditors the
sum of P20,000. Which is correct?

Group of answer choices


69. 1st Statement – Commodatum may be gratuitous or onerous.
V may collect from B, P20,000
2nd Statement – Mutuum may be gratuitous or onerous.

60

0 0
V may collect from B, P5,000 In contracts creating real rights, third persons who come into possession of the
object of the contract are not bound thereby.
V may collect from A, P1,000

V may collect from B, P4,000


79. I. All offices and agencies providing government services shall be
subjected to a Regulatory Impact Assessment which shall be used to obtain
feedback on how provisions in the Citizen’s Charter are being followed and
75. 1st Statement – If the thing pledged is returned by the pledgee to the how the agency is performing.
pledgor or owner, the pledge is extinguished. Any stipulation to the contrary II. The Compliance Cost Analysis shall be used to obtain information and/ or
shall be void. estimates of hidden costs incurred by applicants or requesting parties to access
government services which may include, but is not limited to, bribes and
2nd Statement – The pledgee is entitled to claim from the pledgor any payment to fixers.
deficiency between the amount of the debt or obligation and the process after
the sale of the thing pledged unless there is a stipulation to the contrary. Group of answer choices

Group of answer choices Only I is true

Both statements are wrong. Only II is true

1st statement is correct, 2nd statement is wrong. Both are true

Both statements are correct. Both are false

1st statement is wrong, 2nd statement is correct.

80. It is a contract by virtue of the terms of which the parties thereto promise
and obligate themselves to enter into another contract at a future time, upon
76. To constitute a caso fortuito that would exempt a person from happening of certain events or the fulfillment of certain conditions.
responsibility, which of the following is not necessary:
Group of answer choices
Group of answer choices
Resolutory contract
It must be neither unforeseeable nor unavoidable
Option contract
The occurrence must render it impossible for the debtor to fulfil the obligation
in a normal manner Suspensive contract

The event must be independent of the human will or at least of the debtor’s Auto-contract
will.

The debtor must be free of participation in, or aggravation of, the injury to the
creditor 81. Gary is a tobacco trader and also a lending investor. He sold tobacco leaves
to Homer for delivery within a month although the period of delivery was not
guaranteed. Despite Gary’s efforts to deliver on time, transportation problems
and government red tape hindered his efforts and he could only deliver after 30
77. Which of the following is not included in the attributes of juridical days. Homer refused to accept late delivery and to pay on the ground that the
capacity? agreed term had not been complied with. As lending investor, Gary granted a
Php 1M loan to Isaac to be paid within two years from the execution of the
Group of answer choices contract. As security for the loan, Isaac promised to deliver to Gary his Toyota
Innova within seven days but Isaac failed to do so. Gary was thus compelled to
Juridical capacity is inherent in every natural person and therefore it is not demand payment for the loan before the end of the agreed two-year term.
acquired.
Was Homer justified in refusing to accept the tobacco leaves?
Juridical capacity is lost through death.
Group of answer choices
Juridical capacity is the fitness to be the subject of legal relations.
Yes, Homer was justified in refusing to accept the tobacco leaves. The delivery
Juridical capacity cannot exist without capacity to act. was to be made within a month. Gary’s promise of delivery on a “best effort”
basis made the delivery uncertain. The term therefore was ambiguous.

No, Homer was not justified in refusing to accept the tobacco leaves. He
78. Which of the following statements is wrong? consented to the terms and condition of the sale and must abide by it.
Obligations arising from contracts have the force of law between the
Group of answer choices
contracting parties.
Creditors are protected in cases of contracts entered to defraud them.
Yes, Homer was justified in his refusal to accept delivery. The contract
contemplates an obligation with a term. Since the delivery was made after 30
Contracts take effect only between the parties, their assign and heirs, except in
days, contrary to the terms agreed upon, Gary could not insist that Homer
case where the rights and obligations arising from the contract are not
accepts the tobacco leaves.
transmissible by their nature, or by stipulation or by provision of law.
No, Homer was not justified in refusing to accept the tobacco leaves. There
If a contract should contain some stipulation in favor of a third person, he may
was no term in the contract but a mixed condition. The fulfillment of the
demand its fulfillment provided he communicated his acceptance to the obligor
condition did not depend purely on Gary’s will but on other factors, e.g., the
before its revocation.
shipping company and the government. Homer should comply with his
obligation.

61

0 0
Obligations of bailees in commodatum.

82. Rachelle borrowed P1,000,000 from Madeline, secured by a real estate Liability of principal to a common agent.
mortgage over Rachelle’s lot in Baguio City valued at P1,500,000. When
Rachelle failed to pay Madeline, Madeline decided to extra judicially foreclose Liability of partners arising out of contract.
the real estate mortgage and caused the public sale of Rachelle’s lot where
Alexis was the highest bidder. Until when can Rachelle redeem the property
from Alexis?
87. G, guardian of M, 17 years old, sold the palay harvested from the land
Group of answer choices belonging to M for P37,500. The palay had a value of P50,000. M wants to
recover the damages he suffered under the contract entered into by his
3 months after the foreclosure sale guardian. What remedy is available to M?

Registration of the certificate of sale with the Registry of Deeds Group of answer choices

Registration of the certificate of sale with the Registry of Deeds which shall in Annulment because M was only 17 years old at the time the sale was made.
no case be more than 3 months after the foreclosure sale
Rescission because M suffered lesion of P12,500.
One year from the date of registration of the sale
Either rescission or annulment.

Neither rescission or annulment.


83. Which of the following contracts is void?

Group of answer choices


88. Cruz lent Jose his car until Jose finish his CPA Licensure Examination
A minor selling the ring of her mother without authority from her mother. (CPALE). Soon after Cruz delivered the car, Jose brought it to Toyota
Dagupan for maintenance check up and incurred a cost of Php P18,000. Seeing
A widow selling the land of her father with an oral authority from her father. the car’s peeling and faded paint. Jose also had the car painted for Php 20,000.
After the CPALE, Cruz asked for the return of his car. Jose said he would
A guardian selling for P70,000 the necklace of her ward where the fair market return it as soon as Cruz has reimbursed him for the car maintenance and
value of the necklace is P100,000. repainting cost of P38,000. Is Jose’s refusal justified?

A contract of sale between two deaf-mutes who both don’t know how to read Group of answer choices
or write.
No Jose’s refusal is not justified. In this kind of contract, Jose is obliged to pay
for all the expenses incurred for the preservation of the thing loaned.

84. Which of the following defense is ‘meritorious’? Yes, Jose’s refusal is justified. He is obliged to pay for all the ordinary and
extraordinary expenses, but subject to reimbursement from Cruz.
Group of answer choices
Yes, Jose’s refusal is justified. The principle of unjust enrichment warrants the
An action for recovery of down payment paid under a rescinded oral sale of reimbursement of Jose’s expenses.
real property.
No Jose’s refusal is not justified. The expenses he incurred are useful for the
A defense in an action for ejectment that the lessor verbally promised to extend preservation of the thing loaned. It is Jose’s obligations to shoulder these useful
or renew the lease. expenses.

An action for payment of sum of money filed against one who orally promised
to answer another’s debt in case of the latter defaults.
89. The following contracts are valid, except:
A defense in an action for damages that the debtor has sufficient, but
unliquidated asset to satisfy the credit acquired when it becomes due. Group of answer choices

A contract where a party gave his consent while in a state of drunkenness.

85. Ernesto donated a mobile phone worth P32,000 to Hubert orally and A contract where a party gave his consent because the other party threatened to
delivered the unit to Hubert who accepted it. Which statement is the most sue him for an unpaid debt.
accurate?
A contract where A gave his consent because B threatened to kill A’s spouse.
Group of answer choices
A contract where the wife sells her property to her husband because of
The donation is void and Ernesto may get the mobile phone back. intimidation.

The donation is void but Ernesto cannot get the mobile phone back.

The donation is voidable and maybe annulled. 90. P appointed A1 and A2 as his agents for a common transaction and they
agreed that they shall be solidarily liable to P for damages in case of violations
The donation is valid. of their obligations as such. When is the agent at fault the only one liable for
damages even if solidarity has been stipulated?

Group of answer choices


86. Instance where the law imposes joint liability:
If the other agent is not at fault.
Group of answer choices
Both of them shall be liable always since solidarity was agreed upon.
Obligations arising from tort.

62

0 0
If the one at fault shall answer for all the damages. Group of answer choices

If the one at fault acted in excess of authority. Only I is true

Only II is true

91. The following are the requisites before a contract entered into in fraud of Both are true
creditors maybe rescinded, except:
Both are false
Group of answer choices

There must be credit existing prior to the celebration of the contract.


96. When the condition has been imposed with the intention of suspending the
There must be fraud, or at least, the intent to commit fraud to the prejudice of efficacy of an obligation to give, the following rules shall be observed, except
the creditor seeking rescission. if the thing is:

The creditor cannot in legal manner collect his credit (subsidiary character of Group of answer choices
rescission).
Lost, the obligation shall be extinguished
The object of the contract must be legally in the possession of a 3rd person in
good faith. Improved by nature, the improvement shall inure to the benefit of the creditor.

Improved at the expense of the debtor, he shall have no other right than that
granted to the usufructuary
92. A document issued by the Bureau of Fire Protection (BFP) to a fire victim
in lieu of the Final Investigation Report (FIR) for purposes of insurance claims Deteriorates without the fault of the debtor, the impairment is to borne by the
and other lawful applications. creditor.

Group of answer choices

Business Registration 97. A request or application classified as a complex transaction that poses a
danger to public health must be processed by the concerned government
Fire Safety Inspection Certificate (FSIC) agency under the Ease of Doing Business Act within how many working days?

Certificate of Fire Incidents for Fire Insurance Group of answer choices

Fire Safety Evaluation Clearance (FSEC) 3

93. A and B agreed on a contract of pledge. However, they entered into a 7


contract of mortgage in the honest belief that the mortgage and pledge are the
same. The instrument may be reformed. 20

Group of answer choices 45

No, the document cannot be reformed since the contract is void.

No, it cannot be changed. They are bound by the document which speaks for 98. In payment of 10 grams of shabu , A made a promissory note which reads
itself. “I promise to pay P10,000. Sgd. A”. B transferred the note to C, who acted in
good faith. Which is correct?
Yes, it can be reformed because it does not express the true agreement of the
parties. Group of answer choices

No, it cannot be reformed because there was no meeting of the minds between C can collect from A
the parties and the remedy is annulment of contract.
B can collect from A
94. A and B entered into a contract whereby A was to give B P2M. Later, they
C can collect from either A or B
novated the contract by stipulating that instead of cash, A would give a
particular car. Subsequently, the car was destroyed by a fortuitous event.
C can collect from B
Which is correct?

Group of answer choices


99. A owes B two debts. The first debt is secured by a mortgage, the second is
B was the option to demand either P2M or the value of the car.
not. A tells B that the payment he is making should be applied to the second
debt instead of the first. Which is correct?
A is liable to pay P2M, the amount of the old obligation.
Group of answer choices
The original obligation is extinguished but not the obligation to deliver the car.
B may refuse such application on the ground that the first debt is more
The original obligation and the obligation to deliver the car are both
extinguished. burdensome to the debtor.

B may refuse such application because the payment shall be applied


proportionately.
95. I. Business permits shall be valid for a period of one (1) year.
B cannot refuse the application because of the option of the debtor.
II. The city/ municipality may have the option to renew business permits
within the first month of the year or on the anniversary date of the issuance of
B cannot refuse the application because it is to his benefit.
the business permit.

63

0 0
REG 1 FINAL EXAM

PART 1
Question 1
D owes C P500,000. Before the debt was paid, D died leaving his only son, B. Which is true?
If the value of the properties left by D is P1,000,000, B is obliged to pay the entire P1,000,000
to C.
If the value of the properties left by D is P400,000, B is has to pay the entire P400,000 and
make good the deficiency of P100,000.
Correct!
If the value of the properties left by D is P500,000, B is obliged to pay the entirety to C.
If B did not inherit anything from D and B is very rich in his own right, B can be compelled by
C to pay the debt of his father.

Question 2
G was appointed as the guardian of M who owns a car valued at P1M. M orally sold the car only
for P0.7M to B with 10% down-payment already paid. What is the status of the contract?
Voidable
Unenforceable
Valid
Rescissible
Answer not given

Question 3
If it is not clear whether a contract is contract of lease or sale, it shall be interpreted as
Contract of lease
The contract is void
Answer not given
Contract of sale

0 0
Question 4
The following are the requisites for action to rescind contracts in fraud of creditors, except:
The creditor has other legal remedy to satisfy his claim
The plaintiff asking for rescission has a credit prior to the alienation, although demandable later
The act being impugned is fraudulent
The debtor has made a subsequent contract conveying a patrimonial benefit to a third person
Answer not given

Question 5
Statement 1: When the price of any article or commodity is determined by statute, or by
authority of law, any person paying any amount in excess of the maximum price allowed cannot
recover such excess.
Statement 2: When the law sets, or authorizes the setting of a minimum wage for laborers, and a
contract is agreed upon by which a laborer accepts a lower wage, he cannot recover the
deficiency.
False, true
True, false
False, false
True, true

Question 6
If it is not clear whether a contract is contract of commodatum or donation, it shall be interpreted
as
Contract of commodatum
Answer not given
Contract of donation
The contract is void

Question 7

0 0
When two or more prestations have been agreed upon but only one is due, as a general rule, the
right choice belongs to whom?
To the creditor
To the debtor
To both the creditor and debtor
To third person

Question 8
Legal subrogation is presumed in the following. Which is not correct?
When a third person, not interested in the obligation, pays with the express or tacit approval of
the debtor.
When a third person, not interested in the obligation, pays with the approval of the creditor.
When, even without the knowledge of the debtor, a person interested in the fulfillment of the
obligation pays, without prejudice to the effects of confusion as to the latter’s share.
When a creditor pays another creditor who is preferred, even without the debtor’s knowledge.

Question 9
Which of the following may not be reformed?
Void contract
All choices may not be reformed
Unenforceable contract
Voidable contract
Rescissible contract

Question 10
An obligation wherein various things are due but the complete performance of all of them is
necessary to extinguish the obligation
Conjoint obligation
Facultative obligation
Alternative obligation

0 0
Simple obligation

Question 11
Which of the following is not a characteristic of a joint indivisible obligation?
The creditor must proceed against all the debtors.
If any of the debtor be insolvent, the others shall not be liable for his share.
Each of the debtor can be held liable to pay for the entire obligation.
Demand must be made to all the debtors.

Question 12
In contract of donation of a personal property worth P5,000, which of the following is least likely
the requisite?
Must be in public instrument
All of the choices are requisites
There must be delivery
Donee should consent

Question 13
The obligation of the employer to pay death benefits and funeral expenses for his employee’s
death while in the course of employment as sanctioned by the Workmen’s Compensation Act is
one that arises from:
Anser not given
Quasi-contracts
Law
Contracts

Question 14
R orally rented the car of K for 13 months. The car was delivered to R. What is the status of their
contract?

0 0
Valid
Answer not given
Unenforceable
Voidable
Rescissible

Question 15
In case a government agency fails to approve or disapprove an original application within the
prescribed processing time, the said application shall be __.
Deemed approved
Deemed denied
Deemed pending
Returned to the filer

Question 16
Facultative as distinguished from alternative obligation
If one of the prestations is illegal, the others may be valid and the obligation remains.
If one of the prestations is illegal, the others may be valid and the obligation remains.
Various things are due, but the giving of one is sufficient.
The right of choice is given only to the debtor.

Question 17
Simple transactions with government agencies must be processed within __ working days:
1
3
7
5

0 0
Question 18
G was appointed as the guardian of M who owns a parcel of land valued at P1M. G sold the land
only for P0.75M to B. What is the status of the contract?
Voidable
Answer not given
Rescissible
Valid
Unenforceable

Question 19
What is the basis of the liability of a school when a student is stabbed inside the campus by a
stranger in the school?
Contracts
Quasi-contracts
Quasi-delicts
Delicts

Question 20
Debtor Beth in indebted to Jane, Karla and Babe a total amount of P300,000 made up of: Jane –
P100,000; Karla – P50,000 and Babe – P150,000. On maturity, Beth cannot pay his obligation so
he assigns or cedes to them all his property, to be sold by the creditors and the proceeds thereof
applied to their corresponding credits. The creditor’s sold Beth’s properties for only a total of
P180,000. Is the obligation of Beth extinguished? Decide.
Yes, the obligation is extinguished.
Answer not given
Jane will get P60,000; Karla – P30,000; and Babe – P90,000.
Jane, Babe and Karla will divide the P180,000 equally.

Question 21
The right of the creditor that is enforceable against a definite debtor is

0 0
Moral right
Real right
Natural right
Personal right

Question 22
Unless ratified, suits will not prosper if the contract is
Unenforceable
Rescissible
Void
Voidable

Question 23
Which government agency is mandated to establish a central business portal to receive and
capture application data on business-related transactions, and provide links to online registration
of national government agencies?
DTI
DICT
DOTr
DOF

Question 24
The debtor shall lose every right to make use of the period, except
When through fortuitous events or by his own acts the guaranties or securities have been
impaired, unless he immediately gives new ones equally satisfactory.
When after the obligation has been contracted, he becomes insolvent, unless he gives a guaranty
or security for the debt.
When through fortuitous events or by his own acts the guaranties or securities have been
impaired, unless he immediately gives new ones equally satisfactory.
When he does not furnish the creditor the guarantees or securities which he has promised.

0 0
Question 25
The proper remedy is annulment of contract and not reformation when:
Answer not given.
A mutual mistake of the parties causes the failure of the instrument to disclose their real
agreement.
Mistake, fraud, inequitable conduct, or accident has prevented a meeting of minds of the
parties.
One party was mistaken and the other knew or believed that the instrument did not state their
real agreement, but concealed the fact from former.

Question 26
A contract of loan involving an amount of P501 must appear in writing for what purpose?
Enforceability
Convenience
Answer not given
Validity

Question 27
Statement 1: The action or defense for the declaration of the inexistence of a contract does not
prescribe.
Statement 2: When the nullity proceeds from the illegality of the cause or object of the contract,
and the act constitutes a criminal offense, both parties being in pari delicto
True, false
False, true
True, true
False, false

Question 28

0 0
Statement 1: Interest paid in excess of the interest allowed by the usury laws may be recovered
by the debtor, with interest thereon from the date of contract.
Statement 2: When money is paid or property delivered for an illegal purpose, the contract may
be repudiated by one of the parties before the purpose has been accomplished, or before any
damage has been caused to a third person.
False, true
False, false
True, false
True, true

Question 29
There shall be no reformation in the following cases:
When the real agreement is void
There will be no reformation on all choices
Simple donations inter vivos wherein no condition is imposed
Wills

Question 30
A oblige himself to pay B P100,000 in 30 days plus a penalty of P10,000 if A fails to pay the
obligation in due time. A failed to pay the obligation in 30 days, B can demand from A
The principal of P100,000 plus P10,000 penalty plus legal interest.
The principal of P100,000 plus P10,000 penalty.
The principal of P100,000 plus legal interest, plus damages.
The principal of P100,000 plus P10,000 penalty, plus legal interest, plus damages.

Question 31
This will result to a solidary liability
Quasi-delict committed by one of the partners acting in the ordinary course of business.
Insolvency of one of the debtors.
Default on the part of one of the debtors.

0 0
Vitiated consent on the part of one of the debtors.
All answers are correct

Question 32
The act of offering the creditor what is due him together with a demand that the creditor accepts
the same.
Application of payment
Dation in solutum
Tender of payment
Cession

Question 33
Which of the following contracts is voidable?
Those where both parties are incapable of giving consent to a contract
Those undertaken in fraud of creditors when the latter cannot in any manner collect the claims
due them
Those where the consent is vitiated by mistake, violence, intimidation, undue influence or fraud
All of the choices are voidable contracts
Those whose object is outside the commerce of men

Question 34
A offered to sell his car for P500,000 to B. A stated that he was giving B a period of one week
within which to raise the amount, and that as soon as B is ready, they will sign the deed of sale.
A had been given P5,000 by b in consideration for the option. Which of the following is not
correct?
A cannot sell his car to another because of the money given by B.
A can sell the car only after one week.
A can still sell the car before the acceptance is made known to him by B.
A cannot sell the car until after one week.

0 0
Question 35
The bargaining point, that is, when negotiation is in progress (stage in the life of a contract)
Perfection
Birth
Consummation
Conception

Question 36
A and B joint debtors of C and D, solidary creditors, to the amount of P1,000 – C can demand
P500 from A or P500 from
P500 from A and P500 from B
P1,000 from A or P1,000 from B
P250 from A and P250 from B

Question 37
Which of the following may not be subject to reformation?
When one party was mistaken and the other knew or believed that the instrument did not state
their real agreement, but concealed that fact from the former
When a mutual mistake of the parties causes the failure of the instrument to disclose their real
agreement
If one party was mistaken but the other did not acted fraudulently or inequitably which
nevertheless resulted to the instrument apparently not showing their true intention
You Answered
All of the choices may be subject to reformation

Question 38
A sold to B his cow for P5,000. No date is fixed by the parties for the performance of their
respective obligations. The obligation of A is
To deliver the cow upon the payment by B of P5,000
To deliver the cow immediately as there is a perfected contract.

0 0
To deliver the cow within the reasonable time from the perfection of the contract.
To rescind the contract as there is no time fixed for the delivery and payment.

Question 39
Rescission of contract can take place in this case
When the party seeking resolution can perform only as to part and as to remainder
When the thing which is the object of the contract is legally in the possession of a third person
who acted in bad faith
When he who demands rescission can return whatever he may be obliged to restore
When the seller cannot return the installments paid to him by the buyer

Question 40
On June 2020, which is a leap year, R rented the apartment of K for 366 days. What is the status
of their contract?
Rescissible
Unenforceable
Valid
Voidable
Answer not given

Question 41
Consent is manifested by the meeting of the offer and the acceptance upon the thing and the
cause which are to constitute the contract. Which of the following constitute an offer?
Answer not given.
An offer made to an agent.
Advertisement for bidders.
Business advertisement of things for sale.

Question 42

0 0
What shall provide NGAs/LGUs access to data and information to verify the validity and
existence of business entities?
Philippine Business Databank

Philippine Business Database


Philippine Business Portal
Philippine Business Data Registry

Question 43
The action to annul a voidable contract, such as a contract where one of the parties is incapable
of giving consent to the contract, is extinguished by:
Novation
Ratification
Answer not given
Rescission

Question 44
D forced C to execute a promissory note
The contract is void
Contract is rescissible because the contract is fraudulent
C cannot demand payment from D because the contract is unenforceable
Contract remains to be valid

Question 45
Entails the return of the benefits that each party may have received as a result of the contract.
Ratification
Annulment
Mutual restitution
Rescission

0 0
Question 46
In order for a sale of land through an agent to be valid, the contract of sale and agency must be
in? Choose the minimum requirement.

Oral and writing respectively


Writing and oral respectively
Public instrument and writing respectively
Writing and writing respectively

Question 47
Where demand by the creditor shall be necessary in order that delay may exist.
When the obligor has expressly acknowledged that he is in default.
When the obligor requested for an extension of time.
When time is of the essence of the contract.
When demand would be useless.

Question 48
In case both parties have committed a breach of the obligation:
The second infractor may choose between rescission or fulfillment.
The first infractor is more liable than the second.
Each party must bear his own damage.
The liability of the first infractor shall be equitably tempered by the courts.

Question 49
G was appointed as the guardian of M who owns a car valued at P1M. M orally sold the car only
for P0.7M to B. What is the status of the contract?
Answer not given
Rescissible
You Answered
Voidable

0 0
Valid
Unenforceable

Question 50
The following are some accidental elements of a contract of sale, except
Place of payment
Terms of payment
Interest rate
Accession and accessories in specific real obligations
Answer not given

Question 51
The court determines the amount of damage taking into consideration the price of the thing and
its sentimental value to the injured person.
Restitution
Starvation
Indemnification
Reparation

Question 52
It causes the extinguishment or loss of rights already acquired upon the fulfillment of the
condition, that is, the happening of the event which constitutes the condition. In other words, the
fulfillment of which will extinguish an obligation (or right) already existing.
Condition precedent
Condition subsequent
Facultative condition
Suspensive condition

0 0
Question 53
Although validly agreed upon, courts can nullify this contract because of damage to one of the
parties or to a third person and its enforcement, may cause injustice by reason of some external
facts
Void contracts
Voidable contracts
Rescissible contracts
Unenforceable contracts

Question 54
This is synonymous to solidary obligation
Pro-rata
Mancomunada
Proportionate
Juntos o separadamente

Question 55
The thing itself shall be restored, as a rule
Indemnification
Starvation
Reparation
Restitution

Question 56
R orally rented the car of K for 13 months. What is the status of their contract?
Voidable
Rescissible
Answer not given

0 0
Unenforceable
Valid

Question 57
The cession of actions or rights proceeding from an act appearing in a public document must be
in public document for what purpose?
Validity
Answer not given
Convenience
Enforceability

Question 58
The following is considered fraud or fraudulent.
Failure to disclose facts when there is duty to reveal them.
The usual exaggeration in trade, when the other party had the opportunity to know the facts.
“Caveat Emptor” of let the buyer beware.
Misrepresentation made not in bad faith.

Question 59
The penalty is separate/independent from damages and interest, except:
When the debtor employed fraud
When the debtor refuses to pay the obligation
When it is stipulated that penalty is separate/independent from damages and interest
When the debtor is in delay

Question 60
A contract where the cause with respect to each contracting party is the prestation or the promise
of a thing or service by the other is
An aleatory contract

0 0
An innominate contract
A gratuitous contract
An onerous contract

Question 61
A juridical relation known as negotiorum gestio takes place
Answer not given
When a person is appointed by a court to take the property or business of another.
When a person voluntarily takes charge of another’s abandoned business or property with the
owner’s consent.
When something is received and there is no right to demand it and it was delivered through
mistake

Question 62
Which of the following transactions must be processed within 20 working days?
Basic transactions
Simple transactions
Complex transactions
Highly technical transactions

Question 63
A is obliged to give B 10 kilos of sugar, which of the following is not correct?
B may require another person to deliver the sugar and charge the expenses to A.
B can demand that A obtain the sugar and deliver it to him.
B can just buy 10 kilos of sugar and charge the expense to A.
A can insist on paying B damages or the monetary value of the sugar.

Question 64

0 0
Every obligation whose performance does not depend upon a future or uncertain event, or upon a
past event unknown to the parties, is demandable at once. This refers to
Pure obligation

Joint and solidary obligations


Obligations with a period.
Divisible and indivisible obligations.

Question 65
A owes B P10,000 to be paid after one year. As security A mortgages his car in favor of B. If
before due date the car is lost by fortuitous event:
A does not lose the benefit of the period because he is not in delay.
A does not lose the benefit of the period because it was not stipulated.
A does not lose the benefit of the period because no one shall be liable for fortuitous event.
A does not lose the benefit of the period if he gives another security equally satisfactory.

Question 66
Substitution of debtor where the initiative comes from the debtor.
Expromission
Subrogation
Delegacion
Novation

Question 67
The following statements pertain to the cause of a contract. Which statement refer to motive?
It is illegality does not affect the validity of the contract
there will be no contract without it
It is the essential reason of a contract.
It is always known to the contracting parties

0 0
Question 68
A is indebted to solidary creditors B, C, and D, for P90,000. Without the knowledge of B and C,
D remitted the obligation of A, as a result

The obligation is extinguished only up to P30,000.


The obligation is extinguished up to P60,000.
The obligation is not extinguished because there is no consent from B and C.
The obligation of A to pay P90,000 is extinguished.

Question 69
Where a property is alienated to the creditor in satisfaction of a debt in money.
Dation in payment
Payment by cession
Consignation
Application of payment

Question 70
A delivered his bracelet to B for B’s necklace. No written agreement was signed by the parties.
Which of the following is appropriate description of the contract between A and B?
Bilateral and innominate
Bilateral and innominate
Onerous and bilateral
Aleatory and nominate

Question 71
Which of the following is a civil obligation?
The obligation of a catholic to hear mass every Sunday.
X obliges himself to pay Y P10,000 on October 30, 2020.
The obligation of a husband and wife to observe fidelity.
A is a debtor of B for P20,000 due on September 30, 1995.

0 0
Question 72
This happened when the creditor makes a demand and the obligor fails to deliver the thing.
Ordinary delay
Mora solvendi
Legal delay
Mora accipiendi

Question 73
Statement 1: In case of a divisible contract, if the illegal terms can be separated from the legal
ones, the latter may be enforced.
Statement 2: The defense of illegality of contract is not available to third persons whose interests
are not directly affected.
False, false
True, true
False, true
True, false

Question 74
Which of the following is true with regards to the annulment of voidable contracts due to fraud?
Answer not given
May be annulled within 4 years from date of contract
May be annulled within 10 years from date of contract
May be annulled within 10 years from date of contract

Question 75
The principle that contracting parties may establish such stipulations, clauses, terms and
conditions as they may deem convenient, provided they are not contrary to law, moral, public
policy and public order, refer to the principle of

0 0
Relativity of contract
Mutuality of contract
Liberty of contract
Consensuality of contract

PART 2
Question 1
1 / 1 pts
D borrowed P20.000 from C. To secure the fulfillment of the loan, D mortgaged a land owned by
his ailing father. Which of the following statements is correct?
The contract of loan is null and void because the contract of mortgage is null and void.
The contract of mortgage is null and void because the mortgagor must be the owner of the
property mortgaged at the time it is constituted but the contract of loan remains to be valid.
The contract of mortgage is valid because future property may be pledged or mortgaged.
The contract of mortgage will become valid upon the death of D’s father.

Question 2
In case the cause or consideration of the contract of pledge is not stipulated, what is the cause or
consideration of a contract of pledge?
The cause or consideration is the liberality of the pledgor.
The cause or consideration of the principal obligation or contract of loan.
It has no cause or consideration.
The cause is the service remenerated.

Question 3
It refers to the right of mortgagor to redeem the mortgaged property after his default of the
performance of his obligation by paying the obligation but before the property is sold in order to
prevent the public sale.
Equity of preemption
Right of redemption

0 0
Right of preemption
Equity of redemption

Question 4
How long is the period of suspension for government officials or employees violating the Ease of
Doing Business Act for the first time?
9 months
6 months
3 months
1 year

Question 5
What is the nature of a contract of pledge, of chattel mortgage, of real estate mortgage or of
antichresis?
It is indivisible whether the principal contract is joint or solidary.
It is divisible whether the principal contract is joint or solidary.
If is divisible if the principal contract is joint.
It is indivisible if the principal contract is solidary.

Question 6
Indicate the proper order on how the proceeds of sale from judicial or extrajudicial foreclosure of
Chattel Mortgage shall be distributed:
I. Cost of Sale.
II. Claim of the person foreclosing the mortgage.
III. Claims of junior encumbrances in the order of their priority.
IV. Mortgage or his agent.
III – II – I – IV
II – I – IV – III
I – II – III – IV

0 0
I – III – II – IV

Question 7
Indicate the proper order on how the proceeds of sale from judicial or extrajudicial foreclosure of
Real Estate Mortgage shall be distributed:
I. Cost of Sale.
II. Claim of the person foreclosing the mortgage.
III. Claims of junior encumbrances in the order of their priority.
IV. Mortgagor or his agent.
I – II – III – IV
III – II – I – IV
II – I – IV – III
I – III – II – IV

Question 8
In case the proceeds of the sale from judicial or extrajudicial foreclosure of Real Estate Mortgage
is higher than the principal obligation and costs, which is true?
The mortgagor shall always recover the excess.
The mortgagee shall always retain the excess.
The mortgagee may retain the excess in the absence of stipulation to the contrary.
The mortgagor is entitled to the excess in the absence of stipulation to the contrary.

Question 9
In case of judicial foreclosure, if upon the trial in such action the court shall find the facts set
forth in the complaint to be true, it shall ascertain the amount due to the plaintiff upon the
mortgage debt or obligation, including interest and other charges as approved by the court, and
costs, and shall render judgment found due and order that the same be paid to the court or to the
judgment obligee. This is known as equity of redemption of judgment debtor and the amount
shall be paid by the judgment debtor. What is exercise of equity of redemption in case of judicial
foreclosure?
Within a period of not less than I year nor more than 2 years from the entry of judgment.

0 0
Within a period of not less than 90 days nor more than 1 year from the entry of judgment.
Within a period of not less than 90 days nor more than 120 days from the entry of judgment.
Within a period of not less than 120 days nor more than 1 year days from the entry of judgment.

Question 10
The following requisites are essential to the contracts of pledge, real estate mortgage and chattel
mortgage, except
That when the principal obligation becomes due, the things in which the pledge or mortgage
consists may be alienated for the payment of the creditor.
That the pledgor or mortgagor be the absolute owner of the thing pledged or mortgaged.
That the thing pledged or mortgaged be placed in the possession of the creditor, or of a third
person by common agreement.
That the persons constituting the pledge or mortgage have the free disposal of their property,
and in the absence thereof, that they be legally authorized for the purpose.
That they be constituted to secure the fulfillment of a principal obligation such as contract of
loan.

Question 11
It is a contract whereby the debtor secures to the creditor the fulfillment of a principal obligation,
especially subjecting to such security, immovable property or real rights over immovable
property in case the principal obligation is not complied with at the time stipulated. In this
contract, the creditor acquires the right to receive the fruits of an immovable of his debtor, with
the obligation to apply them to the payment of the interest, if owing, and thereafter to the
principal of his credit.
Real mortgage
Chattel mortgage
Pledge
Antichresis

Question 12
D borrowed P1,000,000 from C. G, a third person, mortgaged his land to secure the fulfillment
of D’s loan. Is the contract of mortgage valid?

0 0
Yes because third persons who are not parties to the principal obligation may secure the latter
by pledging or mortgaging their own property.
No because D must be the owner of the mortgaged land.
No because G is no privy to the contract of loan.
Yes provided G will deliver the land to C.

Question 13
The following are the important characteristics of contract of antichresis, except
Indivisible - It creates a lien on the whole or all of the properties mortgaged, which lien
continues until the obligation is secures has been fully paid.
Real right - It creates a lien on the property mortgaged.
Real contract - It is perfected by the delivery of the tiling mortgaged.
Inseparable - It subjects the property upon which it is imposed, whoever the possessor may be,
to the fulfillment of the obligation for whose security it was constituted.
Accessory - It cannot exist without a principal obligation or contract of loon.

Question 14
The following are the formalities required for the sale of the thing pledged in case of failure of
the debtor to fail the principal obligation, except
It must be by public auction.
It must be sold at the first auction.
There must be a notice to the debtor and the owner of the thing pledged, stating the amount for
which the public sale is to be held.
It must be through a notary public.

Question 15

0 0
It is a type of pledge which refers to the right of a person to retain a thing until he receives
payment of his claim.
Voluntary pledge
Conventional pledge
Legal pledge or pledge by operation of law
Agreed pledge

Question 16
Which of the following is not a function of the Anti-Red Tape Authority?
Initiate investigation motu propio only or file cases for violations
Monitor compliance of agencies and issue notices to erring and non-compliant government
employees and officials
Review proposed major regulations of government agencies, upon submitted regulatory impact
assessments
Implement and oversee national policy on anti-red tape and ease of doing business and
implement reforms to improve competitiveness ranking

Question 17
It is a contract whereby the debtor or third person secures to the creditor the fulfillment of a
principal obligation, specially subjecting to such security immovable property or real rights over
immovable property in case the principal obligation is not complied with at the time stipulated.
Real estate mortgage
Antichresis

Pledge
Chattel mortgage

Question 18
It refers to the right of the mortgagor to repurchase the property within a certain period after it
was sold in public auction for the payment of the mortgage debt.
Right of preemption
Equity of preemption

0 0
Equity of redemption b Right of redemption
Equity of preemption

Question 19
Which of the following statements pertains to equitable mortgage?
It is one which although lacks certain formality, form or words or other requisites provided by
statute, shows the intention of the parties to charge the real property as a security for a debt and
contains nothing contrary to law.
It is one executed pursuant, to an express requirement of a provision of law.
It is one which is created by the agreement of the parties.

Question 20
It is a type of foreclosure made through the filling of a petition in court under Rule 68 of Rules of
Court and availed of when the deed of real estate mortgage does not provide for special power of
attorney (SPA) authorizing the mortgagee-creditor to foreclosure it extrajudicially.
Extra-judicial foreclosure
Judicial foreclosure
Legal foreclosure
Conventional foreclosure

Question 21
What is the form of contract of pledge to bind or to affect third persons?
It must be recorded in the Intellectual Property Office.
It must be registered in the chattel mortgage registry.
It must be in a public instrument showing a description of the thing pledged and the date of the
pledge.
It must be registered in the registry of deeds.

Question 22
Who chairs the Ease of Doing Business/Anti-Red Tape Advisory Council?

0 0
DILG Secretary
DOF Secretary
DTI Secretary
DICT Secretary

Question 23
Upon non-payment or default of the antichretic debtor of the principal obligation, may the
antichretic creditor automatically appropriate the real property used as security?
Yes if it is agreed upon by the parties.
Yes because the prohibition against pactum commissorium is not applicable to contract of
antichresis.
No because it is pactum commissorium which is prohibited by law and public policy.
Yes because it is the right of the creditor.

Question 24
The Ease of Doing Business/Anti-Red Tape Advisory Council is composed of how many
persons?
9
7
5
3

Question 25
Who appoints the two (2) representatives from the private sector to sit in the Ease of Doing
Business/AntiRed Tape Advisory Council?
Executive Secretary
DTI Secretary
DOF Secretary
President

0 0

You might also like